Uworld Notes CK PDF
Uworld Notes CK PDF
The most beneficial therapy to reduce the progression of diabetic nephropathy is strict blood
pressure control.
Vitamin D deficiency most commonly causes osteomalacia and can be due to malabsorption,
intestinal bypass surgery, celiac sprue, or chronic liver or kidney disease. Patients can be
asymptomatic or present with bone pain, muscle weakness or cramps, and difficulty walking.
Hypocalcemia and low vitamin D levels lead to secondary hyperparathyroidism, which brings
the serum calcium levels to normal or near-normal by increasing bone calcium resorption and
renal calcium reabsorption
Increased serum calcium, low serum phosphate, and increased serum PTH levels are seen in
patients with primary hyperparathyroidism
Normal serum calcium, normal serum phosphate, and normal serum PTH levels are seen in
patients with osteoporosis and Paget's disease. In osteoporosis, bone pain (without fracture) and
muscle weakness are not present. The serum alkaline phosphatase level is markedly elevated in
Paget's disease. Hypercalcemia can occur in patients with moderately severe Paget's disease
following immobilization.
An increased extracellular pH (due to respiratory alkalosis in this patient) causes hydrogen ions
to dissociate from albumin molecules, thereby freeing up the albumin to bind with calcium. This
increase in the affinity of albumin for calcium leads to decreased levels of ionized calcium
If left untreated, patients with hyperthyroidism can develop rapid bone loss leading to
osteoporosis and increased risk of fracture. Initial treatment of TA and MNG includes a beta
blocker to alleviate the symptoms of hyperthyroidism and a thionamide (eg, methimazole,
propylthiouracil) to decrease thyroid hormone secretion. Options for definitive management of
TA include surgery and radioiodine ablation
Fetal hyperthyroidism can be seen in patients with active Graves disease, in whom TSH receptor
(TSHR) antibodies can cross the placenta and affect the fetal thyroid.
Parathyroidectomy is recommended for patients with symptomatic hypercalcemia. It is also
recommended for those with complications (eg, osteoporosis, nephrocalcinosis, nephrolithiasis,
impaired renal function) or at significant risk for complications (eg, serum calcium >1 mg/dl
above normal, urinary calcium excretion >400 mg/24 hr). In addition, younger patients (age <50)
are likely to have complications during their lifetime and should be offered surgery
G6PD is responsible for catalyzing the reduction of NADP to NADPH, the first step in the HMP
shunt. This is the only source of NADPH in RBCs. NADPH is necessary to form reduced
glutathione, which protects RBCs from oxidative injury. In the absence of G6PD and in the
presence of oxidizing agents (eg, bacterial toxins, sulfa-containing drugs), hemoglobin becomes
oxidized to form methemoglobin, denatured globin, and sulfhemoglobin. These molecules form
insoluble masses (Heinz bodies) that attach to the RBC membrane, decrease membrane pliability,
and promote RBC removal in the spleen’s RES
Diagnosis of DKA, three things are necessary: blood glucose level >250 mg/dl, pH < 7.3 or low
serum bicarbonate (< 15- 20 mEq/L), and detection of plasma ketones. The most appropriate
initial management is rapid, intravenous administration of normal saline and regular insulin.
Essential measures in the management of DKA include the following:
1. restoration of intravascular volume: using 0.9% saline (normal saline)
2. correction of hyperglycemia using intravenous regular insulin
3. correction of electrolyte abnormalities (Potassium correction is very crucial)
4. treatment of precipitating factors such as infections: using antibiotics
NPH insulin is not appropriate for the initial treatment of DKA due to its delayed onset and
prolonged action. NPH insulin is generally started when patients recover from DKA and begin to
tolerate oral feeding
Bicarbonate administration is reserved for DKA patients with severe acidosis (pH less than or
equal to 7.1), plasma bicarbonate < 5 mEq/L, or severe hyperkalemia
Struma ovarii is a very rare cause of thyrotoxicosis and is due to production of thyroid hormone
by an ovarian teratoma. It typically presents in women over age 40 with a pelvic mass, ascites, or
abdominal pain. The thyroid gland is not enlarged with this condition.
Causes of hypercalcemia can be categorized on the basis of PTH levels as PTH-dependent or
PTH-independent
• PTH-independent hypercalcemia (suppressed PTH) is usually due to malignancy, vitamin D
toxicity, or extrarenal conversion of 25-hydroxyvitamin D to 1 ,25 dihydroxyvitamin D in
granulomatous diseases (eg, sarcoidosis).
• PTH-dependent hypercalcemia (elevated or inappropriately normal PTH) is usually due to
primary hyperparathyroidism (PHPT)
This patient has hypercalcemia with an elevated PTH level, suggesting PHPT. Normally,
elevated plasma calcium suppresses PTH secretion
Milk-alkali syndrome is caused by excessive ingestion of calcium and absorbable alkali (eg,
calcium-containing antacids). Calcium would be elevated, but PTH would be suppressed
Suspect malignant otitis externa in any diabetic patient with severe ear pain, otorrhea, and
evidence of granulation tissue in the ear canal. Pseudomonas aeruginosa is the most frequent
cause of malignant otitis externa in diabetics
Humoral hypercalcemia of malignancy (HHM) is the most common cause of PTH independent
hypercalcemia and frequently presents with very high calcium levels (eg, >14 mg/dL),
symptomatic (eg, polyuria, constipation, nausea). PTHrP causes increased bone resorption and
increased reabsorption of calcium in the distal renal tubule. However, PTHrP does not induce the
conversion of 25-hydroxyvitamin D to 1 ,25-dihydroxyvitamin D to the same extent as PTH
does, and so 1 ,25-dihydroxyvitamin D levels will be low or low-normal
Sarcoidosis and certain hematologic malignancies (eg, lymphoma) can cause hypercalcemia due
to increased conversion of 25-hydroxyvitamin D to 1 ,25 dihydroxyvitamin D. However, this
patient's low-normal 1,25-dihydroxyvitamin D level makes this less likely
The decreased ADH action in both types of DI leads to decreased renal water reabsorption and
water loss with polyuria. This results in dilute urine and hypernatremia. Central DI usually has
significant hypernatremia (>150 mEq/L) due to an impaired thirst mechanism. Patients with
nephrogenic DI usually have an intact thirst mechanism and adequate water intake; they usually
compensate for renal water loss and may have a normal sodium level.
Polyuria in nonhospitalized patients can be due to primary polydipsia or DI. Primary polydipsia
is due to increased water intake that leads to a dilute urine (urine osmolality < 1/2 plasma
osmolality) and hyponatremia (sodium <137 mEq/L). Serum Na >145 mEq/L with dilute urine
excludes primary polydipsia and favors DI. DI can be central (decreased ADH release from
pituitary) or nephrogenic (normal ADH level but with renal ADH resistance)
Tight blood glucose control in patients with diabetes decreases the risk of microvascular
complications (eg, retinopathy, nephropathy), increases the risk of hypoglycemia, and has an
uncertain effect on macrovascular complications (eg, MI, stroke) and all-cause mortality
Most patients with HHS or DKA have normal or elevated serum potassium levels at initial
evaluation. This is due to the combined effects of insulin deficiency and hyperosmolality, which
promote the movement of potassium out of cells into the extracellular space. Despite normal
serum levels, patients with HHS or DKA have a total body potassium deficit (3- 5 mg/kg) due to
excessive urinary loss caused by osmotic diuresis induced by hyperglycemia. The clinical
implication of potassium depletion is that insulin therapy for HHS can abruptly lower serum
potassium levels and cause severe hypokalemia
Milk-alkali syndrome is caused by excessive intake of calcium and absorbable alkali. It can be
seen in patients taking calcium bicarbonate for osteoporosis. The resulting hypercalcemia causes
renal vasoconstriction and decreased glomerular blood flow. In addition, inhibition of the Na-
K-2CI cotransporter (due to activation of calcium-sensing receptors in the thick ascending loop)
and impaired ADH activity lead to loss of sodium and free water. This results in hypovolemia
and increased reabsorption of bicarbonate (augmented by the increased intake of alkali). In
addition to hypercalcemia, metabolic findings in MAS include hypophosphatemia,
hypomagnesemia, metabolic alkalosis, and acute kidney injury. PTH levels are suppressed
Diagnosis of follicular thyroid cancer (FTC) based on a limited tissue sample is not possible as
the cytologic findings are similar to benign follicular adenomas. However, examination of a
surgically excised nodule will show invasion of the tumor capsule and or blood vessels. FTC can
metastasize via hematogenous spread to distant tissues.
Papillary thyroid cancer is characterized by slow spread into local tissues and regional lymph
nodes. Pathologic findings include large cells with ground glass cytoplasm; pale nuclei with
inclusion bodies and central grooving; and grainy, lamellated calcifications known as psammoma
bodies.
Patients with hypoalbuminemia can have decreased total serum calcium. However, ionized
calcium (physiologically active form) is hormonally regulated and remains stable.
Diabetic patients age 40-75 should receive statin therapy regardless of baseline lipid levels.
Current guidelines recommend blood pressure <140/90 mm Hg in patients with diabetes. Patients
with diabetic nephropathy may benefit from lower blood pressures (<130/80 mm Hg), but more
aggressive blood pressure control is not necessary in otherwise healthy diabetics.
MEN 2A and 2B (usually due to RET proto-oncogene mutations) are associated with pheo which
can be asymptomatic at the time of diagnosis but cause life-threatening hypertensive crisis
during surgical procedures (eg, thyroidectomy). In light of this risk, most patients with MTC
should undergo RET mutation testing and screening for pheochromocytoma with a plasma
fractionated metanephrine assay. If found, pheochromocytoma should be resected prior to
thyroidectomy
Toxic adenoma presents as symptoms suggestive of thyroid toxicosis. There is radioactive iodine
uptake in the nodule, and suppression of uptake in the rest of the thyroid gland. Patients with
toxic nodule do not have infiltrative ophthalmopathy
The radioactive iodine uptake in patients with painless thyroiditis is markedly reduced
Glucagonoma often presents with nonspecific symptoms and requires a high index of suspicion
to make the diagnosis. Glucagonoma should be suspected in patients presenting with mild
diabetes mellitus or hyperglycemia with necrotic migratory erythema, diarrhea, anemia, and
weight loss. Glucagon >500 pg/ml usually confirms the diagnosis
Glucagon balances the effects of insulin by regulating the amount of sugar in your blood. If you
have too much glucagon, your cells don’t store sugar and instead sugar stays in your blood.
Glucagonoma tumor cells produce large amounts of glucagon, and these high levels create
severe, painful, and life-threatening symptoms
The whole thyroid gland is hyperfunctional in Graves' disease, radioiodine is taken up by the
entire thyroid gland, thereby resulting in complete thyroid ablation.
Glucocorticoids suppress the secretion of CRH from the hypothalamus and ACTH secretion from
the pituitary. The diagnosis of central AI begins with measurement of morning cortisol (low) and
ACTH (low in central AI; high in primary AI). Aldosterone levels will be low in primary AI but
normal in central AI (aldosterone is primarily regulated by the RAAS)
Increased ACTH and low cortisol are seen in primary AI. These patients have low levels of all
adrenal cortical hormones, including cortisol, adrenal sex steroids, and aldosterone. These
patients are typically hyperpigmented, hyponatremic, and hyperkalemic
Chronic adrenal insufficiency presents with weight loss, fatigue, and gastrointestinal symptoms.
Volume depletion can cause hypotension and syncope. Most patients have hyponatremia due to
renal sodium loss and increased release of ADH. Hyperkalemia is also common due to
mineralocorticoid deficiency (aldosterone)
This patient has paresthesias, carpal spasm (Trousseau sign), and hyperreflexia. In the setting
of high-volume transfusion, these neurologic features are most likely due to hypocalcemia.
Other potential features of severe hypocalcemia include tetany, laryngospasm, seizures,
encephalopathy, and heart failure. Patients with chronic hypocalcemia may have only mild
symptoms, but patients with acute hypocalcemia will have more severe manifestations at the
same calcium concentration. High-volume blood transfusion can cause symptomatic
hypocalcemia due to chelation of ionized calcium by citrate in transfused blood. Patients with
impaired hepatic function are at increased risk due to decreased clearance of citrate by the liver.
Other infused substances that can chelate calcium in the blood include lactate, foscarnet, and
sodium ethylenediaminetetraacetic acid (EDTA)
Hyperventilation due to anxiety can cause acute respiratory alkalosis, increasing the binding of
calcium to albumin and reducing ionized calcium concentrations
Most patients with hypothyroidism have an increased requirement for levothyroxine after
starting oral estrogen (estrogen replacement therapy or oral contraceptives). Oral estrogen
formulations decrease clearance of thyroxine-binding globulin (TBG), leading to elevated TBG
levels. TBG is synthesized and sialylated in the liver. Transdermal estrogen bypasses the liver
and does not affect TBG levels. Patients with normal thyroid function can readily increase
thyroxine production to saturate the increased number of TBG binding sites, but hypothyroid
patients are dependent on exogenous thyroid replacement and cannot compensate. This results in
decreased free thyroxine and increased TSH. As a result, higher dosing of levothyroxine may be
required. A rise in estrogen levels is also one of the main reasons for higher levothyroxine
requirements during pregnancy
Due to increased TBG, the level of total thyroid hormones may increase slightly or remain the
same in hypothyroid patients started on oral estrogen. However, the levels of free T4 decreases.
Drugs that inhibit conversion of T4 to T3 include glucocorticoids, beta blockers, iopanoic acid
(oral cholecystography contrast agent), and propylthiouracil
T4 and T3 are metabolized by deiodination (major method), glucuronidation, and sulfation in the
liver. Phenobarbital, rifampicin, phenytoin, and carbamazepine increase hepatic clearance of
thyroid hormone by increasing the activity of these enzymes (mainly deiodination)
A 34-year-old woman complains of fatigue for the last several months. She becomes tired easily
after walking short distances. She also has difficulty combing her hair due to problems holding
her hands over her head for long periods. The patient reports anxiety, irritability, and an
unintentional weight loss of 3.6 kg (8lb) over the last 2 months. She has no fever or loss of
appetite. She does not smoke or consume alcohol. Her father died of a stroke at age 54 and her
mother has diabetes mellitus. On examination, the patient is afebrile with a pulse of 115/min.
Cardiac examination shows regular rhythm with no murmur. Her lungs are clear to auscultation.
Her gait is normal, but she drops into the chair when asked to sit down slowly. A fine finger
tremor is evident when the patient extends her arms. Her muscles are nontender to palpation. She
appears to have decreased muscle mass in her shoulders. Deep-tendon reflexes are normal.
Which of the following is the most likely cause of this patient’s symptoms
This patient's presentation (inability to comb hair with difficulty holding arms up) indicates a
proximal muscle weakness and likely myopathy. This type of myopathy can be due to conditions
such as connective tissue (eg, polymyositis/dermatomyositis), endocrine (eg, hypo- or
hyperthyroidism, Cushing's disease), and neuromuscular (eg, Lambert-Eaton syndrome,
myasthenia gravis) diseases, and drugs (eg, steroids). However, this patient's other findings
(fatigue, anxiety, tremor, weight loss, and tachycardia) favor hyperthyroidism over the other
causes. Up to 60%-80% of untreated hyperthyroid patients can develop acute or chronic
myopathy. Proximal muscle weakness with or without muscle atrophy can occur in 60%-80% of
patients with untreated hyperthyroidism and correlates to the duration of the hyperthyroid state.
Hip flexors and quadriceps are predominantly affected, and the weakness can gradually progress
to involve the proximal muscles of the upper extremities.
Dermatomyositis and polymyositis are examples of idiopathic inflammatory myopathies that
present with symmetric proximal muscle (deltoids and hip flexors) weakness and tenderness.
However, this patient's other findings (tremor and tachycardia) make thyroid disease more likely.
Patients with PCOS are at increased risk of developing insulin resistance and type 2 diabetes. A
75g 2-hour oral glucose tolerance test can identify patients with impaired glucose tolerance
better than a fasting glucose level. Menstrual irregularities and male-pattern baldness is typical
BRCA testing should be offered to patients with a history of ovarian cancer in first or second-
degree relatives at age <50 or in multiple generations. Patients with PCOS are not at increased
risk of ovarian cancer. However, they are at increased risk for endometrial cancer due to chronic
anovulation. This patient had 1 relative age >50 with ovarian cancer and does not require BRCA
testing
PTU inhibits thyroid hormone synthesis and also inhibits the extrathyroid conversion of T4 to
T3; however, in more than two-thirds of patients, hyperthyroidism recurs within 6 months after
stopping therapy. Therefore, PTU is generally only used when radioactive iodine is
contraindicated, such as in pregnancy
A 65-year-old man comes to the physician complaining of decreased appetite, nausea, abdominal
bloating, and early satiety for the past several months. He has no heartburn or epigastric pain but
does have occasional vomiting The patient has longstanding type 2 diabetes mellitus complicated
by nonproliferative retinopathy and has taken insulin for the last 15 years. His blood glucose
readings using the home monitor are 40-400 mg/dL. Most of the low blood glucose readings
occur after meals. Which of the following would be most helpful in treating this patient's
condition?
Diabetic gastroparesis (delayed gastric emptying) presents with symptoms of anorexia, nausea,
vomiting, early satiety, postprandial fullness, and impaired glycemic control. Hypoglycemic
episodes can occur with insulin administration prior to meals in patients with impaired gastric
emptying or delayed absorption. Metoclopramide has both prokinetic and antiemetic properties
and is useful for symptomatic relief of nausea, bloating, and postprandial fullness in patients with
diabetic gastroparesis. The use of metoclopramide can be associated with a small (<1%) risk of
extrapyramidal side effects (eg, tardive dyskinesia). Other alternate therapeutic options include
erythromycin (more useful when used intravenously for acute exacerbations) and cisapride
(restricted use in the United States due to the risk of cardiac arrhythmias and death)
Progesterone analogs (eg, megestrol acetate, medroxyprogesterone acetate) are useful in the
palliation of anorexia and in improving appetite and weight gain in patients with cancer-related
anorexia/cachexia syndrome
Fever and sore throat in any patient taking antithyroid drugs suggests agranulocytosis.
Antithyroid drugs should be stopped and WBC count checked. Routine WBC count
measurement in patients taking antithyroid drugs is not cost effective. Current recommendations
state that once the patient complains of fever and sore throat, the antithyroid drug should be
discontinued promptly and the WBC count measured. A total WBC count less than 1,000/cubic
mm warrants permanent discontinuation of the drug. If the total WBC count is more than 1,500
per cubic mm, antithyroid drug toxicity is unlikely to be the cause of the sore throat and fever.
Graves ophthalmopathy is due to the effects of activated T cells and thyrotropin receptor
antibodies (TRAB) on TSH receptors on retro-orbital fibroblasts and adipocytes. Titers of TRAB
increase following RAI therapy thus worsening of ophthalmopathy. Administration of
glucocorticoids with RAI prevents complications in patients with mild ophthalmopathy and
thyroidectomy is preferred over RAI for those with moderate or severe ophthalmopathy
Destruction of thyroid follicles by RAI can cause unregulated release of thyroid hormone,
temporarily worsening the hyperthyroid state. Younger patients without cardiovascular disease
usually tolerate RAI without major problems. However, elderly patients and those with
cardiovascular disease may not tolerate the temporary increase in thyroid hormone and often are
given an antithyroid drug to deplete existing thyroid hormone stores prior to RAI.
Painless muscle weakness associated with weight gain, bone loss, hypertension, and hirsutism is
consistent with hypercortisolism (Cushing syndrome). It is due to the direct catabolic effects of
cortisol on skeletal muscle, which leads to muscle atrophy. Glucocorticoid-induced muscle
atrophy is thought to be mediated by the inhibition of Akt-1, an intracellular signaling molecule
with tyrosine kinase activity. Interference of IGF1 signaling may also contribute. Furthermore,
glucocorticoids may decrease muscle cell differentiation and protein synthesis
Patients on Coumadin (warfarin) can have adrenal hemorrhage with acute stress such as sepsis.
This can occur even when the INR is in the therapeutic range. Stress increases the ACTH levels,
which in turn increases the adrenal blood flow, thus predisposing such patients to adrenal
hemorrhage. Hemorrhagic necrosis of the adrenal glands may also be seen in patients with some
systemic infections (e.g , children with meningococcemia or Pseudomonas sepsis)
A 25-year-old woman comes to the physician with a 3-month history of weight loss, irritability,
insomnia, and palpitations. Her past medical history is insignificant .She is not currently taking
any medications. The patient does not use tobacco, alcohol, or illicit drugs. Blood pressure is
155/70 mm Hg and pulse is 110/min. Physical examination shows lid retraction, dry skin, and
fine tremor of the hands. Which of the following is the most likely cause of this patient’s
hypertension? increased myocardial contractility
Systemic hypertension may be seen in both hyper- and hypothyroidism. Hyperthyroidism causes
a decrease in systemic vascular resistance, but blood pressure (primarily systolic) rises due to
positive inotropic and chronotropic effects. In contrast, hypothyroidism causes hypertension due
to an increase in systemic vascular resistance.
Initial testing for suspected hypercortisolism can include a 24-hour assay for urine free cortisol, a
late-night salivary cortisol measurement, or a low-dose dexamethasone suppression test. Once
hypercortisolism is established, an ACTH level can determine whether it is ACTH-dependent
(Cushing disease or ectopic ACTH production) or ACTH-independent (adrenal disease or
exogenous glucocorticoid intake). Patients with an elevated ACTH should have a high-dose
dexamethasone suppression test to determine whether ACTH production is pituitary
(dexamethasone suppresses cortisol production) or ectopic (dexamethasone does not suppress
cortisol)
Hypercalcemia due to malignancy, serum calcium is typically much higher (often >14 mg/dl) and
patients are more likely to be symptomatic (eg, polyuria, dehydration, mental status changes).
Rapidly progressive hirsutism with virilization suggests very high androgen levels due to
androgen-producing neoplasm. Elevated DHEAS levels are seen in androgen-producing adrenal
tumors. Women normally produce a number of androgens, including testosterone (T),
androstenedione (AS), dehydroepiandrosterone (DHEA), and dehydroepiandrosterone sulfate
(DHEAS). AS, DHEA, and T are produced by both the ovaries and the adrenals. In contrast,
DHEAS is produced predominantly in the adrenal glands. Although DHEA and DHEAS are used
as diagnostic markers, they have negligible androgenic activity, and the clinical features are due
to the conversion of DHEA and DHEAS to more potent androgens (ie, AS and T). Therefore, this
patient would most likely have elevated DHEAS and T levels. LH would be low due to negative
feedback by T
The primary ovarian androgens include testosterone, androstenedione, and DHEA. The adrenals
produce these as well as dehydroepiandrosterone-sulfate (DHEAS). Therefore. women with a
suspected androgen-producing tumor should be evaluated with serum testosterone and DHEAS
levels
• Elevated testosterone levels with normal DHEAS levels suggest an ovarian source (more
common)
• Elevated DHEAS levels suggest an adrenal tumor (far less common)
Features of diabetic neuropathy depend on the type of nerve fibers involved; small fiber
neuropathy causes pain, allodynia, and paresthesias, whereas large fiber involvement causes
numbness, loss of proprioception, diminished vibration sense, and loss of ankle reflexes. Both
types are seen together. As diabetic neuropathy progresses, the pain may diminish but the sensory
deficits persist. Initial treatment options for painful diabetic neuropathy include TCAs (eg,
amitriptyline), dual SNRI (eg, duloxetine), and anticonvulsants (eg, pregabalin, gabapentin).
Although amitriptyline is effective for reducing neuropathic pain, it should be used with caution
in patients age >65 due to its anticholinergic effects, and it is not recommended in patients with
pre-existing cardiac disease due to the risk of precipitating conduction abnormalities
Risk factors for diabetic foot ulcers include diabetic neuropathy, previous foot ulceration,
vascular disease, and foot deformity. Diabetic neuropathy is the most important factor and is
found in >80% of patients with ulcers. Monofilament testing predicts the risk of future ulcers
Screening for diabetes is recommended in patients with sustained blood pressure >135/80 mmHg
and may be considered in all patients over age 45, as well as those with additional risk factors for
diabetes. Screening options include fasting plasma glucose, 2-hour OGTT and HBA1c
The HPV vaccine is approved for use in male and female patients age 9-26
For women with average risk for cervical cancer, a Pap test is recommended every 3 years from
age 21-29. Between ages 30-65, patients with initial negative testing may have either a Pap test
alone every 3 years or a combination of Pap test and HPV testing every 5 years (preferred)
The 13-valent pneumococcal conjugate vaccine (PCV13) is recommended for all adults age >65,
followed by the 23-valent pneumococcal polysaccharide vaccine (PPSV23) 6-12 months later.
Sequential PCV13 and PPSV23 are also recommended for adults age <65 with certain very high-
risk underlying conditions (eg, CSF leaks, sickle cell disease, cochlear implants, congenital or
acquired asplenia, immunocompromised status, chronic renal failure). PPSV23 alone is
recommended for adults age <65 who are current smokers or have certain chronic medical
conditions, including heart or lung disease, diabetes, and chronic liver disease.
Hypocalcemia: In asymptomatic patients, a single low serum calcium level must be confirmed by
a second serum calcium value. Hypoalbuminemia, hypomagnesemia, and certain drugs (eg,
calcium chelators, bisphosphonates, phenytoin) can also cause hypocalcemia and should be
considered. After hypocalcemia is confirmed, the next step is to check PTH level. The MCC of
hypoparathyroidism seen in hypocalcemia is due to a recent parathyroid surgery or surgeries
involving the surrounding area (eg, thyroidectomy). An elevated PTH level is commonly seen in
vitamin D deficiency and chronic kidney disease. This patient has normal serum creatinine, and
the next step is to check the 25-hydroxy vitamin D level to diagnose vitamin D deficiency
Hot nodules are almost always benign and can be treated for hyperthyroidism. Fine-needle
aspiration is indicated for patients with normal or high TSH, cold nodules, thyroid cancer family
history, or suspicious thyroid ultrasound findings.
Postpartum AI may be due to adrenal (primary AI) or pituitary (secondary AI) disease.
Hyperpigmentation and signs of mineralocorticoid deficiency suggest primary rather than
secondary AI.
Cardiology
Septic shock is a form of distributive shock due to an underlying systemic infection involving the
circulatory system. The systemic inflammatory response causes peripheral vasodilatation and
decreased systemic vascular resistance (SVR). The decreased blood flow returning to the heart
also lowers PCWP. Cardiac output is often increased to compensate and maintain adequate tissue
perfusion. MvO2 is high due to hyperdynamic circulation, improper distribution of cardiac
output, and inability of the tissues to adequately extract oxygen. Hypotension, warm (early)/cool
(late) extremities, and elevated lactate levels are common clinical findings.
Cardiogenic shock is usually due to significant LV dysfunction and reduced pump function.
There is often low CO, elevated PCWP, and increased SVR. The decreased CO decreases tissue
perfusion, which signals tissues to extract more oxygen from the blood and decrease MvO2
Hypovolemic shock has reduced preload (PCWP) and reduced CO. SVR increases in an attempt
to maintain adequate perfusion to the vital organs. MvO2 is low due to reduced tissue perfusion
and increased oxygen extraction by hypoperfused tissue.
Patients with right ventricular infarction have reduced blood delivery to the left ventricle, which
in turn decreases cardiac output despite normal left ventricular systolic function. It also leads to
increased right atrial and/or ventricular pressures along with low or normal PCWP. MvO2 would
be expected to be low due to reduced cardiac output and tissue hypoperfusion
Symptomatic sinus bradycardia (HR <60/min with regular rhythm and a constant PR interval).
Pathologic causes include sick sinus syndrome, myocardial ischemia or infarction, obstructive
sleep apnea, hypothyroidism, increased ICP, and medications. The first step in symptomatic
patients is to identify and treat reversible causes, followed by administration of IV atropine for
initial treatment of the bradycardia and hypotension. In patients with an inadequate response,
further treatment options include IV epinephrine or dopamine, or transcutaneous pacing
Adenosine causes a transient block of impulse conduction at the level of the AV node, which is
useful in identifying (and in some cases, terminating) supraventricular tachycardia.
IV glucagon increases the intracellular levels of cAMP thus used in treating BB or CCB toxicity.
Acute MR (or acute AR) leads to excessive diastolic volume overload, which in turn causes
elevated LVEDP (LV filling pressure). This elevated LV filling pressure is reflected back in the
left atrium and pulmonary circulation and is responsible for the signs and symptoms of acute
pulmonary edema and CHF. Unlike chronic MR, acute MR does not cause any significant
change in left atrial or ventricular size and/or compliance
The clinical presentation of progressive dyspnea on exertion, fatigue, and exertional syncope is
suggestive of outflow obstruction. Patients with fixed outflow obstruction (eg, severe AS) cannot
increase cardiac output in response to exercise-induced vasodilation, leading to hypotension,
transient cerebral hypoperfusion, and decreased exercise tolerance, presyncope, or syncope.
Physical examination typically reveals a delayed and diminished carotid pulse, soft second heart
sound, and mid- to late-peaking systolic murmur with maximal intensity at the second right
intercostal space and radiation to the carotids.
An opening snap and a late diastolic murmur best heard at the apex are characteristic of mitral
stenosis, which can present with decreased exercise tolerance, exertional dyspnea, and, less
commonly, chest pain, hemoptysis, or lower extremity edema. Exertional syncope is not seen.
Prominent capillary pulsations in the fingertips or nail beds can be seen with AR as a result of
widened pulse pressure. AR can present with progressive dyspnea and fatigue but is unlikely to
cause exertional syncope
Pulsus paradoxus refers to an exaggerated decrease (>10 mm Hg) in systolic blood pressure
(SBP) with inspiration. It is commonly seen with pericardial diseases (eg, cardiac tamponade)
and can occur with severe asthma and chronic obstructive pulmonary disease
Orthostatic (postural) hypotension refers to a drop in SBP >20 mm Hg or diastolic blood pressure
>10 mm Hg within 2-5 minutes of standing from a supine position. It does not cause exertional
syncope
Survivors of Hodgkin lymphoma are at increased risk for cardiac disease, which can present as
much as 10-20 years or more after mediastinal irradiation and/or anthracycline therapy. Cardiac
complications of Hodgkin lymphoma include acute or delayed pericardial disease, myocardial
ischemia/infarction, restrictive cardiomyopathy, CHF, valvular abnormalities, and conduction
defects. It should be suspected in patients with progressive peripheral edema, elevated JVP,
hepatomegaly, and ascites. Other findings include the presence of hepatojugular reflux,
Kussmaul's sign (increase in JVP on inspiration), and a pericardial knock (mid-diastolic sound)
Isolated systolic hypertension (ISH), defined as a SBP >140 with a diastolic BP <90. The
pathophysiologic mechanism leading to ISH is believed to be increased stiffness or decreased
elasticity of the aortic and arterial walls in elderly patients. This explains the increases in pulse
pressure (ie, difference between systolic and diastolic BP). Similar to the way that primary HTN
is managed, treatment for ISH should include lifestyle modifications and pharmacologic therapy
ECG findings of irregularly irregular rhythm with narrow-complex tachycardia and no organized
P waves are consistent with AFRVR. AF is a SVT with unorganized atrial activity. AF with RVR
can present with ventricular rates as high as 150/min, and the symptoms are usually due to the
fast ventricular rate rather than the arrhythmia itself. Hemodynamically unstable patients require
emergency cardioversion. Stable patients can receive medical therapy (eg, BB, diltiazem,
digoxin) to control the ventricular rate. Rhythm control for maintaining sinus rhythm should be
considered in patients unable to achieve adequate HR control or in those with recurrent
symptomatic episodes (eg, palpitations, dyspnea, lightheadedness, angina) or heart failure
symptoms in the setting of underlying LV systolic dysfunction
Excessive alcohol intake (>2 drinks a day) or binge drinking (>5 drinks in a row) is associated
with increased incidence of hypertension compared to nondrinkers.
Cigarette smoking causes a transient rise in BP. However, some observational studies have
shown that chronic light to moderate smokers have lower BP than nonsmokers.
Epigastric burning provoked by exertion (heavy lifting) and relieved over several minutes by rest
is concerning for atypical angina (lack of typical chest discomfort). SLE is a known risk factor
for accelerated atherosclerosis and premature coronary heart disease. Exercise ECG is
recommended as an initial stress test for diagnosis and risk stratification of most patients with
suspected stable ischemic heart disease
Uremic pericarditis (UP) occurs in 6%-10% of renal failure patients, typically those with BUN
levels >60 mg/dl. Most UP patients do not present with the classic ECG changes of pericarditis
(eg, diffuse ST-segment elevations). Hemodialysis leads to rapid resolution of chest pain and
reduces the size of any associated pericardial effusion. Systemic anticoagulation (eg, heparin)
can cause hemorrhage into the pericardial space and should be avoided during hemodialysis.
Although this patient has a history of myocardial infarction, post-myocardial pericarditis
(Dressler's syndrome) usually occurs within 1-6 weeks after myocardial infarction.
Glucocorticoids (eg, prednisone) can be used in patients with inadequate response to initial
dialysis. They have a low success rate and have been associated with a higher risk of recurrence.
This patient's clinical presentation (dyspnea, orthopnea, paroxysmal nocturnal dyspnea, bibasilar
crackles, hypoxemia) is consistent with acute pulmonary edema, most likely due to acute
decompensated heart failure (ADHF). ADHF is most commonly due to LV systolic or diastolic
dysfunction. However, pulmonary edema can also occur in the setting of normal LV function in
conditions such as severe hypertension, renal artery stenosis, or severe renal disease with fluid
overload. Acute management of ADHF includes supplemental oxygen and intravenous loop
diuretics (eg, furosemide). Intravenous nitroglycerin is a possible adjunctive therapy in patients
without hypotension, especially those with mitral regurgitation or symptomatic myocardial
ischemia. Intravenous nitroglycerin rapidly decreases preload to relieve dyspnea and tachycardia
associated with pulmonary edema.
Usually, ST-segment elevations resolve within a few weeks of an MI. Ventricular aneurysms
(VA) present with ECG findings of persistent ST-segment elevation after a recent MI and deep Q
waves in the same leads. Large VAs can lead to progressive LV enlargement, causing heart
failure, refractory angina, ventricular arrhythmias, mural thrombus with systemic arterial
embolization, or mitral annular dilation with MR (explaining this patient’s murmur). Diagnosis is
confirmed by echocardiography (thinned, dyskinetic LV portion in the area of prior MI)
BNP is a natriuretic hormone released from ventricular myocytes in response to high ventricular
filling pressures and wall stress in patients with CHF. Elevated levels of circulating BNP
correlate with the severity of LV systolic dysfunction. The third heart sound (S3) is a low-
frequency diastolic sound produced by the passive ventricular filling during early diastole; it is
best heard over the cardiac apex in the left lateral decubitus position. An abnormal S3 (louder
and higher pitch, S3 gallop) is commonly heard in patients with CHF due to left ventricular
systolic dysfunction (up to 99% specificity), and it correlates with elevated left atrial and/or
ventricular filling pressures and serum BNP levels.
GERD and esophageal motility disorders are common causes of non-cardiac chest pain. Features
suggestive of an esophageal origin of chest pain include prolonged episodes lasting more than an
hour, postprandial symptoms, heartburn or dysphagia, and relief of pain by antireflux therapy
Pulmonary HTN occurs in patients with long-standing primary pulmonary or cardiac disease.
Patients have associated symptoms of dyspnea on exertion, fatigue, or lower-extremity edema
The patient's clinical presentation - acute pain, paresthesia, pallor, and pulselessness - is
consistent with acute arterial occlusion of the lower extremity, likely due to thromboembolism in
the setting of AF (patient's irregular pulse) and should be managed with anticoagulation, which
reduces the long-term risk of systemic embolization and could have prevented this episode of
acute arterial occlusion. Apixaban is a non-vitamin-K antagonist oral anticoagulant (along with
dabigatran, rivaroxaban, and edoxaban) that has been shown to significantly reduce the risk of
systemic embolization in patients at moderate to high risk of thromboembolic events.
Amiodarone is a class III antiarrhythmic agent frequently used for conversion and maintenance
of sinus rhythm in patients with AF. Despite its efficacy in maintaining sinus rhythm, it does not
prevent thrombus formation in the left atrium or left atrial appendage, and patients continue to
remain at risk of systemic embolization.
Cilostazol is a PDEi used for symptomatic management of patients with intermittent claudication
The USPSTF recommends screening male active or former smokers aged 65-75 years with a
one-time abdominal ultrasound to evaluate for an abdominal aortic aneurysm
This patient's clinical presentation is consistent with systemic atheroembolism from disruption of
atherosclerotic aortic plaques (cholesterol crystal embolism) and can lead to systemic emboli
(cerebral, visceral, or lower extremity). Embolism can happen spontaneously, but occurs more
commonly during vascular procedures such as peripheral angiography or interventions,
guidewire or catheter manipulations during cardiac catheterization, and intraaortic balloon pump
insertion. The diagnosis should be suspected in patients with a constellation of acute/subacute
renal failure, typical skin manifestations, and/or GI symptoms (eg, nausea, abdominal pain) after
a recent vascular procedure. Livedo reticularis is a reticular, erythematous or purple discoloration
of the skin that blanches when pressure is applied. Laboratory studies may show eosinophilia,
eosinophiluria, and hypocomplementemia in cases of recent atheroembolism. Urinalysis is
typically benign, with few cells or casts. When the diagnosis remains unclear, skin or renal
biopsy may be performed for confirmation
Patients undergoing coronary angiography are at risk for contrast-induced nephropathy (CIN).
Urinalysis in CIN typically shows muddy-brown granular and epithelial cell casts. CIN usually
begins to resolve within 3-5 days; renal atheroembolism often shows delayed-onset kidney
injury. Moreover, CIN is not associated with multisystem involvement, which favors the
diagnosis of cholesterol embolization.
Situational syncope is a form of reflex or neurally mediated syncope associated with specific
triggers (eg, micturition, defecation, cough). These triggers cause an alteration in the autonomic
response and can precipitate a predominant cardioinhibitory, vasodepressor, or mixed response
IV adenosine is useful in the initial diagnosis and management of patients with narrow QRS
complex tachycardia. It slows the sinus rate, increases AV nodal conduction delay, or can cause a
transient block in AV node conduction. IV adenosine can be useful in identifying P waves to
clarify diagnosis of atrial flutter or atrial tachycardia. It can also terminate PSVT by interrupting
the AV nodal reentry circuit.
Panic attacks typically cause sinus tachycardia, with normal P wave morphology and relationship
with the QRS complex. In PSVT, P waves are usually "buried" within or are seen just after the
QRS complex. Alprazolam is useful for patients with panic attacks or for reducing the anxiety
associated with the episode of SVT. It is not useful for diagnosis or treatment of SVT.
Amiodarone or lidocaine is often used in the treatment of hemodynamically stable patients with
wide-QRS-complex tachycardia (ventricular tachycardia). Amiodarone is also used to treat AF.
Stable patients with wide-complex tachycardia can be initially managed with antiarrhythmic
drugs (eg, amiodarone, procainamide, sotalol, lidocaine). Synchronized electrical cardioversion
is indicated for patients with persistent tachyarrhythmia who are severely symptomatic (altered
mental status, acute heart failure or pulmonary edema, ischemic chest pain) or hemodynamically
unstable (hypotension, signs of shock).
Diastolic and continuous murmurs are usually due to an underlying pathologic cause, and their
presence should prompt further evaluation with a TTE. Midsystolic murmurs in otherwise young,
asymptomatic adults are usually benign and do not require further evaluation.
A 63-year-old white woman comes to the emergency department with the recent onset of left-
sided weakness. She reports increased fatigue, low-grade fevers, and occasional palpitations over
the last 3 months. She has lost 6 kg (132 lb) during this period. Her temperature is 37C (98.6 F),
blood pressure is 120/76 mm Hg, pulse is 80/min and regular, and respirations are 14/min.
Cardiac examination shows normal 1st and 2nd heart sounds and a mid-diastolic rumble at the
apex. Lungs are clear on auscultation. Transthoracic echocardiography shows a mass in the left
atrium. Which of the following is the most likely diagnosis?
This patient's presentation of chronic systemic symptoms (eg, fever, weight loss, fatigue),
embolic phenomenon (left-sided weakness), and echocardiogram findings is suggestive of
cardiac myxoma. Myxomas are the most common benign primary cardiac tumors with
approximately 80% located in the left atrium. Left atrial myxomas can cause obstruction of
blood flow across the mitral valve, mimicking mitral valve disease and producing early diastolic
sound ("tumor plop”). Patients often present with fatigue, cough, dyspnea, orthopnea, pulmonary
edema, or hemoptysis. Left atrial tumors can also cause systemic embolization (eg, transient
ischemic attack, stroke, splenic infarcts). About 50% of patients report constitutional symptoms
(due to overproduction of interleukin-6) such as fever, weight loss, or Raynaud phenomenon.
Although transesophageal echocardiography is the most sensitive test for diagnosis, transthoracic
echocardiography is usually adequate. Once diagnosis is established, prompt surgical resection is
recommended to avoid complications of embolization and risk of sudden death.
Myxomatous valve degeneration is the pathologic cause of MVP, causing a midsystolic click
Mitral stenosis (MS) also produces a mid-diastolic rumble. MS can cause AF and left atrial
thrombus. The MCC of MS is rheumatic heart disease, which typically presents at age 30-50
with symptomatic MS and is common in developing countries. In developed countries,
congenital MS is more common and presents with symptoms at a much earlier age
Patient's clinical presentation is consistent with tricuspid valve endocarditis with septic emboli to
the lungs. Septic pulmonary emboli typically cause cough, chest pain, and hemoptysis and can
produce numerous round alveolar infiltrates on chest imaging. IV drug users are at increased risk
for bacterial endocarditis involving the right-sided heart valves. TR is a common complication
and typically causes a holosystolic murmur that increases in intensity with inspiration.
AR causes an early decrescendo diastolic murmur that begins immediately after A2 (aortic
component of second heart sound). The murmur is high pitched and has a blowing quality that is
best heard along the left sternal border at the third and fourth intercostal space when the patient
is sitting up and leaning forward while holding the breath in full expiration. Aortic valve
endocarditis can cause septic emboli to systemic circulation, but it would not explain the septic
pulmonary emboli seen with trisucpid valve endocarditis
Upright tilt table testing is used to establish the diagnosis of vasovagal syncope in patients with
recurrent unexplained syncope in the absence of organic heart disease.
WPW has characteristic ECG findings of a short PR interval (<120 msec), slurred initial
upstroke of the QRS complex (delta wave), and widening of the QRS complex with ST/T wave
changes. WPW is due to an accessory pathway that bypasses the AV node and directly connects
the atria to the ventricles. Most patients are asymptomatic; however, WPW can be a/w cardiac
arrhythmias. If there is retrograde conduction from the ventricles to the atria, a re-entrant SVT
may occur. ECG features of WPW will not be apparent during episodes of SVT, but the delta
wave should be visible once the rhythm is slowed. Sudden death can occur if WPW is untreated
Hypokalemia can lead to nonspecific ST segment changes (eg, ST segment depression, low T
wave amplitude) and a prominent U wave on ECG.
Long QT syndrome (LQTS) is characterized by prolonged QT interval on ECG (QTc >450 msec
in males or >470 msec in females). It can be congenital or acquired and is a marker for increased
risk of sudden cardiac death due to torsades de pointes or polymorphic VT
Acute pericarditis typically occurs in the first several days after MI. It is characterized by sharp,
pleuritic pain that is worse in the supine position and improved by sitting up and leaning
forward. Diffuse ST elevations, especially with PR depressions, are typical ECG findings
Interventricular free wall rupture and papillary muscle rupture typically occur 3-7 days after
myocardial infarction and present with new onset systolic murmur.
Ventricular free wall rupture typically occurs 3-7 days after anterior wall MI. It presents with
rapid deterioration secondary to pericardial tamponade. Pulseless electrical activity is common.
Pulmonary infarction secondary to PE can cause chest pain, but not diffuse ST elevations. In
addition, while the pain of PE is pleuritic, it should not improve with leaning forward.
Ventricular aneurysm is a complication of anterior wall MI that can occur days to months after
the initial infarction. It presents with akinesis of the involved portion of the left ventricular wall,
ventricular arrhythmias, and systemic embolization
Amiodarone is an antiarrhythmic agent indicated for ventricular arrhythmias. It is also often used
for rhythm control in AF and underlying LV systolic dysfunction. However, it is a/w a broad
range of adverse effects, including hypo- and hyperthyroidism, hepatotoxicity, bradycardia and
heart block, pneumonitis, neurologic symptoms, and visual disturbances.
AAA rupture should be suspected in a patient with a pulsatile abdominal mass, hypotension, and
abdominal pain. In this scenario it is appropriate to forego imaging and emergently send the
patient to the OR
This patient's cool, dusky fingertips are the result of norepinephrine-induced vasospasm. NE has
alpha-1 agonist properties which cause vasoconstriction; this property is useful when trying to
increase the BP of hypotensive patients. However, in some patients with decreased blood flow,
vasoconstriction can result in ischemia and necrosis of the distal fingers and toes. A similar
phenomenon can occur in the intestines (resulting in mesenteric ischemia) or kidney (causing
renal failure). Pressors such as NE can cause ischemia of the distal fingers and toes secondary to
vasospasm. The diagnosis is suggested by symmetric duskiness and coolness of all fingertips
Cholesterol emboli can occur in patients with atherosclerosis and can affect the distal
portions of the digits, thus its occasional name "blue toe syndrome." However, symmetrical
involvement of all digits would be unusual
Digitalis toxicity causes increased ectopy and increased vagal tone. Atrial tachycardia with AV
block occurs from the combination of these two digitalis effects, and is relatively specific for
digitalis toxicity
Diabetic patients age 40-75 should be treated with statin therapy in addition to lifestyle
modification and glucose control. Those with a 10-year risk of atherosclerotic cardiovascular
disease <7.5% should receive moderate-intensity statin therapy, and those with a risk >7.5%
should receive high-intensity statin therapy
Beta blockers are recommended as first-line therapy for controlling anginal symptoms and
improving exercise tolerance in stable angina pectoris. They reduce myocardial oxygen demand
by decreasing heart rate and myocardial contractility
Short-acting nitrates are indicated for acute angina. Long-acting nitrates (eg, isosorbide dinitrate,
isosorbide mononitrate) or CCB (eg, diltiazem, felodipine) are used for chronic stable angina
patients with contraindications to beta blockers or inability to tolerate their side effects. They can
also be used in combination with BB if initial therapy with BB alone is not effective.
Post-MI acute pericarditis typically occurs in the first several days after the infarction. This
patient most likely has Dressler's syndrome, a pericarditis that can occur weeks after an MI.
Dressler's syndrome is believed to be due to immunologic phenomena. Malaise and sometimes
fever are also characteristic. The ESR is typically elevated. NSAIDs are the mainstay of therapy.
Corticosteroids can be used in refractory cases or when NSAIDs are contraindicated.
Anticoagulation should be avoided to prevent development of a hemorrhagic pericardial effusion
Statins inhibit intracellular HMG-CoA reductase, prevent conversion of HMG CoA to mevalonic
acid, and increase the number of cell membrane LDL receptors. Statins also decrease coenzyme
Q10 synthesis, which is involved in muscle cell energy production and possibly contributes to
statin-induced myopathy.
Patient shows evidence of IV drug use and AR (early diastolic murmur). When AR is due to
valvular disease, the early diastolic murmur is best heard along the left sternal border (3rd and
4th intercostal spaces). However, when the AR murmur is due to aortic root disease, it is best
heard along the right sternal border. In this patient, the ECG shows evidence of conduction
abnormality with 2:1 second-degree AV block. These features are suggestive of aortic valve
endocarditis complicated by a perivalvular abscess and resulting AV conduction block and
syncope. Development of AV block in a patient with infective endocarditis should raise suspicion
for perivalvular abscess extending into the adjacent cardiac conduction tissues. Aortic valve
endocarditis and IV drug abuse are a/w an increased risk of periannular extension of endocarditis
The tricuspid valve is the most frequent site of endocarditis in IV drug users. However, tricuspid
endocarditis usually presents with a systolic murmur (holosystolic murmur of tricuspid
regurgitation that becomes accentuated with inspiration). Cardiac conduction abnormalities are
uncommon in patients with tricuspid valve endocarditis.
Primary PCI is recommended within 90 minutes for acute STEMI. Initial stabilization measures
include oxygen, full-dose aspirin (chewed to enter the bloodstream quickly), platelet P2Y12
receptor blocker (eg, clopidogrel, ticagrelor), nitroglycerin for pain control (with morphine if
nitroglycerin is inadequate), beta blocker, and anticoagulation (ie, heparin). Prompt coronary
reperfusion with PCI or fibrinolytic therapy restores blood flow, limits myocardial damage, and
reduces mortality. Current guidelines recommend primary PCI for patients with acute STEMI as
follows
• Within 12 hours of symptom onset AND
• Within 90 minutes from first medical contact to device time at a PCI-capable facility OR
• Within 120 minutes from first medical contact to device time at a non-PCI-capable facility (to
allow time for transport to a PCI-capable facility)
Hepatojugular reflux is a useful clinical tool that can differentiate between cardiac- and liver
disease-related causes of lower-extremity edema. Patients with peripheral edema due to heart
failure have elevated JVP and positive hepatojugular reflux. Those with peripheral edema from
primary hepatic disease and cirrhosis have reduced or normal JVP and negative hepatojugular
reflux. The hepatojugular reflux is not specific to any particular disorder but rather is a reflection
of a failing RV that cannot accommodate an increase in venous return with abdominal
compression. Constrictive pericarditis, RV infarction, and restrictive cardiomyopathy are the
MCC of a positive hepatojugular reflux. This patient's history of tuberculosis is suggestive of
constrictive pericarditis as the likely cause of her presentation.
In patients presenting with dyspnea, serum BNP levels are obtained to differentiate between
cardiogenic pulmonary edema (elevated BNP levels) and noncardiac causes of dyspnea
TEE has excellent sensitivity and specificity for the diagnosis of aortic dissection and is the
preferred diagnostic study in patients with hemodynamic instability or renal insufficiency. CT
angiography and MR angiography should be avoided in patients with kidney disease
Renovascular hypertension should be suspected in all patients with resistant hypertension and
diffuse atherosclerosis, asymmetric kidney size, recurrent flash pulmonary edema, or elevation in
serum creatinine >30% from baseline after starting an ACEi or ARB. The presence of a
continuous abdominal bruit has a high specificity for the presence of renovascular hypertension
Atrial fibrillation is most commonly caused by ectopic foci within the pulmonary veins. Atrial
flutter commonly involves a reentrant circuit around the tricuspid annulus. Atrioventricular nodal
reentry tachycardia results from a reentrant circuit formed by 2 separate conducting pathways
within the AV node. Atrioventricular reentrant tachycardia is due to a reentrant circuit involving
an accessory atrioventricular bypass tract
Pulsus paradoxus is defined as an exaggerated fall in systemic blood pressure >10 mm Hg during
inspiration. It is a frequent finding in cardiac tamponade but can also occur in conditions without
pericardial effusion such as severe asthma or chronic obstructive pulmonary disease.
The clinical manifestations of HCM include exertional dyspnea, chest pain, fatigue, palpitations,
presyncope or syncope. Harsh crescendo-decrescendo systolic murmur with change in intensity
on physiologic maneuvers. Patients with HCM have increased LV wall thickness, most
commonly noted in the basal anterior septum (asymmetric septal hypertrophy). Some patients
have systolic anterior motion of the mitral valve leaflets toward the interventricular septum.
Contact between the mitral valve and the thickened septum during systole leads to LVOT
obstruction and is responsible for the harsh systolic murmur best heard at the apex and left lower
sternal border. Abrupt standing from a sitting position decreases venous return and enhances
LVOT obstruction, resulting in an increase in the intensity of the murmur
This patient has conduction system abnormalities, including prolonged PR interval and
intraventricular conduction delay (prolonged QRS duration), on his initial ECG. This is
suggestive of a bradyarrhythmia or high-grade AV block as the most likely cause of his syncopal
episode. AV block can be quite intermittent and is likely responsible for this patient's prior
episodes of lightheadedness over the past month. His normal heart rate does not exclude
bradyarrhythmia due to the intermittent nature of the block in many patients. He should be
referred for additional electrophysiologic testing and evaluation for a pacemaker implantation
Acute limb ischemia after myocardial infarction suggests possible arterial embolus from LV
thrombus. Management includes immediate anticoagulation, vascular surgery consultation, and
transthoracic echocardiogram to screen for LV thrombus and evaluate LV function.
Cor pulmonale refers to impaired function of the RV caused by pulmonary hypertension that
occurs due to underlying diseases of the lungs (COPD, interstitial lung disease), pulmonary
vasculature (idiopathic pulmonary arterial hypertension), or obstructive sleep apnea. The
diagnosis of cor pulmonale is based primarily on clinical features and echocardiographic findings
(eg, right ventricular hypertrophy, tricuspid regurgitation with right atrial enlargement). If
necessary, definitive diagnosis can be made using right heart catheterization showing elevated
pulmonary artery systolic pressure (>25 mm Hg).
The PCWP is an estimation of LVEDP, and is elevated in patients with LV systolic and/or
diastolic dysfunction. Patients with elevated PCWP show signs of pulmonary edema on lung
auscultation.
Superior vena cava (SVC) syndrome is caused by external or internal compression of the SVC.
This leads to elevated upper body venous pressure, with facial swelling and distended veins in
the neck and/or chest wall. Ascites and lower extremity edema are typically not present
Mitral stenosis results in LA dilation and risk of AFib and cardiac emboli. The pressure is also
transmitted to the pulmonary vasculature, which can result in dyspnea, cough, and hemoptysis
RV MI is due to occlusion of the proximal right coronary artery, and is seen in 30%-50% of
patients with acute inferior wall MI (ST elevation in leads II, III, AVF). Patients typically have
hypotension, JVD, and clear lung fields. Such patients are preload dependent and should be
treated with IV fluids; Avoid preload-reducing medications such as nitrates and diuretics
Arteriovenous fistula causes high-output cardiac failure by shunting the blood from arterial to the
venous side, thereby increasing cardiac preload. The patient develops heart failure despite
maintaining a normal or high cardiac output because the circulation is unable to meet the oxygen
demand of the peripheral tissues.
Normally, blood flows from your arteries to your capillaries to your veins. Nutrients and oxygen
in your blood travel from your capillaries to tissues in your body.With an arteriovenous fistula,
blood flows directly from an artery into a vein, bypassing some capillaries. When this happens,
tissues below the bypassed capillaries receive a diminished blood supply.
Aortic valve vegetation can cause acute AR (usually without a dilated heart) and chronic AR
(which causes eccentric hypertrophy of the heart). Patients with infective endocarditis usually
appear sick with low-to-high-grade fevers, chills, or night sweats.
Pulmonary toxicity is a serious adverse effect of long-term amiodarone use and can occur
months to several years after the initiation of therapy. Interstitial pneumonitis due to amiodarone
presents with progressive dyspnea, nonproductive cough, and new reticular or ground-glass
opacities on chest radiograph
Scleroderma renal crisis typically presents with acute renal failure (without previous kidney
disease) and malignant hypertension (eg, headache, blurry vision, nausea). Urinalysis may show
mild proteinuria. Peripheral blood smear can show microangiopathic hemolytic anemia with
fragmented red blood cells (eg, schistocytes) and thrombocytopenia
Direct renin blockers enhances natriuresis, decreases serum angiotensin II concentration, and
decreases aldosterone production
ARBs prevent angiotensin II from acting on angiotensin receptors. Unlike ACE inhibitors, ARBs
do not decrease angiotensin II levels but do cause natriuresis and decreased aldosterone
production.
Dual antiplatelet therapy (aspirin and a P2y12 receptor blocker) leads to a reduction in recurrent
MI and cardiovascular death compared to aspirin alone in patients with NSTEMI. It also reduces
the risk of stent thrombosis and is recommended in all patients for at least 12 months following
drug-eluting stent placement
Acute type A aortic dissection can extend into the pericardial space, causing hemopericardium
and rapidly progresses to cardiac tamponade and cardiogenic shock. CT angiography is the initial
diagnostic study of choice in hemodynamically stable patients and reveals an intimal flap
separating the true and false lumens in the ascending or descending aorta.
Dihydropyridine CCB can cause peripheral edema and should always be considered in the
differential diagnosis of this condition, along with other causes, such as heart failure, renal
disease and venous insufficiency.
Adrenal vein sampling can differentiate between adrenal hyperplasia and adenoma,
which are causes of primary hyperaldosteronism. Although these patients can also develop
hypertension and hypokalemia, the plasma aldosterone is >15 ng/dl and suppresses the renin
through a feedback inhibition. As a result, the plasma aldosterone concentration/plasma renin
activity ratio is >20.
First-line treatment for erectile dysfunction in diabetic patients consists of PDE5i (eg, sildenafil).
It causes severe hypotension, especially in patients who are also taking alpha blockers or nitrates.
Dyspnea, orthopnea, lower extremity edema, displaced apical impulse, and bilateral lung
crackles are consistent with decompensated CHF due to LV systolic dysfunction. Hyponatremia
in patients with CHF usually parallels the severity of heart failure. In patients with CHF, low CO,
along with decreased perfusion pressure at the baroreceptors and renal afferent arterioles, leads
to neurohumoral activation with the release of renin, NE, and ADH. ADH (vasopressin) binds to
V2 receptors in the renal collecting ducts and promotes water reabsorption, while renin (via
angiotensin II) and NE increase proximal sodium and water reabsorption and limit water delivery
to the distal tubules. These actions promote free water retention and lead to dilutional
hyponatremia. Treatment involves fluid restriction, ACEi, and loop diuretics
Coarctation of the aorta is characterized by upper extremity HTN, diminished femoral pulses
with brachial-femoral delay, and a continuous murmur. Chest radiograph can reveal notching of
the third to eighth ribs (due to erosion by enlarged intercostal arteries) and a classic "3" sign due
to indentation of the aorta at the site of coarctation with pre- and poststenotic dilation.
RCA occlusion can cause acute inferior and posterior wall MI. Inferior wall MI presents with ST
elevations in the inferior leads (II, III, and aVF). ST depression in leads V1 and V2 suggests a
posterior wall MI. Inferior MI is also associated with hypotension, bradycardia, and AV block.
Occlusion of the left main coronary artery is usually a catastrophic event and presents as sudden
cardiac death in the majority of patients. Occlusion of both LCX and LAD causes ST-segment
elevations in anterior and lateral leads (I, AVL, V1-V6)
LAD usually supplies the anterior walls of the LV as well as anterior 2/3rd of the septum.
Occlusion causes ST elevation in all precordial leads, but mostly in V1-V4. Second-degree AV
block can be seen.
The LCX supplies the posterolateral wall of the LV. LCX occlusion can cause ST elevation in
leads I, II, III, and aVL. The ST depression in leads I and aVL seen in this patient is more
suggestive of RCA occlusion.
PAD is a coronary artery disease risk equivalent, and the medical therapy for such patients
should include aggressive risk factor modification with counseling for smoking cessation, lipid-
lowering therapy, and evaluation and treatment for HTN and DM. This patient is started
appropriately on low-dose aspirin and statin therapy. The next step is to enroll the patient in a
supervised exercise program. Pharmacologic therapy with cilostazol (vasodilator) and
percutaneous or surgical revascularization should be reserved for those with persistent symptoms
despite adequate supervised exercise therapy
A supervised graded exercise program is the most useful intervention to improve functional
capacity and reduce symptomatic claudication in patients with peripheral arterial disease.
Antiplatelet agents (eg, aspirin, clopidogrel) reduce overall cardiovascular mortality. Lipid-
lowering therapy with statins should also be given to all patients with clinically significant
atherosclerotic cardiovascular disease. Surgical revascularization is reserved for patients with
limb-threatening complications (eg, nonhealing ulcers), significant limitation in activities of
daily living, or failure to respond to exercise and pharmacologic therapy (cilostazol)
Aortic dissection involving the ascending aorta is associated with a mortality rate of 1%-2% per
hour following symptom onset, and emergent surgical repair is warranted to prevent
complications and death. Retrograde extension of the intimal tear can involve the aortic valve
and cause acute AR, affected patients can develop sudden onset of worsening chest pain,
hypotension, and pulmonary edema, along with the early decrescendo diastolic murmur of AR.
Ascending aortic dissection can propagate proximally from the site of intimal tear to involve the
aortic valve and cause acute aortic regurgitation
Aortic dissection involving the carotid or vertebral arteries can cause cerebral hypoperfusion,
leading to stroke or transient ischemic attacks.
Ascending aortic dissection can propagate proximally into the pericardial space and lead to
hemopericardium and cardiac tamponade. Patients with cardiac tamponade present with
hypotension, pulsus paradoxus, elevated JVP, and cardiogenic shock with clear lung fields.
This patient has orthostatic hypotension, which is defined as a postural decrease in systolic BP
by 20 mmHg or 10 mmHg diastolic (sometimes accompanied by an increase in heart rate) that
occurs on standing. In general, orthostatic hypotension results from insufficient constriction of
capacitance blood vessels in the lower extremities on standing, which may be due to a defect in
autonomic reflexes, decreased intravascular volume, or medications. Some baroreceptor
sensitivity is lost as a normal part of aging. Arterial stiffness, decreased NE content of
sympathetic nerve endings, and reduced sensitivity of the myocardium to sympathetic
stimulation all contribute to a tendency toward orthostatic hypotension with age
An ECG is the essential first step in a patient with history and risk factors for coronary artery
disease who has symptoms consistent with ACS. There are a number of well-known atypical
presentations (ie, no overt chest pain) of ACS that should prompt a cardiac evaluation. Women,
the elderly, and patients with diabetes in particular are more likely to have atypical symptoms
(Dyspnea, epigastric pain, and nausea/vomiting)
Electrical alternans with sinus tachycardia is a pathognomonic ECG finding for pericardial
effusion. It is due to the swinging motion of the heart in the pericardial cavity that causes a beat-
to-beat variation in QRS amplitude. Large pericardial effusions can also cause low voltage of
QRS complexes on ECG. An echocardiogram is used to confirm the pericardial effusion in
hemodynamically stable patients. Treatment involves emergency pericardiocentesis for rapid
symptom resolution and improvement of hypotension
A temporary transvenous pacemaker is used to pace the heart before a permanent pacemaker can
be placed. It is indicated in patients with sick sinus syndrome or symptomatic second-degree or
third-degree heart block.
Hepatitis A is the most common vaccine-preventable disease among travelers. The hepatitis A
vaccine should be considered for people anticipating travel to developing countries.
In otherwise young healthy patients who develop CHF, myocarditis should be considered high on the
differential. Viral infection, especially with Coxsackie B virus, is the most common cause.
Elevated levels of circulating BNP correlate with the severity of LV dysfunction and are useful in
differentiating CHF from other causes of dyspnea. In studies of patients presenting with acute
dyspnea, most patients with dyspnea due to CHF had plasma BNP levels >400 pg/ml whereas
levels <100 pg/ml had a high negative predictive value for CHF as a cause of dyspnea. Such low
values should prompt the search for noncardiac causes of dyspnea.
High-dose niacin therapy to treat lipid abnormalities frequently produces cutaneous flushing and
pruritus. This side effect is explained by prostaglandin- induced peripheral vasodilatation and can
be reduced by low-dose aspirin
Furosemide commonly causes hypokalemia and hypomagnesemia which can lead to ventricular
tachycardia. Furthermore, hypokalemia potentiates side effects of digoxin
Most cases of first-degree AV block with normal QRS duration are due to delayed AV nodal
conduction and require no further evaluation. Patients with first-degree AV block and prolonged
QRS duration likely have a conduction delay below the AV node and should have ECG testing to
determine its nature.
This patient's clinical presentation - recent URI, dyspnea, elevated JVP, clear lung fields, and
increased cardiac silhouette on chest x-ray is suggestive of early cardiac tamponade due to a
large pericardial effusion. Pericardial effusion is most commonly idiopathic (thought to be likely
from a viral illness). Large pericardial effusions typically appear on chest x-ray as an enlarged
and globular cardiac silhouette with clear lung fields. The inability to palpate the point of
maximal apical impulse is consistent with large pericardial effusion. Patients with cardiac
tamponade may also have Beck's triad of hypotension, elevated JVP, and muffled heart sounds.
Physical examination findings of effusion without cardiac tamponade include diminished heart
sounds on auscultation and a maximal apical impulse that is difficult to palpate
Fixed splitting of the second heart sound is heard with ASD. Chest x-ray may show an enlarged
right atrium and ventricle along with prominent hilar or proximal pulmonary arterial vasculature.
An opening snap is heard with MS causing LA enlargement, which presents on chest x-ray as
straightening of the left heart border, elevated left main bronchus, and pulmonary vascular
congestion with cephalization of the blood flow to upper lobes. Lateral chest x-ray projection can
reveal posterior displacement and impingement of the esophagus.
BB, CCB, and nitrates are anti-anginal agents that should be withheld for at least 48 hours prior
to cardiac stress testing. However, these medications should be continued in patients with known
coronary artery disease undergoing stress testing to assess the efficacy of antianginal therapy.
Diuretics do not directly affect the accuracy of stress testing. However, diuretic induced
hypokalemia (potassium <3 mEq/L) can cause ST depression and false positive results on
exercise stress testing. Testing should be delayed until potassium has been repleted
ACEi, ARB, and statins do not affect the diagnostic accuracy of stress testing for CAD. These
medications should not be withheld prior to stress testing.
Dipyridamole can be used during myocardial perfusion scanning to reveal the areas of restricted
myocardial perfusion. The redistribution of the coronary blood flow to 'non-diseased' segments
induced by this drug is called coronary steal phenomenon. Dipyridamole and adenosine are
coronary vasodilators. Infusion of these substances in patients without coronary artery disease,
increases coronary blood flow three to five times above the baseline levels. However, in patients
with coronary artery disease, the diseased vessels distal to the obstruction are already maximally
dilated, and their ability to increase myocardial perfusion is limited; therefore, redistribution of
coronary blood flow to 'non-diseased' areas occurs, and the perfusion of 'diseased' segments
diminishes known as coronary steal
Digoxin is a cardiac glycoside with adverse effects that include nausea, vomiting, diarrhea,
vision changes, and arrhythmias. Patients chronically taking digoxin should have routine
monitoring of their digoxin levels. An inciting event, such as a viral illness or excessive diuretic
use, can lead to volume depletion or renal injury that acutely elevates the digoxin level.
Hypokalemia, often associated with loop diuretic use, increases the patient's susceptibility to the
toxic effects of digoxin.
PSVT is the MC paroxysmal tachycardia in people without structural heart disease. Attacks
begin abruptly and are characterized by HR between 160-220bpm. The MC mechanism
underlying PSVT is re-entry into the AV node. Mechanical and medical therapies for PSVT
decrease AV node conductivity. Vagal maneuvers, such as Valsalva, carotid sinus massage, and
immersion in cold water increase vagal tone and decrease conduction through the AV node. This
slows the HR and can often break the rhythm. Adenosine is a very short acting AV-nodal blocker
that is often used as well
The sudden onset of severe chest pain radiating to the back and widened mediastinum on chest x-
ray are suggestive of aortic dissection. The presence of hypotension, tachycardia, JVD, and
respiratory variation in systolic blood pressure (ie, pulsus paradoxus) are all consistent with
cardiac tamponade. Cardiac tamponade can occur as a complication of aortic dissection, with
rupture of the aorta and rapid accumulation of blood in the pericardial space. An abrupt
accumulation of even small amounts of blood can significantly raise the pressure inside the
pericardial cavity. Increased pericardial pressure causes compression of cardiac chambers and
limits diastolic filling of the right-sided chambers. This causes a decrease in preload and reduces
CO, resulting in hypotension and syncope
Vagal hyperactivity is the underlying mechanism behind vasovagal syncope. This disorder is
characterized by prodromal symptoms, including dizziness, nausea, diaphoresis, pallor, and
visual disturbances prior to syncope
The MC sign and symptoms of PE include dyspnea, pleuritic chest pain, tachypnea, and
tachycardia. Chest radiograph is frequently abnormal but has poor sensitivity and specificity for
pulmonary embolism, and may be normal
The edema is likely related to preferential dilation of precapillary vessels (arteriolar dilation),
which leads to increased capillary hydrostatic pressure and fluid extravasation into the
interstitium. Dihydropyridine (DHP) CCBs such as amlodipine and nifedipine are potent
arteriolar dilators and cause more peripheral edema than non-dihydropyridine CCBs (eg,
diltiazem, verapamil).
ACEi or ARBs cause post-capillary venodilation and can normalize the increased capillary
hydrostatic pressure. The combination of CCB and ACEi is associated with a significantly lower
risk of CCB-associated peripheral edema compared with CCB monotherapy
The S3 is believed to result when inflow from the LA strikes blood that is already in the LV
causing reverberation of blood between the left ventricular walls. This sound can be normal in
younger individuals and well-trained athletes, its appearance later in life is often a sign of LV
failure. IV diuretics provide symptomatic benefits to patients with decompensated heart failure.
Patients with ADPKD usually present with hypertension, hematuria, proteinuria, palpable renal
masses, or progressive renal insufficiency. They may also have flank pain due to renal calculi,
cyst rupture or hemorrhage, or URI. Hypertension is a common early finding in patients with
ADPKD and usually precedes the decline in renal function.
This patient was initially admitted for chest pain (possible unstable angina) and appropriately
treated. His subsequent acute-onset dyspnea with wheezing and prolonged expiration indicates
bronchoconstriction. A history of rhinitis and eczema and his intermittent dyspnea and coughing
suggest undiagnosed asthma now exacerbated by aspirin or possibly BB therapy. Aspirin is a
common trigger for bronchoconstriction in patients with asthma, especially those with concurrent
chronic rhinitis and nasal polyps.
Carotid endarterectomy (CEA) is indicated in patients with symptomatic carotid artery stenosis
of 70%-99%. Men with asymptomatic carotid artery stenosis of 60%-99% may also benefit from
CEA compared to medical management alone. All patients with cerebrovascular disease should
be started on antiplatelet agents and statins with optimization of risk factors (eg, smoking
cessation, diabetes mellitus and hypertension control).
Cardiogenic shock causes a reduced cardiac index and elevated PCWP due to ventricular pump
failure. SVR is typically increased to maintain adequate tissue perfusion pressure. The lower
tissue perfusion signals the tissue to extract more oxygen from the blood, which decreases mixed
venous oxygen saturation. All of this manifests clinically as depressed mental status, dyspnea,
decreased urinary output, and cool extremities.
Congenital bicuspid aortic valve is the MCC of isolated AR in young adults in developed
countries. AR leads to an early decrescendo diastolic murmur, best heard along the left third and
fourth intercostal spaces while the patient is sitting up, leaning forward, and holding a breath in
full expiration. Rheumatic heart disease involving the aortic valve is the MCC of AR in the
developing world.
ASD usually presents with a wide and fixed splitting of the S2. An ejection systolic murmur can
be heard over the left second intercostal space due to increased blood flow across the pulmonic
valve. Some patients with large ASDs and significant left-to-right shunting also have a mid-
diastolic murmur resulting from increased flow across the tricuspid valve.
This patient's crescendo-decrescendo systolic murmur along the left sternal border without
carotid radiation is the description of the murmur present in HOCM, which is interventricular
septal hypertrophy. Syncope in HOCM is often multifactorial and can be due to outflow
obstruction, arrhythmia, ischemia, and a ventricular baroreceptor response that inappropriately
causes vasodilation.
Acute peri-infarct pericarditis can occur within 1-3 days after an Ml and can present with a
pericardial friction rub with or without chest pain. It is self-limited and resolves with supportive
care. Dressler’s syndrome is post-MI pericarditis occurring weeks to months after MI. Patients
present with pleuritic chest pain and pericardial friction rub and usually improve with NSAIDs
Abrupt LV rupture often leads to hemopericardium and eventual cardiac tamponade. Blood in the
pericardial sac compresses the LV and decreases stroke volume, resulting in hypotension with
compensatory sinus tachycardia. The severe mechanical compromise can rapidly progress to
PEA with the ECG showing low voltage from the ensuing cardiac tamponade. LV free wall
rupture should be suspected in patients with PEA after recent first MI and no signs of heart
failure. Rapid diagnosis with echocardiogram, supportive care, pericardiocentesis, and possible
surgical repair are required to save the patient's life.
IV wall rupture is similar to ventricular free wall rupture and both have a peak incidence around
5 days after infarction. However, IV wall rupture causes a VSD and not pericardial tamponade.
Patients typically present with sudden onset of hypotension, CHF (predominantly right heart
failure), and a loud holosystolic murmur heard best at the lower left sternal border
Papillary muscle rupture occurs 3-5 days after infarction and can cause hypotension secondary to
severe acute mitral regurgitation (holosystolic murmur at the apex). Although acute in onset, it
typically presents with hypotension and pulmonary edema.
His ECG shows an irregularly irregular rhythm with varying R-R intervals, no clearly discernible
P waves, and narrow-complex tachycardia consistent with AF with rapid ventricular response.
AF can be asymptomatic or present with symptoms such as palpitations, weakness, dizziness,
presyncope, dyspnea, and/or chest pain. Hyperthyroidism is the most common and likely cause
of sudden-onset AF. All such patients with new-onset AF should have TSH and free T4 levels
measured to screen for occult hyperthyroidism as an underlying cause
High-frequency hearing loss is associated with aging and certain congenital long QT syndromes.
Patients with congenital long QT or short QT syndrome have an increased risk of developing AF
but is uncommon
ADPKD is occasionally associated with valvular abnormalities (eg, mitral valve prolapse) that
can progress and possibly predispose to AF but is uncommon
Patients with suspected acute arterial occlusion leading to an immediately-threatened limb (eg,
sensory loss, pain at rest, muscle weakness) should be immediately started on anticoagulation
with intravenous heparin prior to further evaluation with noninvasive or invasive imaging
Torsades de pointes (TdP) refers to polymorphic ventricular tachycardia that occurs in the setting
of a congenital or acquired prolonged QT interval. Immediate defibrillation is indicated in
hemodynamically unstable patients with TdP, while intravenous magnesium is the first-line
therapy for stable patients with recurrent episodes of TdP.
Amiodarone is a class III antiarrhythmic, and is used both for atrial and ventricular tachycardia.
It is used occasionally in patients with polymorphic VT (and normal baseline QT interval) due to
myocardial ischemia or infarction.
Sodium bicarbonate is useful in the management of patients with Torsades de pointes due to
quinidine use. It is also beneficial in patients with cardiac arrest due to metabolic acidosis,
hyperkalemia, or tricyclic antidepressant overdose
Sotalol is a class III antiarrhythmic agent that causes QT prolongation and TdP. Patients being
initiated on sotalol therapy for other indications should be admitted to the hospital for serial ECG
and cardiac rhythm monitoring
This patient has a regular, narrow-complex SVT causing hemodynamic instability with
hypotension and signs of poor perfusion (cool extremities). Common causes of regular, narrow-
complex SVT include inappropriate sinus tachycardia, atrial tachycardia, atrial flutter, and AV
nodal reentrant tachycardia. All patients with persistent tachyarrhythmia (narrow- or wide-
complex) causing hemodynamic instability (eg, hypotension, signs of shock, ischemic chest
discomfort, mental status changes, acute pulmonary edema) should be managed with immediate
synchronized direct current cardioversion due to the risk of rapid clinical deterioration.
IV antiarrhythmics (eg, amiodarone, procainamide) can be used in patients with stable recurrent
or refractory wide-complex tachycardia .These drugs can worsen hypotension and are not
recommended acutely in patients with hypotension or hemodynamic instability
IV BB (eg, metoprolol, esmolol) or CCB (eg, verapamil, diltiazem) can be occasionally used in
stable patients who fail to respond to initial vagal maneuvers and/or IV adenosine. They should
not be used in patients with persistent hypotension and hemodynamic instability
This patient's presentation with exertional dyspnea, pounding heart sensation, and widened
pulse pressure is most likely due to chronic AR. In chronic AR, a portion of LV output leaks
back into the LV, causing an increase in LVEDV, myocardial hypertrophy, and chamber
enlargement. The increase in LV size brings the ventricular apex close to the chest wall,
causing a pounding sensation and an uncomfortable awareness of the heartbeat especially in
the left lateral decubitus position. Patients have widened pulse pressure (increased systolic
BP and decreased diastolic BP)
Patients with AS have a characteristic pulse with delayed upstroke (pulsus tardus), delayed peak,
and small pulse amplitude (pulsus parvus). Pulse pressure remains normal in patients with AS.
Patients presenting to the emergency department with chest pain and suspected acute coronary
syndrome (ACS) should be administered aspirin as soon as possible. Early antiplatelet therapy
with aspirin reduces the rate of myocardial infarction and overall mortality in patients with ACS.
Heparin is used for MI (positive cardiac markers or ST elevation on ECG) or a high suspicion
and/or confirmation of PE.
Sublingual nitroglycerin is used as a first-line agent for rapid relief of symptoms in patients with
angina pectoris. The anti-ischemic effect of nitrates is mediated by systemic vasodilation with a
decrease in LVEDV and wall stress resulting in decreased myocardial oxygen demand
BB and CCB exert an antianginal effect partially via a decrease in the HR (negative chronotropic
effect) or contractility (negative inotropic effect). Nitrates do not have a direct effect on cardiac
chronotropy; they may cause decreased inotropy due to decreased preload, but their antianginal
effect is thought to be due primarily to a reduction in wall stress
Patients with HOCM symptoms (eg, syncope, heart failure, angina) should be treated with
negative inotropic agents (eg, BB, verapamil, disopyramide) as the initial medical therapy. BB
(eg, metoprolol, atenolol) are the most commonly used agents for initial monotherapy. They
prolong diastole and decrease myocardial contractility, which in turn decreases LVOT
obstruction and improves symptoms of angina. Non-dihydropyridine CCB (eg, verapamil) and
disopyramide are sometimes used when symptoms persist despite BB therapy
Variant angina is managed with CCB and/or nitrates to prevent coronary vasoconstriction.
Nonselective BB and aspirin should be avoided because they can promote vasoconstriction.
This patient has variant angina (Prinzmetal's angina), which causes chest pain by coronary
vasospasm. It typically occurs in young females, and the greatest risk factor is smoking. Aside
from smoking, affected patients often lack cardiovascular risk factors. The episodes occur at
night (from midnight to 8 am) and can be associated with transient ST elevations on ECG. The
treatment involves elimination of risk factors such as smoking, as well as pharmacologic therapy
with CCB or nitrates. These medications work in variant angina by promoting vasodilation and
preventing vasoconstriction.
Aspirin should be avoided in variant angina because it causes prostacyclin inhibition, which may
promote coronary vasospasm
Medications that improve long-term survival in patients with LV systolic dysfunction include
BB, ACEi, ARBs, mineralocorticoid receptor antagonists (spirinolactone, epleronone), and (in
African American patients) a combination of hydralazine and nitrates.
Excessive vagal tone is the mechanism behind vasovagal syncope. This disorder is characterized
by nausea, diaphoresis, and pallor prior to syncope. It frequently occurs in response to stress,
pain, and certain actions (eg., urination).
In patients with CHF, activation of the RAAS and production of angiotensin II causes
vasoconstriction of efferent renal arterioles, which increases intraglomerular pressure in order to
maintain adequate GFR
Systemic hypertension is the most important predisposing risk factor associated with aortic
dissection. Marfan syndrome is responsible for almost 50% of the aortic dissections seen in
patients age < 40; however, it is an uncommon cause in patients age > 60.
Patients with RVMI are initially treated similarly to others with acute MI such as dual antiplatelet
therapy, statins, anticoagulation, and urgent revascularization (thrombolytics or PCI). Patients
with RVMI require a high preload to maintain cardiac output. Therefore, patients with
hypotension and low/normal JVP should be given high-flow IV fluids to increase RV preload (if
JVP is elevated, IV fluids are less likely to be helpful). Drugs that decrease preload, such as
nitrates, diuretics and opioids, can cause profound hypotension and should be avoided. Drugs
that slow the HR (BB) or decrease contractility (CCB) should be used with caution.
This patient's presentation is consistent with digoxin toxicity, most likely due to addition of
amiodarone to his medication regimen. Acute digoxin toxicity typically presents with
GI symptoms (eg, anorexia, nausea, vomiting, abdominal pain), along with possible weakness
and confusion. Chronic digoxin toxicity presents with less pronounced GI symptoms but more
significant neurologic (eg, lethargy, fatigue, confusion, disorientation) and visual (eg, changes in
color vision, scotomas, blindness). Amiodarone (or verapamil, quinidine, and propafenone)
increases the serum levels of digoxin and can lead to toxicity in a patient who has previously
been on a stable digoxin regimen. It is recommended that the digoxin dose be decreased by
25%-50% when initiating amiodarone therapy, with close monitoring of digoxin levels once
weekly for the next several weeks
This patient has symptomatic AF with rapid ventricular response. She also has additional features
(weight loss, lid lag, hand tremor) to suggest hyperthyroidism as the underlying cause. AF is the
most common supraventricular arrhythmia in hyperthyroidism. Thyroid hormones cause an
increase in beta-adrenergic receptor expression, which leads to an increase in sympathetic
activity. BB (eg, propranolol, atenolol) are recommended as initial therapy to control HR and
hyperadrenergic symptoms. In addition, propranolol decreases conversion of T4 to T3 in
peripheral tissues. The BB should be initiated as soon as hyperthyroidism is diagnosed and
should be continued until the hyperthyroidism is adequately treated with thionamides,
radioiodine, or surgery
Adenosine induces a transient block at the AV node and is often used in patients with SVT in
whom the diagnosis is unclear. Its short duration of action limits its role in management of
patients with AF, in whom it can have proarrhythmic effects.
Intravenous BB is the treatment of choice for the initial management of acute aortic dissection as
they lower HR and BP and reduce LV contractility.
Hydralazine and Nitroprusside are vasodilators that are sometimes used in hypertensive
emergencies. However, they can cause reflex sympathetic stimulation with consequent rises in
HR, LV contractility, and aortic wall stress. These changes increase the risk of propagation of an
aortic dissection. In patients with aortic dissection, nitroprusside is used as a second-line agent
only if BB therapy is inadequate
Cardiac catheterization describes a procedure in which small catheters are inserted into arteries
or veins and snaked up into the heart. Cardiac angiography refers specifically to one of the
procedures that can be done during cardiac catheterization where a contrast dye visible in X-rays
is injected through the catheter.
Rheumatology
Psoriatic arthritis involves the DIP joints, morning stiffness, deformity, dactylitis ("sausage
digit"), and nail involvement. Current treatment options for psoriatic arthritis include NSAIDs
methotrexate, and anti-tumor necrosis factor agents
This patient's presentation with 3-4 weeks of dyspnea, fatigue, and nonproductive cough with
bilateral hilar fullness and interstitial infiltrates on chest x-ray is consistent with sarcoidosis.
Hypercalcemia causing frequent urination can be an additional clue for diagnosis. The
mechanism responsible for hypercalcemia is granulomatous activity of 1-alpha hydroxylase
causing increased production of 1,25-dihydroxyvitamin D and increased intestinal calcium
absorption. Hypercalciuria is present more often than hypercalcemia and can result in
nephrolithiasis. Patients with sarcoidosis who are asymptomatic are often followed without
treatment due to a high rate of spontaneous remission. However, patients with symptomatic
disease (eg, cough, dyspnea, fatigue, hypercalcemia) are treated with systemic glucocorticoids. It
is important to rule out infectious granulomatous diseases with similar presentations, such as TB
and histoplasmosis, prior to initiation of glucocorticoid
Antifungal therapy (eg, itraconazole) would be indicated for treatment of infection with endemic
fungi such as histoplasmosis. Sarcoidosis and histoplasmosis may present with similar symptoms
and chest x-ray findings, but infection in an immunocompetent host is extremely rare and
leukocytosis would be expected
Radial tunnel syndrome causes posterolateral elbow pain similar to that in lateral epicondylitis
and may occur in conjunction with that condition. Findings include weakness of extension at the
wrist and third digit, reproduction of pain on resisted supination of the forearm, and pain at the
radial tunnel on resisted hyperextension of the wrist
This patient with SLE, who is on prednisone, has atraumatic hip pain with normal x-ray findings,
most likely due to osteonecrosis (avascular necrosis) of the femoral head. Osteonecrosis can
occur in disorders that disrupt the circulation of bone through micro-occlusion, abnormal
endothelial function, or increased intra-osseus pressure. It is a common complication of SLE, and
the risk is greatly increased in patients treated with glucocorticoid. Osteonecrosis of the femoral
head is characterized by pain in the groin, thigh, or buttock that is worsened by activity. Early
examination findings may be normal, but progression of the disease can lead to reduced range of
motion (particularly internal rotation and abduction) and joint instability. Early examination
findings and x-rays may be normal, and MRI is a more sensitive test
Acute gout presents as an acute monoarthritis, usually in the first MTP joint or knee. Synovial
fluid analysis shows leukocytosis (2,000-100,000/mm with > 50% neutrophils) and monosodium
urate crystals, which are needle-shaped and negatively birefringent. Patients generally respond to
NSAIDs (eg, indomethacin), colchicine, or corticosteroids (systemic or intraarticular). Long-
term prophylactic medications (eg, allopurinol) are indicated for patients with recurrent attacks.
Calcium pyrophosphate dihydrate deposition (CPPD) can also cause an acute inflammatory
arthritis ("pseudogout"), usually in the knee. Diagnosis is confirmed with synovial fluid analysis
showing calcium pyrophosphate crystals (rhomboid-shaped, and weakly positively birefringent)
In patients with gout, synovial crystals may be seen between flares, so the presence
of these crystals alone does not rule out septic arthritis. Septic arthritis can cause acute
monoarticular arthritis, but it is typically associated with fever, systemic manifestations (eg,
chills), and progressively worsening symptoms over a few days, rather than the abrupt onset seen
in gout (24 hours)
Enthesitis is characterized by inflammation and pain at sites where tendons and ligaments attach
to bone. Enthesitis at the insertion of the Achilles tendon at the heel is often the most prominent
presentation. However, enthesitis can also be seen at the costosternal junction, shoulders, elbows,
hips, iliac crests, tibial tuberosities, and other locations. Chronic complications of enthesitis
include fibrosis and calcification. It is a common finding in ankylosing spondylitis and other
spondyloarthropathies. It is particularly prominent in AS and classically manifests with heel pain
at the insertion of the Achilles tendon that worsens with activity. Enthesitis of multiple regions is
also frequently seen. Low back pain (that classically improves with activity), loss of spinal
mobility, and peripheral arthritis are other common musculoskeletal manifestations of AS.
Lumbosacral strain is the MCC of acute back pain. The typical clinical scenario includes acute
onset of the back pain after physical exertion, absence of radiation, presence of paravertebral
tenderness, negative straight-leg raising test, and normal neurologic examination
This patient has podagra from an acute attack of gout. Gout occurs as a result of overproduction
or underexcretion of uric acid. Overproduction of uric acid is typically related to consumption of
a high-urate diet, but any condition that increases catabolism and turnover of purines can raise
uric acid levels and trigger a gout attack. Common causes include hematologic malignancies,
tumor lysis syndrome, and psoriasis. Myeloproliferative disorders are common secondary causes
of gout. This patient has several clinical features suggesting polycythemia vera (PV), including
pruritus triggered by hot baths (aquagenic pruritus), headaches, and hepatosplenomegaly. PV is
characterized by increased cell turnover due to clonal hyperproliferation in all 3 primary bone
marrow lineages (ie, red cells, white cells, platelets). Up to 40% of patients with PV have gout.
Allopurinol inhibits uric acid formation and is used to prevent gout attacks in patients with
hyperuricemia due to PV
This patient has a number of findings (eg, esophageal dysmotility, fibrotic lung disease,
arthralgias) consistent with extradermal manifestations of systemic sclerosis (SSc). SSc causes
smooth muscle atrophy and fibrosis in the lower esophagus. Common symptoms include
dysphagia, choking, heartburn, and hoarseness. Esophageal manometry in affected patients
typically shows hypomotility and incompetence of the LES
Achalasia presents with dysphagia and regurgitation of undigested food. As in SSc, manometry
shows aperistalsis in the distal esophagus. However, achalasia causes increased LES pressure
and incomplete LES relaxation, whereas SSc causes decreased LES pressure
Most patients with acute, uncomplicated low back pain experience spontaneous resolution of
their symptoms in the first few weeks. Patients should be advised to continue moderate activity.
Nonopioid analgesics (eg, NSAIDs, acetaminophen) are preferred as initial management
Physical therapy referral for a supervised exercise program is appropriate for patients with
persistent pain (6-12 weeks or longer). Most patients with acute pain improve without physical
therapy intervention.
Anterior cruciate ligament (ACL) injuries are common in young athletes in sports requiring rapid
direction changes or twisting movements of the lower extremity. ACL injuries usually present
with rapid onset of pain and a “popping sensation” in the knee followed by rapid onset of
swelling with hemarthrosis and a feeling of instability when bearing weight on the affected leg.
Examination findings include laxity of anterior motion of the tibia relative to the femur.
Medial collateral ligament (MCL) tears may also occur in pivoting/twisting injuries or if the knee
is struck from the lateral side with the foot planted. Examination shows tenderness at the medial
knee, and laxity when the foot is gently forced into abduction with the knee stationary (valgus
stress test). However, MCL injuries are not usually associated with significant hemarthrosis
unless there is a concurrent ACL injury
Meniscal tears are possible in athletes subjected to rapid direction changes and are characterized
by a subacute or chronic locking or popping sensation in the knee. However, immediate
symptoms at the time of the injury may be mild. Effusions are possible but typically develop
slowly, and hemarthrosis is rare.
This patient has characteristic features of Behcet syndrome, including recurrent painful oral and
genital aphthous ulcers, uveitis, and erythema nodosum (tender red nodules usually in the
pretibial area). These patients are also at high risk of vascular disease (vasculitis) with venous
and arterial thrombosis. Patients may also demonstrate pathergy, an exaggerated ulcerating skin
response following minor injuries (eg, needlestick)
Herpes simplex can cause oral and genital ulcers and uveitis. However, it presents as vesicles
(which coalesce to form lobe-shaped ulcers) that recur at the same location. The combination of
erythema nodosum, recurrent ulcers, and uveitis makes Behcet syndrome more likely
SLE is a multisystem disease typically presenting with a malar rash, arthritis, and constitutional
symptoms (eg, fever). Oral ulcers are a common feature, but are typically painless
This patient has features of malignant pain (age > 50, nocturnal pain, weight loss). Plain x-rays
can be performed quickly and can identify signs of malignant disease, infection, or vertebral
compression. Inflammatory markers (ESR, C-reactive protein) can increase sensitivity and are
also advised. If the ESR or x-rays are abnormal, MRI should be done to evaluate for possible
cancer or spinal infection. MRI is also indicated for patients with significant neurologic deficits
or features of cauda equina syndrome (eg, saddle anesthesia, urine retention). Radionuclide bone
scan can be considered for patients who are not able to have an MRI. For patients in whom
multiple myeloma is suspected (eg, bone pain, anemia, hypercalcemia, kidney injury), skeletal
surveys (including x-rays of chest, spine, skull, and pelvis) are preferred to a bone scan as they
better detect the lytic lesions seen in MM
This patient has uncomplicated chronic (~12 weeks) low back pain (LBP). Patients with acute (<
4 weeks) LBP should be advised to maintain moderate activity with short courses of
acetaminophen or NSAIDs to provide pain relief. However, those with chronic ( 4-12 weeks)
LBP are more likely to continue to have recurrent or persistent pain. In patients with chronic
LBP, exercise has proven to be beneficial in reducing pain and improving function. Patients often
start with a supervised exercise program that emphasizes stretching and strengthening of the
back muscles. Some patients may benefit from tricyclic antidepressants or duloxetine, but
opioids, benzodiazepines, and muscle relaxants are not advised.
Osteonecrosis occurs in disorders that disrupt the circulation of bone through micro-occlusion,
abnormal endothelial function, or increased intraosseous pressure. It is a common complication
of long-term glucocorticoid use, possibly due to effects on osteocytes or abnormal plasma lipid
levels causing microemboli. Osteonecrosis causes bone and bone marrow infarction. Abnormal
bone remodeling subsequently results in trabecular thinning and collapse over months to years.
Osteonecrosis of the femoral head is characterized by pain in the groin, thigh, or buttock that is
worsened by activity and relieved by rest. Progression of the disease can lead to reduced range of
motion (usually abduction and internal rotation), rest pain, and joint instability. In the first few
months, x-rays are often normal, and MRI is a more sensitive test
Ankylosing spondylitis (AS) is characterized by chronic inflammatory back pain and stiffness,
lumbosacral tenderness, and reduced spinal range of motion. In young patients with
characteristic pain, plain x-rays showing sacroiliitis can confirm the diagnosis. AS is strongly
associated with HLA-B27, but HLA-B27 is not specific for AS and not necessary for diagnosis
Methotrexate (MTX) is a folate antimetabolite and the preferred initial DMARD of choice in
patients with moderate-to-severe RA. Common side effects of MTX include GI symptoms, oral
ulcers/stomatitis, rash, alopecia, hepatotoxicity, pulmonary toxicity, and bone marrow
suppression. Folic acid supplementation reduces the SE of MTX therapy without loss of efficacy
Stress fractures are common in patients who suddenly increase their physical activity, especially
in female runners with the female athlete triad (oligomenorrhea, osteoporosis, and decreased
caloric intake). Typical symptoms include localized pain to palpation and possible swelling
In patients with suspected GCA and visual loss, the most appropriate next step is to initiate
therapy with high-dose IV corticosteroids, followed by oral therapy with a slow taper over
several months. Immediate therapy with systemic corticosteroids reduces the progression of
visual complications in the affected and unaffected eyes; it should be started promptly while
awaiting confirmation of the diagnosis. Physical examination may show diminished pulses and/
or bruits on auscultation of the axillary, brachial, or femoral arteries and over the supraclavicular
and/or carotid areas. Anterior ischemic optic neuropathy is the most common ocular
manifestation and is detected on funduscopy by the presence of a swollen and pale disc with
blurred margins.
CTS is caused by compression of the median nerve where it passes under the transverse carpal
ligament (flexor retinaculum) in the wrist. Patients have reproduction of pain on percussion over
the median nerve at the wrist (Tinel sign) or by holding the wrists in full flexion with the dorsum
of the hands pressed together (Phalen test). Most patients respond to nocturnal wrist splinting,
but those with significant weakness or refractory symptoms may require surgical decompression
Septic arthritis is more common in patients with underlying joint abnormalities, such as
osteoarthritis, RA, or prosthetic joints. Patients with RA are at especially high risk due to
impaired phagocytosis and systemic immunosuppression. Occasionally, septic arthritis in
patients with underlying RA may present without overt signs of infection (eg, fever, chills,
peripheral leukocytosis); this is especially common in elderly patients and may lead to a delayed
diagnosis. Diagnosis of septic arthritis is confirmed with synovial fluid analysis showing
elevated leukocytes (50,000-150,000), positive Gram stain, and positive fluid culture. Blood
cultures are positive in 50% of patients and should be obtained prior to antibiotic therapy.
Treatment of septic arthritis requires prompt administration of IV antibiotics and adequate joint
drainage to lessen the likelihood of joint destruction
RA is a chronic, systemic inflammatory disorder with progressive erosion of the bone and
cartilage and significant joint destruction and deformity. Patients present with morning stiffness,
involvement of PIP and MCP joints, and radiographic evidence of erosions and/or periarticular
osteopenia. Treatment goals in RA are to induce and maintain early remission, control synovitis,
and prevent progression of joint damage. All patients diagnosed with RA should be started on
DMARDs as soon as possible as joint damage begins early in its course. NSAIDs and COX-2
inhibitors (eg, celecoxib) are adjunctive therapies for symptomatic relief but do not reduce
disease progression. Glucocorticoids can also relieve symptoms but are not effective in
preventing eventual joint destruction. In fact, they can result in generalized bone loss (ie,
osteoporosis). DMARDs include nonbiologic agents (MTX, hydroxychloroquine, sulfasalazine,
leflunomide, azathioprine) and biologic agents (eg, etanercept, infliximab, adalimumab,
tocilizumab, rituximab). Methotrexate is the preferred initial DMARD in patients with
moderately to severely active RA due to its efficacy and long-term safety profile. Patients should
be tested for hepatitis B and C and tuberculosis before starting therapy. Methotrexate should not
be used during pregnancy and those with severe renal insufficiency, liver disease, or excessive
alcohol intake. Patients who do not respond after 6 months may require biologic DMARDs such
as tumor necrosis factor-alpha inhibitors (eg, etanercept, infliximab) as step-up therapy
All RA patients should receive DMARDs as early as possible in the disease course. MTX is the
initial DMARD of choice in most patients with active RA. NSAIDs or glucocorticoids should be
used for initial temporary symptomatic relief while awaiting response to DMARD therapy
This patient has systemic symptoms (fatigue and weight loss), inflammatory arthritis, serositis
(pericardial friction rub), pancytopenia, and protein-losing nephropathy (edema, proteinuria,
microscopic hematuria). In a young woman, this is very concerning for SLE, even in the absence
of the characteristic rash. The initial laboratory evaluation in suspected SLE should include
blood counts to screen for cytopenias as well as urinalysis and creatinine to screen for
nephropathy. In addition, ANA titer should be obtained early in the evaluation as it is a very
sensitive marker for SLE. If ANA is elevated, more specific autoantibodies can be obtained to
confirm the diagnosis. In addition, complement levels and ESR or C-reactive protein should be
assayed to assess ongoing disease activity
Anti-double-stranded DNA and anti-Smith antibodies are specific markers for SLE. However,
they are less sensitive than ANA, and many patients with SLE will have negative titers even in
the presence of multi-system involvement. These tests are better used as confirmatory tests in
patients with a positive ANA
Polymyositis is characterized by proximal muscle weakness and elevated creatine kinase. Initial
remission can be induced with high-dose glucocorticoids (eg, prednisone), and most patients also
receive a glucocorticoid-sparing agent (eg, methotrexate, azathioprine) to minimize the adverse
effects of treatment
NSAIDs (indomethacin) can relieve pain associated with musculoskeletal conditions but would
not stop the progression of this patient's inflammatory myopathy. Colchicine is used for gout in
patients who have contraindications to NSAIDs
The USPSTF recommends a one-time screening for osteoporosis in all women age >65 year old
with dual-energy x-ray absorptiometry scan. Earlier screening can be considered for women with
additional risk factors such as calcium/vitamin D intake, smoking, corticosteroid use, lack of
weight-bearing exercise, low body mass index, and heavy alcohol use
This patient's presentation is most consistent with adhesive capsulitis (AC) (frozen shoulder
syndrome), characterized by markedly decreased range of motion (ROM). The glenohumeral
joint loses its normal distensibility due to chronic inflammation, fibrosis, and contracture of the
joint capsule. AC can be idiopathic or secondary to underlying conditions such as rotator cuff
tendinopathy (most common), subacromial bursitis, paralytic stroke, diabetes mellitus, or
humeral head fracture. Patients with AC typically report a gradual onset of shoulder stiffness,
with or without mild pain, that limits their ability to flex, abduct (eg, reach overhead), or rotate
the humerus. Diagnosis is confirmed on examination with >50% reduction in both passive and
active ROM.
Osteoarthritis of the glenohumeral joint can also cause a decrease in both active and passive
ROM. Symptoms are usually gradual in onset, and most patients have a history of significant
past injury to the shoulder.
Rotator cuff (RC) tears occur mainly in patients age > 40, usually after a fall on an outstretched
arm, and may present with pain on lifting or abduction. RC tears characteristically show defects
in active ROM, but passive ROM is preserved
RC tendinopathy can present similarly to adhesive capsulitis, and they may coexist. However,
patients with RC tendinopathy usually have more pain than stiffness. RC tendonitis may show
guarding with active ROM due to pain, but ROM is otherwise normal.
Rupture of the tendon of the long head of the biceps may occasionally be seen associated with
overuse in older patients. It typically causes sudden onset of pain, often with an audible pop and
visible bulge. Patients may report weakness at the shoulder, but passive ROM is normal.
It is difficult to distinguish gonococcal septic arthritis from reactive arthritis because synovial
fluid cultures can be negative in 50% of patients with gonococcal septic arthritis. However, this
patient is afebrile and has other symptoms suggestive of reactive arthritis, such as mouth ulcers,
enthesitis and low back pain
Colchicine is an anti-inflammatory drug used for the treatment of acute gout. It acts by
decreasing leukocyte activity primarily by inhibiting tubulin polymerization
Alkaline phosphatase is highest in bone and hepatobiliary tissues thus userful marker for
cholestatic liver disease and diseases of bone causing increased bone turnover. The MCC of an
isolated, asymptomatic elevation of alkaline phosphatase in an elderly is Paget disease of bone
(osteitis deformans). It is frequently discovered incidentally on routine blood tests, and alkaline
phosphatase can be elevated often >10 times the upper limit of normal. Paget’s disease is caused
by osteoclast dysfunction, with defective osteoid formation, increased bone remodeling, and
focal hypertrophy. The most commonly affected bones are the skull, clavicles, pelvis, and long
bones. Laboratory markers for bone turnover (eg, procollagen type I N propeptide, urine
hydroxyproline) are frequently elevated. X-rays of affected bones will show osteolytic or mixed
lytic-sclerotic lesions, and radionuclide bone scan is useful to fully stage the disease.
Statins (eg, atorvastatin) can cause minor elevations in liver markers, but this typically manifests
with a hepatocellular pattern (predominant elevations in transaminases) rather than a cholestatic
pattern (predominant elevation in alkaline phosphatase).
Prostate cancer with bone metastasis may cause an elevation of alkaline phosphatase but will
usually cause additional features such as bone pain or systemic symptoms (eg, weight loss,
fatigue). Calcium is often elevated, and prostate-specific antigen will usually be >4 ng/mL
The diagnosis of Sjogren syndrome requires evidence of dry mouth and eyes (eg, positive
Schirmer test result for decreased lacrimation) with either histologic evidence of lymphocytic
infiltration of the salivary glands or serum autoantibodies against SSA (Ro) and/or SSB (La).
Biopsy of salivary glands can confirm the diagnosis of SS if other evaluations are nondiagnostic.
However, samples are taken from the minor glands of the lip rather than submandibular glands.
Chronic anterior knee pain that is worse with climbing stairs is typical feature of patellofemoral
syndrome (PFS). Risk factors include overuse, malalignment, and trauma, although overuse
during training is probably most common. The patellofemoral compression test (pain elicited by
extending the knee while compressing the patella) and reproduction of pain with squatting are
highly suggestive. X-ray of the knee and MRI are usually normal and are generally needed only
if the diagnosis is in doubt or the patient fails to improve as expected. Management of PFS is
primarily mechanical exercises to stretch and strengthen the thigh muscles and avoiding
activities that aggravate the pain. NSAIDs are often prescribed but not helpful. Persistence is
required on the part of the patient as resolution of symptoms may often take weeks to months.
Tenderness at the tibial tubercle is a typical finding for Osgood-Schlatter disease. This condition
is caused by chronic/repetitive strain at the insertion of the patellar tendon on the tibial tubercle
and is seen in pre-adolescent and adolescent patients undergoing rapid growth spurts. Like PFS,
Osgood-Schlatter disease can cause anterior knee pain that is worse with squatting; however, it
would not be seen in adult patients.
This patient has acute lumbosacral radiculopathy (sciatica). Patients with uncomplicated lumbar
strain can have radiation of pain to the buttocks or posterior thigh, but radiation to the calf and
foot is more consistent with sciatica. Traction on the nerve root during the straight leg raise test
causes worsening or reproduction of pain (Lifting the left leg to 70 degrees with the knee held
straight causes burning pain radiating from the low back to the left foot). Most patients with
acute sciatica will experience spontaneous resolution; therefore, initial management is primarily
focused on acute relief of symptoms. NSAIDs and acetaminophen are the preferred first-line
drugs. Short-term use of opioids or muscle relaxants can be considered in patients with persistent
pain but is associated with significant sedation. Activity modification is often advisable, but
patients should be encouraged to maintain moderate physical activity.
MRI can confirm disc herniation or other causes of sciatica but would not change initial mgt.
Indications for MRI include progressive sensory or motor deficits, signs of cauda equina
syndrome (eg, saddle anesthesia), or concern for epidural abscess (eg, fever, IV drug abuse)
Osteoarthritis (OA) causes chronic pain, stiffness, and bony enlargement of the large weight-
bearing joints of the lower extremity, the small peripheral joints in the hands, and the cervical
and lumbar spine. Prominent osteophytes at the DIP (Heberden nodes) and PIP (Bouchard nodes)
joints are characteristic of OA. Rheumatoid arthritis most frequently involves the MCP joints
Charcot joint is due to decreased sensation in the lower extremity; patients unknowingly
traumatize the weightbearing joints (eg, ankle), leading to joint degeneration, deformation, and
functional limitation. Charcot joint is common in patients with diabetic neuropathy, and is
frequently seen in association with foot ulcers
Long-term cyclophasphamide use is A/W increased incidence of acute hemorrhagic cystitis and
bladder carcinoma. Hemorrhagic cystitis and bladder cancer are caused by acrolein, a toxic
metabolite of cyclophosphamide. Drinking plenty of fluids, voiding frequently, and taking
MESNA are all helpful in preventing these complications
The symptoms of lumbar spinal stenosis are posture-dependent. SS is caused by narrowing of the
spinal canal, leading to compression of one or more spinal roots. It is primarily seen in
degenerative arthritis with osteophyte formation affecting the facet joints (spondylosis).
However, other factors may contribute, including hypertrophy of the ligamentum flavum,
bulging of the intervertebral discs, and spondylolisthesis (displacement of one vertebral body
relative to another). Extension of the lumbar spine (eg, standing, walking upright) further
narrows the spinal canal and worsens the symptoms, whereas lumbar flexion (eg, walking uphill,
leaning on a cane) relieves the pain. The onset of pain with walking is referred to as "neurogenic
claudication" as it may resemble symptoms seen in vascular claudication. However, vascular
claudication causes pain with exertion and relief with rest, whereas neurogenic claudication is
relieved by walking while leaning forward ("shopping cart sign" ), and exercise with the spine
flexed (eg, cycling) does not incite symptoms. The diagnosis of SS can be confirmed on MRI of
the spine. Most patients are treated conservatively with physical therapy and exercise, although
some require surgical intervention.
Lumbar disk herniation causes acute back pain with unilateral radiation down to the foot
(sciatica). It usually follows an inciting event, and lumbar flexion makes the pain worse.
Vertebral metastasis presents as dull, non-radiating pain that is worse at night and not related to
position or activity. Patients often have a known malignancy or systemic symptoms (weight loss)
This patient has the classic triad of polyarthralgia, tenosynovitis, and painless vesiculopustular
skin lesions for disseminated gonococcal infection. Her complaints of wrist, elbow and ankle
pain are consistent with gonococcal polyarthralgias, and her pain elicited along the tendon
sheaths suggests tenosynovitis. A history of recent unprotected sex with a new partner is
frequently associated with disseminated gonococcal infection
Hydroxychloroquine is used for patients with active SLE. However, it can cause retinal toxicity
after 5-7 years. Patients should have a baseline ophthalmologic evaluation, with annual
reassessment beginning after 5 years
Spondyloarthritis is a type of arthritis that attacks the spine and, in some people, the joints of the
arms and legs. It can also involve the skin, intestines and eyes. Spondyloarthritis is an umbrella
term for inflammatory diseases that involve both the joints and the entheses (the sites where the
ligaments and tendons attach to the bones)
This patient likely has inflammatory bowel disease (IBD) complicated by spondylarthritis. Both
Crohn disease and ulcerative colitis have multiple extraintestinal manifestations, including
arthritis, eye (eg, uveitis, episcleritis) and skin (eg, pyoderma gangrenosum) involvement, and
hepatobiliary disease (eg, primary sclerosing cholangitis). Arthritis occurs in up to 45% of
patients with IBD and can involve axial spine or knee joints. Patients with spondylarthritis or
sacroiliitis have prolonged stiffness and low back/buttock pain that improves with activity. There
are no specific tests for the diagnosis of IBD-associated arthritis. NSAIDs can relieve arthritis
symptoms but exacerbate the underlying bowel disease. Consequently, although NSAIDs are
frequently tried, many of the medications (eg, sulfasalazine) used to treat the bowel disease itself
are also used to treat joint disease.
Reactive arthritis may follow infectious diarrhea caused by Shigella, Salmonella, Yersinia,
Campylobacter, or Clostridium difficile. It typically occurs within 2-3 weeks of the onset of
diarrhea and is often characterized by concomitant urethritis, conjunctivitis/uveitis, malaise, and
cutaneous findings (eg, keratoderma blennorrhagica, balanitis circinata)
Parvovirus B19 is a nonenveloped, ssDNA virus that infects red cell precursors. Viral arthritis
due to parvovirus B19 is polyarticular and symmetric. It is self-limited and usually resolves
within 1-2 months. Clinical manifestations correlate with anti-parvovirus IgM levels, which may
be assayed for diagnostic purposes. Treatment is symptomatic, primarily with NSAID
Neisseria gonorrhoeae is the MCC of septic arthritis in young, sexually active patients and
present with asymmetric polyarthralgias (often associated with tenosynovitis and skin rash) or as
an isolated purulent mono- or polyarthritis. Tenosynovitis means wrist, ankles, fingers and knees.
Diagnosis may be confirmed by Gram stain of the synovial fluid, blood cultures, and genital/
pharyngeal mucosal nucleic acid amplification tests.
Paget disease is the MCC of an asymptomatic elevation in alkaline phosphatase in older patients
and is often discovered on routine lab testing. Urine hydroxyproline is elevated which is derived
from the breakdown of collagen. Urine calcium may be increased but serum calcium and
phosphorus are normal in the absence of other complicating factors (Prolonged immobility,
concurrent hyperparathyroidism).
This patient has ankylosing spondylitis (AS) with reduced range of motion in the spine. He also
has a murmur suggesting AR, a common finding in chronic AS. Although a ground-level fall
would be unlikely to cause a fracture in an otherwise healthy person, suspicion for vertebral
fracture should be much higher in a patient with AS. Patients with longstanding AS can develop
osteopenia/osteoporosis due to increased osteoclast activity in the setting of chronic
inflammation (mediated by TNF-a and IL-6). In addition, spinal rigidity in these patients can
increase the risk of vertebral fracture, which often results from minimal trauma. Associated
findings may include thoracic wedging and hyperkyphosis
Erythema nodosum (EN) is characterized by painful, subcutaneous nodules on the anterior lower
legs. Diseases associated with EN include streptococcal infection, sarcoidosis, TB, IBD,
coccidioidomycosis, and Behcet disease. The initial workup includes basic laboratory tests
(CBC, liver function, renal function), ASO antibodies, and TB skin testing. In addition, a chest x-
ray should be obtained to assess for findings consistent with sarcoidosis (eg, bilateral hilar
lymphadenopathy, reticular opacities) or with TB (unlikely in the absence of symptoms).
Patients with diffuse systemic sclerosis (SSc) frequently have extradermal organ involvement,
especially of the lungs such as interstitial fibrosis (interstitial lung disease)
Sarcoidosis affects the lungs in >90% of cases. In addition to noncaseating granulomas, patients
can develop hilar adenopathy, fibrosis, bronchiectasis, and pleural effusions. Skin manifestation
may include erythema nodosum
Posttraumatic shoulder pain with limited active abduction is most likely due to a rotator cuff tear.
It occurs in patients age >40, usually after a fall on an outstretched arm. Patients present with
pain at the lateral shoulder that is worsened with raising the arm overhead or with external
rotation and abduction. Examination shows the drop arm sign (unable to smoothly control
shoulder adduction) and weakness with external rotation (passive range of motion is normal).
Rotator cuff tendinopathy (tendinitis) may also present with pain on abduction, but tendinopathy
without tear does not cause weakness. X-ray of the shoulder in a rotator cuff tear may show
underlying calcific tendinitis but has low sensitivity for confirming the diagnosis of a tear. MRI
can visualize the soft tissues around the humeral head and can accurately diagnose a rotator cuff
tear. Treatment of an acute tear usually involves surgery, with best results if performed within 6
weeks of the injury (A 65-year-old man comes to the emergency department due to right
shoulder pain and weakness after falling on his outstretched hand. The pain is worse when he
tries to lift his arm above shoulder level or when he pulls or pushes with his right arm. He also
has pain when he lies on the affected shoulder. Examination shows limited, active abduction and
external rotation of the humerus).
This patient's presentation with fever and acute monoarticular arthritis warrants urgent
synovial fluid analysis (cell count, Gram stain, culture) to exclude septic arthritis. Although gout
can cause low-grade fever, the progressive nature of this patient’s symptoms over 3 days (gout
flares typically have an abrupt onset with maximal symptoms within 12-24 hours) increases the
likelihood of a septic joint, particularly in the setting of diabetes. Underlying joint disorders (eg,
gout, pseudogout, osteoarthritis) increase the risk for secondary joint infection. In patients with
crystal- induced arthritis (eg, gout), the presence of crystals alone does not rule out septic
arthritis as these can be present in synovial fluid between attacks. If the Gram stain is positive
and joint fluid white cell count is >50,000/mm', the patient should be started on empiric
antibiotics until culture results are known. If crystals are present, the fluid is nonpurulent, and
Gram stain is negative, the patient may be managed as for a standard gout flare
Spinal cord compression (eg, due to malignancy) presents acutely or subacutely with back pain,
paralysis, hyperreflexia, fecal incontinence, or urinary incontinence/retention. Given this
patient's 6-month history of positional back pain and his normal physical examination, his
symptoms are likely due to nerve root compression rather than spinal cord compression
Polymyalgia rheumatica (PMR) affects patients age >50 and is characterized by pain and
stiffness in the neck, shoulders, and pelvic girdle, along with an elevated ESR. The treatment of
choice for uncomplicated PMR is low-dose prednisone, which results in rapid relief of symptoms
De Quervain’s tenosynovitis is a condition that classically affects new mothers who hold their
infants with the thumb outstretched (abducted/extended). The abductor pollicis longus and
extensor pollicis brevis tendons are affected; passive stretch of these tendons elicits pain.
This patient has symmetric polyarticular arthritis with a brief, self-limited course. In light of her
exposure to young children, this is likely viral arthritis due to parvovirus B19. Viral arthritis is
often associated with joint swelling and tenderness. Children present with the characteristic
"slapped cheek" rash. A similar arthritis may also be seen with HIV, mumps, rubella, and other
viruses. The self-limited arthritis of parvovirus B19 may resemble early RA or SLE, and may be
associated with a weakly positive rheumatoid factor or ANA titer. However, RA and SLE are
significantly less common, typically produce more obvious synovitis, and usually follow a
chronic, protracted course. If there is doubt regarding the diagnosis, parvovirus B19 infection can
be confirmed with assay for anti-parvovirus IgM or NAAT
Crystalline arthritis (ie, gout, pseudogout) and purulent gonococcal arthritis cause acute,
inflammatory monoarthritis. Disseminated gonococcal infection can also present as chronic
polyarticular arthralgias but is asymmetric, often favors large joints, and is associated with
tenosynovitis and pustular skin lesions
Connective tissue thickening, myofibroblast proliferation, and increased collagen and ground
substance production, is the underlying pathogenesis of systemic sclerosis. Involvement of the
kidneys produces hypertension, and before the advent of ACE inhibitors, scleroderma renal crisis
was the MCC of death in these patients. Pulmonary arterial hypertension ultimately results in
right heart failure in patients. Esophageal and gastric dysmotility cause these patients to suffer
GERD. ANA autoantibodies are present in all patients with scleroderma; anti-topoisomerase-1
antibodies would most likely be present
RA is characterized by pain, prolonged morning stiffness, and doughy synovitis involving the
MCP joints and wrists. Synovial fluid will show an inflammatory effusion, and x-rays will reveal
periarticular osteopenia with erosions of the joint margin.
X-ray in septic arthritis will show a normal joint space with swelling of adjacent soft tissues.
Synovial fluid has a very high cell count with visible organisms on Gram stain.
Chronic knee pain and a bland synovial effusion (no organisms, <2,000 WBC) has typical
features of osteoarthritis (OA). Most patients will have exertional pain with few associated
symptoms, although a subset can develop swelling, rest pain, and brief morning stiffness. The
diagnosis of OA is based on clinical and x-ray findings. Crepitus on movement, periarticular
bony enlargement, and painful or decreased range of motion are characteristic. Although the
synovium can have signs of chronic inflammation on arthroscopic inspection or biopsy, joint
fluid cell counts will be significantly lower than in RA or crystal-induced arthritis. Diagnostic
findings on x-ray include a narrowed joint space, osteophytes, and subchondral sclerosis/cysts
This young patient has chronic back pain and stiffness that improve with activity (stretching),
warm showers, and nonsteroidal agents, suggestive of inflammatory back pain. In addition, she
has reduced spinal range of motion and an x-ray showing arthritis and erosions involving the
sacroiliac joints. Anterior uveitis (iritis) is the most common extraarticular manifestation of AS
characterized by inflammation of the uveal tract (iris, ciliary body, and choroid). Anterior uveitis
typically presents with intense pain and photophobia in one eye.
AS is associated with aortic valve disease, typically aortic regurgitation, due to scarring of the
valve cusps. AS can cause restrictive lung physiology due to impaired chest expansion and apical
pulmonary fibrosis. It does not cause obstructive lung disease.
Episcleritis is characterized by inflammation seen at the white of the eye, without involvement of
the uveal tract. It is most strongly associated with RA and inflammatory bowel disease
Ischemic optic neuropathy is a feared complication of temporal arteritis. It results from vasculitis
of the retinal artery and may cause blindness.
Oral ulcers are a/w a number of autoimmune disorders (eg, SLE, Behcet disease, Crohn disease)
Giant-cell arteritis (temporal arteritis) usually occurs in association with polymyalgia rheumatica
and presents with systemic symptoms, headache, jaw claudication, visual disturbances, and a
significantly elevated ESR
Polymyalgia rheumatica almost always occurs in patients age >50. Patients develop pain and
stiffness of the shoulders and pelvic girdle but do not have significant muscle tenderness. ESR is
usually significantly elevated. Polymyalgia rheumatica is associated with temporal arteritis.
Polyarteritis nodosa typically presents with systemic symptoms, skin findings (eg, livedo
reticularis, purpura), kidney disease, abdominal pain, and muscle aches or weakness. ESR is
usually elevated
Polymyositis usually presents with symmetrical proximal muscle weakness, mild pain, and
elevated muscle enzymes (eg, creatine kinase, AST, lactate dehydrogenase, aldolase)
Young African American woman with constitutional symptoms (fatigue), thrombocytopenia, MR
(holosystolic murmur to the apex), and a positive ANA, suggestive of SLE, now likely has a
cerebrovascular accident (sudden-onset unilateral weakness, negative CSF findings).
Antiphospholipid syndrome presents with a thrombotic event (DVT or arterial thrombus) or
pregnancy morbidity (fetal loss, severe preeclampsia, placental insufficiency) plus a positive
serology for 1 of 3 antiphospholipid antibodies: anticardiolipin antibody, anti-beta2-
glycoprotein-1 antibody, or lupus anticoagulant. The biggest risk factor for APS is SLE; A
minority of healthy individuals may develop transient antiphospholipid antibodies, so all positive
serology for APS should be repeated at 12 weeks to confirm diagnosis. Patients with APS often
require anticoagulation for life (heparin product or warfarin) as the risk of recurrent thrombus is
high. For those with concomitant SLE, hydroxychloroquine is also added
Multiple sclerosis (MS) presents with neurologic symptoms, but transverse myelitis causing a
paraplegia or paraparesis is much more common than hemiparesis. As MS is inflammatory,
symptoms tend to develop over hours or days, not suddenly like APS
Rheumatic fever causes chorea-like movements (Sydenham chorea). It commonly presents with
carditis and migratory arthritis.
A popliteal cyst is due to extrusion of synovial fluid from the knee joint into the gastrocnemius
or semimembranosus bursa through a communication between the joint and the bursa. Excessive
synovial fluid formation (eg, due to osteoarthritis or rheumatoid arthritis) and positive pressure in
the knee during extension can cause passage of fluid into the bursa and gradual enlargement of
the cyst. Popliteal cysts are often asymptomatic and present as a chronic, painless bulge behind
the knee. The diagnosis is usually apparent on examination, with a soft mass in the medial
popliteal space that is most noticeable with knee extension and less prominent with flexion.
Rupture of a popliteal cyst (eg, following strenuous exercise) can cause posterior knee and calf
pain, with tenderness and swelling of the calf resembling DVT. An arc of ecchymosis is often
visible distal to the medial malleolus ("crescent sign”). Ultrasound can rule out DVT and confirm
the popliteal cyst.
A popliteal (Baker) cyst is due to extrusion of synovial fluid from the knee joint into the
gastrocnemius or semimembranosus bursa, and is most common in patients with underlying
arthritis. Popliteal cysts may present as a painless bulge in the popliteal space, but cyst rupture
can cause acute pain in the calf.
Stress fractures of the tibia present with progressive pain at the shin and focal tenderness over the
fracture site.
Tears of the medial collateral ligament are caused acutely by a twisting injury or blow to the
lateral knee. Examination findings include tenderness at the medial joint line and valgus laxity
Tears of the medial meniscus result from a twisting force on a fixed foot. Patients report a
popping sound followed by acute pain. Examination shows a small effusion and crepitus,
locking, or catching with range of motion
Needle-shaped crystals with negative birefringence are diagnostic of gout. Gout can also cause
renal stones (uric acid) and an acute painful monoarthritis of the knee, but is more common in
the first MTP joints and is not associated with hypercalcemia
Hemorrhagic joint fluid is usually seen in trauma (eg, anterior cruciate ligament tear), bleeding
disorders (eg, hemophilia), or anticoagulant use. Atraumatic hemarthrosis is uncommon
This patient with poorly localized anterior knee pain has typical features of patellofemoral pain
syndrome (PFPS). PFPS is one of the MCC of knee pain in young athletes and predominantly
affects women. It is usually triggered by chronic overuse or malalignment (eg, angular
deformities, weakness of hip abductors) but can also be seen acutely following trauma. The
diagnosis is primarily based on clinical findings with pain localized to the anterior knee, usually
described as an aching sensation, and worsened by activities such as climbing up or down stairs.
The patellofemoral compression test (reproduction of pain when the patella is compressed into
the trochlear groove) is often helpful, although no single finding has high sensitivity or
specificity. The initial management of PFPS includes reduced intensity of exercise (especially
running), activity modification, and NSAID. Patients also should be counseled on stretching and
strengthening exercises, with an emphasis on the quadriceps, knee extensors, and hip abductors.
This patient is most likely suffering from cervical spondylosis. It is estimated that cervical
spondylosis affects 10% of people older than 50 years of age. The history of chronic neck pain is
typical. Limited neck rotation and lateral bending is due to osteoarthritis and secondary muscle
spasm. Sensory deficit is due to osteophyte-induced radiculopathy and isolated sensory
abnormalities are associated with good prognosis. Typical radiographic findings include bony
spurs and sclerotic facet joints. Interestingly, such 'osteoarthritic' changes are common in
asymptomatic patients older than 50 years of age; therefore, specificity of these findings are low.
Other findings during cervical spondylosis may include narrowing of the disk spaces and
hypertrophic vertebral bodies
RA does not affect the thoracic, lumbar, sacral spine or sacroiliac joints. The thoracic spine has
great stability and is rarely affected by arthritis, disc herniation, or other spinal disorders. The
lumbar spine is the most common site of disc herniation and spinal stenosis. The sacroiliac joints
are commonly affected in seronegative spondyloarthropathies (eg, ankylosing spondylitis,
reactive arthritis, and psoriatic arthritis)
The carpal tunnel is an anatomic space in the wrist defined by the carpal bones and the transverse
carpal ligament. It contains the median nerve along with the tendons of the flexor digitorum
profundus, flexor digitorum superficialis, and flexor pollicis longus. Compression of the median
is characterized by pain and paresthesias in the first 3 digits and the radial half of the fourth.
These symptoms are often worse at night. Motor involvement in severe cases can cause
weakness of thumb abduction and opposition, and atrophy of the thenar eminence. This patient
has sensory deficits in the distribution of the median nerve and multiple risk factors for CTS
(female sex, obesity, and hypothyroidism). During examination, tapping over the median nerve at
the wrist (Tinel sign) or holding the wrists in extreme flexion with the dorsum of the hands
pressed together (Phalen test) can often reproduce the symptoms. However, if the diagnosis is
uncertain (or severe symptoms prompt consideration for surgery), CTS can be confirmed with
nerve conduction studies, which will show slowing in the median nerve at the wrist
The nerves in the body control how the muscles work by sending electrical impulses. An
electromyogram (EMG) is a test that measures the electrical activity of muscles both at rest and
during contraction. Nerve conduction velocity studies (NCVs) measure how fast the nerves can
send electrical signals. An EMG is performed by inserting small needles into the muscles of the
arm, leg or back to examine the electrical activity caused by a muscle contraction. When the
electrodes are in place, the electrical activity in that muscle is recorded while the muscle is at
rest. Then the doctor asks you to contract the muscle slowly and steadily. This electrical activity
is recorded. The electrical activity in the muscle shows as wavy lines on a monitor and also heard
as popping noises when the muscle is contracted. The electrode may be moved a number of times
to record the activity in different areas of the muscle or in different muscles.
Nerve Conduction Studies: In this test, several electrodes are attached to your skin with tape or a
paste. A shock-emitting electrode is placed directly over the nerve, and a recording electrode is
placed over the muscles controlled by that nerve. Several quick electrical pulses are given to the
nerve, and the time it takes for the muscle to contract in response to the electrical pulse is
recorded. The speed of the response is called the conduction velocity. The same nerves on the
other side of the body may be studied for comparison. Nerve conduction studies are done before
an EMG if both tests are being done. Nerve conduction/EMG tests may take from 15 minutes to
1 hour or more, depending on how many nerves and muscles are studies.
MRI of the cervical spine is indicated to evaluate suspected cervical radiculopathy (neck and
upper arm pain, diminished reflexes) or cervical spondylotic myelopathy (bilateral upper motor
neuron weakness, bilateral sensory deficits).
X-ray can confirm the diagnosis of osteoarthritis, which would cause joint pain that is worse with
activity and better with rest. Decreased range of motion is common, Heberden and Bouchard
nodes would often be seen, and focal paresthesias would be atypical.
Low back pain (LBP) is most commonly due to mechanical causes (eg, muscle strain,
ligamentous sprain). Mechanical LBP usually improves with rest, and most episodes resolve over
2-4 weeks with symptomatic treatment. Red flags indicating more serious causes (eg, infection,
malignancy) include age >50, neurologic signs (eg, weakness, urine retention/incontinence,
saddle anesthesia), history of malignancy, weight loss, worsening pain at night, and systemic
symptoms (eg, fever). This patient has chronic LBP that is worse at night but improves with
physical activity rather than rest. In a young patient, this pattern suggests an inflammatory
spondylarthritis such as ankylosing spondylitis (AS), psoriatic arthritis, reactive arthritis, or
arthritis associated with inflammatory bowel disease. The spondyloarthropathies are immune-
mediated disorders that are most common in patients age <40 and affect the sites of ligamentous
insertion (enthesitis), leading to gradual onset of LBP and progressive stiffness. AS eventually
leads to destruction of the articular cartilage, especially at the sacroiliac joints and apophyseal
joints of the spine
Features of low back pain suggesting an inflammatory cause include gradual onset of pain, onset
at age <40, pain at night that does not improve with rest, and improvement with activity or
exercise
Abnormal bone mineralization (osteomalacia) can lead to pathologic bone fractures, with back
pain that is worse with activity or positional changes
Ligamentous sprains can often be traced to a specific event or action. Patients typically
experience more pain with movement and improvement with rest. Symptoms generally resolve
within days to weeks.
Lumbar osteoarthritis often develops in the setting of advanced age and disk degeneration; the
pain typically worsens with activity and is relieved with rest
Nerve root demyelination can be due to immune-mediated (eg, Guillain-Barre), toxic, metabolic,
hereditary, or infectious processes
This patient's presentation suggests an interdigital (Morton) neuroma, which commonly occurs in
runners. The disorder is not a true neuroma but is a mechanically induced neuropathic
degeneration of the interdigital nerves that causes numbness, aching, and burning in the distal
forefoot from the metatarsal heads to the third and fourth toes. The symptoms are worsened by
walking on hard surfaces and wearing tight or high-heeled shoes. The diagnosis of Morton
neuroma is primarily made clinically. On examination, squeezing the metatarsal joints will cause
pain on the plantar surface of the foot along with crepitus between the third and fourth toes
(Mulder sign). Treatment involves metatarsal support with a bar or padded shoe inserts to
decrease pressure on the metatarsal heads. Surgery is usually reserved for patients who fail
conservative treatment
Stress fractures occur due to a sudden increase in exertion without adequate rest that eventually
breaks the bone. Patients have sharp, localized pain and tenderness over a bony surface. The
female athlete triad (oligomenorrhea, decreased caloric intake, and osteopenia/osteoporosis) is
associated with stress fracture
Plantar fasciitis causes focal pain in the plantar area of the rearfoot. It usually worsens with the
first steps in the morning, decreases with activity during the day, and often worsens again later in
the day with prolonged weight bearing. Examination will show point tenderness at the plantar
surface of the heel.
Tarsal tunnel syndrome is due to compression of the tibial nerve as it passes through the ankle
caused by a fracture of the bones around the ankle. Patients have burning, numbness, and aching
of the distal plantar surface of the foot or toes that sometimes radiate up to the calf.
This patient with medial knee pain and focal tenderness has pes anserinus pain syndrome
(PAPS). This condition is often referred to as anserine bursitis, but most patients do not have true
inflammation in the bursa, and multiple regional structures can contribute to the pain. The pes
anserinus is formed by the conjoined tendons of gracilis, sartorius. and semitendinosus. The
anserine bursa is located anteromedially over the tibial plateau, just below the joint line of the
knee and deep to the pes anserinus. PAPS can be caused by an abnormal gait, overuse, or trauma.
Localized pain is typical over the anteromedial tibia and is often exacerbated by pressure from
the opposite knee while lying on the side. Examination shows a well-defined area of tenderness
over the medial tibial plateau just below the joint line. A valgus stress test does not aggravate the
pain, indicating no medial collateral ligament involvement. The diagnosis is primarily based on
clinical features, although x-ray can exclude concurrent osteoarthritis of the knee.
Medial compartment osteoarthritis is most common in patients age > 40, and presents with pain
and stiffness of the knee joint. Findings include crepitus and bony tenderness at the medial joint
line. X-ray of the knee will reveal narrowing of the joint space and osteophyte formation.
Patellofemoral syndrome causes anterior knee pain and is most common in women. Patients
present with peripatellar pain worsened by activity or prolonged sitting (due to sustained flexion)
and may also have crepitus with motion of the patella
Prepatellar bursitis presents with pain and swelling directly over the patella, usually following
trauma. Examination shows cystic swelling over the patella with variable signs of inflammation.
This patient with slowly progressive pain relieved by rest has degenerative joint disease of the
hip. Osteoarthritis is characterized by progressive destruction of the articular cartilage.
Predisposing factors include advanced age, obesity, and diabetes mellitus. Osteoarthritis typically
affects the interphalangeal joints of the hands and the large weight-bearing joints of the lower
extremity. The pain of hip osteoarthritis is typically felt in the groin, buttock, or pelvis and can
radiate to the lower thigh or knee. Patients may have mild pain and stiffness with prolonged rest,
but the worst pain usually occurs with activity and weight bearing. Examination findings include
decreased internal and external rotation, but synovitis (redness, warmth, tenderness) is absent. X-
ray findings include loss of joint space, osteophyte formation, and subchondral sclerosis.
Osteonecrosis is caused by occlusion of arteries supplying the femoral head, leading to collapse
of the periarticular bone. It can present with groin pain that is worse with activity, but rest and
night pain are often present. Patients usually have decreased range of motion, and an underlying
disorder that disrupts flow in the microcirculation (eg, sickle cell disease, glucocorticoid use).
Trochanteric bursitis is caused by friction of the tendons of the gluteus medius and tensor fascia
lata over the greater trochanter of the femur. Pain is localized to the lateral hip and is worsened
by direct pressure
Impingement of the lumbar nerve roots can cause referred pain to the posterior hip, thigh, and
lower leg. Hip mobility is normal, but patients may have exacerbation of pain on flexion of the
hip with the knee extended (straight leg raise maneuver)
This patient's laboratory results reveal macrocytic anemia (Hb < 12 g/dl and MCV > 100), which
is one of the adverse effects of methotrexate. Other side effects of methotrexate include: nausea,
stomatitis, rash, hepatotoxicity, interstitial lung disease, alopecia and fever. MTX is a DMARD
that inhibits dihydrofolate reductase. It interferes with the cellular utilization of folic acid, and
folate depletion is the cause of most of these complaints which can be alleviated by the addition
of supplemental folic acid, without changing the efficacy of MTX. The more serious
complication is development of pancytopenia. Routine peripheral blood counts are recommended
every three months. Hydroxychloroquine is also a DMARD. Common adverse effects are GI
distress, visual disturbances, and hemolysis in G6PD deficiency
Prednisone is a glucocorticoid that is often used in RA. Prednisone has many side effects such as
iatrogenic Cushing's syndrome, osteoporosis, adrenocortical atrophy, and poor wound healing.
Azathioprine is associated with pancreatitis, liver toxicity, and dose dependent bone marrow
suppression
This patient has features suggesting osteoarthritis (OA), including chronic joint pain,
periarticular bony enlargement, and a bland synovial effusion (ie, low synovial leukocyte count,
no crystals). Risk factors for OA include advancing age, obesity, diabetes, and prior joint injury.
The diagnosis can be confirmed with x-ray, revealing narrowed joint space, osteophytes, and
possible subchondral sclerosis. The initial management emphasizes nonpharmacologic measures
including exercise, weight loss, and activity modification. NSAIDs (eg, diclofenac) are the DOC
for relief of pain but do not alter the progression of the disease.
Patients with OA who fail conservative measures can be considered for an intra-articular
glucocorticoid injection. Oral glucocorticoids (eg, prednisone) are used in rheumatoid arthritis
and other inflammatory arthropathies but not in OA
Acetaminophen use for OA is less effective than NSAIDs, but can be considered for patients
with contraindications to NSAIDs (eg, peptic ulcer, coronary artery disease). Opioids (eg,
oxycodone) are also less effective and not recommended for routine use in OA
Colchicine is used in the prevention and treatment of acute gout flares. Febuxostat is a xanthine
oxidase inhibitor used in management of chronic gout. Gout is characterized by intermittent
inflammatory arthritis. Synovial fluid will show many leukocytes (eg, 2,000-10,000/mm3) as well
as needle-shaped, negatively birefringent crystals
Gastroenterology
This patient's presentation (hematemesis, melena, anemia, and a BUN/Cr >20) is consistent with
upper GI bleeding (UGIB), likely due to NSAID use. Supportive measures for UGIB include
supplemental oxygen, bowel rest, and IV fluids through large-bore catheters. An IV PPI should
also be administered for acid suppression. Packed RBC transfusion can increase oxygen-carrying
capacity in patients with significantly low Hb and is recommended in acute GI bleeding for
patients with Hb < 7 g/dL. A higher threshold of Hb < 9 g/dL is considered for unstable patients
with acute coronary syndrome or with active bleeding and hypovolemia. FPP contains all
clotting factors and plasma proteins from one unit of blood. It is usually indicated for severe
coagulopathy (eg, liver disease, DIC) with active bleeding. FPP is generally not needed to correct
a minimally abnormal INR (<1.6), which is a common finding in GI bleeding.
PPI reduce rebleeding and the need for transfusions, and help stabilize clots in patients with
UGIB. Histamine-2 blockers, such as famotidine, have not demonstrated such benefits and
consequently are not recommended as first-line therapy for UGIB.
Somatostatin analogs such as octreotide are a mainstay in management of variceal bleeding. This
patient has relatively limited alcohol use and no manifestations of cirrhosis, making variceal
bleeding less likely
Platelet transfusions are typically given for a platelet count <10,000 (increased risk of
spontaneous hemorrhage) or for a platelet count < 50,000 with active bleeding
Whole blood transfusion, which includes PRBCs in addition to plasma, may be used in patients
with severe hemorrhage (eg, major trauma) requiring massive blood transfusions to assist in
volume expansion.
This patient's symptoms (spontaneous pain, odynophagia for cold and hot food) and resolution of
chest pain after taking nitroglycerin is consistent with diffuse esophageal spasm. Nitrates and
CCB relax not only myocytes in coronary vessels, but also those in the esophagus, thereby
alleviating the pain. Esophagography (barium swallow) may or may not show other anomalies
(nodules or plaques, ulcers, strictures, and rings). Esophageal manometry (measures the strength
and muscle coordination of your esophagus when you swallow) should show repetitive, non-
peristaltic, high-amplitude contractions, either spontaneously or after ergonovine stimulation.
Episodes of dysphagia, regurgitation, and/or chest pain precipitated by emotional stress should
raise the suspicion for esophageal motility disorder such as diffuse esophageal spasm.
Manometry establishes the diagnosis of diffuse esophageal spasm.
Routine colon cancer screening should be done in all patients age >50 using high-sensitivity
fecal occult blood testing (FOBT) annually, flexible sigmoidoscopy every 5 years combined with
FOBT every 3 years, or colonoscopy every 10 years. Patients with a history of colon cancer in a
first-degree relative should be screened at age 40 or 10 years before the age of the relative's
diagnosis. This patient's father was diagnosed with colon cancer at age 50, and he should begin
screening at age 40.
This patient's recent binge drinking, severe epigastric pain radiating to his back and leukocytosis
suggest acute pancreatitis. Diagnosis requires 2 of the following criteria: acute onset of severe
epigastric pain radiating to the back, increased amylase or lipase >3 times the upper limit of
normal, and characteristic abdominal imaging findings (eg, focal or diffuse pancreatic
enlargement). Early in pancreatitis, the pancreas synthesizes digestive enzymes but cannot
secrete them. These enzymes leak out of the acinar cells into the systemic circulation. Amylase
rises within 6-12 hours of symptom onset and may remain elevated for 3-5 days. Lipase rises
within 4-8 hours of symptom onset but remains elevated (8-14 days). Imaging is not required for
diagnosis in patients with typical abdominal pain and significantly elevated amylase and/or
lipase. Contrast-enhanced CT scan of the abdomen may be performed in patients with unclear
diagnosis or in those who fail to improve with conservative management (to identify infection or
necrosis). This patient has characteristic abdominal pain and would first need laboratory testing.
If the diagnosis is still unclear, he may require imaging such as abdominal CT scan
Uncomplicated patients with acute onset of severe epigastric pain radiating to the back and
increased amylase or lipase (>3 times normal) do not need confirmatory imaging for diagnosing
acute pancreatitis. Contrast-enhanced CT scan of the abdomen may be performed in patients with
unclear diagnosis or those who fail to improve with conservative management. Abdominal
ultrasound may identify gallstones as the cause of pancreatitis
Abdominal ultrasound may reveal a classic diffusely enlarged and hypoechoic pancreas in acute
pancreatitis. However, nearly 30% of patients have an ileus with bowel gas that prevents full
visualization of the pancreas and may not clearly identify inflammation or necrosis. Once the
diagnosis of pancreatitis is confirmed, ultrasound may be useful to identify gallstones
ERCP is not indicated for the routine diagnosis of acute pancreatitis. However, early ERCP for
suspected biliary pancreatitis may decrease morbidity and mortality. ERCP is also effective for
evaluating patients with recurrent pancreatitis or draining pancreatic pseudocysts.
This patient with weeks of lower abdominal pain, bloody diarrhea, and fecal urgency likely has
undiagnosed IBD (UC). His acute worsening with fever, abdominal distention, leukocytosis,
hypotension, and tachycardia suggests toxic megacolon. Other manifestations of toxic
megacolon include altered mental status, peritonitis, and electrolyte abnormalities. Toxic
megacolon may be the initial presentation of IBD and is diagnosed by abdominal Xray showing
colonic distention. Treated initially with conservative management which includes bowel rest,
nasogastric suction, and either corticosteroids with broad spectrum-antibiotics (due to IBD) or
antibiotics targeted at Clostridium difficile. Opioids are inadvisable because their antimotility
effects can promote colonic perforation. Other antimotility agents (eg, loperamide) and
anticholinergic agents should also be discontinued. Toxic megacolon should be confirmed before
treatment is initiated; IV corticosteroids are preferred. Sulfasalazine for IBD should be started
only once acute symptoms have resolved.
This patient with burning localized pain and regional hyperesthesia/allodynia (feel pain from
stimuli that don't normally cause pain) in the context of recent cancer treatment has common
features of herpes zoster (shingles). Pain from shingles may precede the onset of the classic
vesicular rash by several days. In some cases, patients may develop persistent hypersensitivity of
afferent pain fibers leading to chronic pain known as post-herpetic neuralgia. Treatment with
antiviral medications (acyclovir, valacyclovir, or famciclovir) in the first few days of a shingles
outbreak can shorten the duration of symptoms and decrease the risk of post-herpetic neuralgia
Black stools (melena) are a typical symptom of upper GI hemorrhage (above the ligament of
Treitz). Peptic ulcer would be the most likely explanation
Small-bowel obstruction is a common cause of abdominal pain, but it most often occurs in
patients with adhesions from prior abdominal surgery. It is usually associated with nausea and
abnormal bowel sounds
This patient's presentation is consistent with colovesical fistula (connection between the colon
and bladder), which can be a complication of acute diverticulitis. The mechanism is usually due
to direct extension of a ruptured diverticulum or erosion of a diverticular abscess into the
bladder. Patients typically develop fecaluria (stool in the urine) or pneumaturia (air in the urine)
that usually occurs at the end of urination as the gas collects at the top of the bladder. Patients
can also develop recurrent UTI (sometimes due to mixed flora with coliform organisms).
Colovesical fistula can also occur in patients with Crohn disease or malignancy (usually of the
colon). Abdominal CT scan with oral or rectal (not intravenous) contrast can confirm the
diagnosis by showing contrast material in the bladder with thickened colonic and vesicular walls.
Colonoscopy is usually recommended in patients diagnosed with colovesical fistula to exclude
colonic malignancy. Treatment is typically surgical after resolution of the infection.
Acute pancreatitis presents with sudden onset of persistent, severe epigastric pain radiating to
back that is partially relieved by sitting up and leaning forward, nausea, and vomiting. Gallstones
and chronic alcohol abuse account for about 75% of all cases of acute pancreatitis. Nearly one-
third of patients have chest x-ray abnormalities, including pleural effusions, atelectasis, elevated
hemidiaphragm, or pulmonary infiltrates. These complications are often due to activated
pancreatic enzymes (eg, phospholipase, trypsin) and cytokines (TNF) that are released from the
inflamed pancreas into the circulation and cause inflammation in other parts of the body. Other
complications include ileus, ARDS and renal failure.
Acute cholecystitis (inflammation of gallbladder) typically presents as RUQ pain that radiates to
the right shoulder. It is often accompanied by nausea, vomiting, fever, leukocytosis and positive
Murphy’s sign
Mesenteric ischemia presents with severe acute periumbilical abdominal pain that is out of
proportion to exam findings. Risk factors include advanced age, diffuse atherosclerosis, valvular
abnormality, cardiac arrhythmias, or recent myocardial infarction
Patients with perforated peptic ulcer usually present with the sudden onset of epigastric pain,
nausea, vomiting, hematemesis, and peritoneal signs (eg, guarding, rigidity, rebound tenderness)
on physical examination. Chest x-ray shows free air under the diaphragm
Pedal edema, ascites (shifting dullness), bilateral gynecomastia, and spider angiomata suggests
cirrhosis and portal HTN. Other clinical features include palmar erythema, caput medusa, and
splenomegaly. The MCC of cirrhosis in the United States include viral hepatitis (C more than B),
chronic alcoholism, NAFLD, and hemochromatosis. The initial evaluation should consist of
medical history, including inquiring about medications, social habits (eg, alcohol use, drug use,
high-risk sexual activity), and family history (eg, to exclude hemochromatosis). Given this
patient's history of IV drug use, the most likely cause of his cirrhosis is an infection with viral
hepatitis (which, along with alcohol use, accounts for >50% of cases of cirrhosis). Laboratory
studies, including viral hepatitis serologies and iron studies, should be obtained.
This patient's epigastric abdominal discomfort and melena are likely due to PUD. PUD refers to
ulcerations in the stomach or duodenum that are most commonly caused by H. pylori infection or
NSAID. Patients may have epigastric pain, nausea, and/or early satiety in association with food.
The classic symptoms of duodenal ulcer occur in the absence of a food buffer and can include
epigastric pain 2-5 hours after meals, on an empty stomach, or at night. Patients can have melena
due to processed blood from upper GI bleeding (proximal to the ligament of Treitz). PUD is
diagnosed with upper Gl endoscopy.
Colon cancer and diverticulosis are possible etiologies of lower Gl bleeding that typically present
with hematochezia as opposed to melena. Colon cancer may also result in changes in bowel
habits (eg, constipation, changes in stool caliber).
UC typically causes hematochezia as opposed to melena. Melena can occur with Crohn’s disease
involving the upper Gl tract or small bowel. Nonetheless, IBD is typically accompanied by
diarrhea and systemic symptoms (eg, fatigue, fever, weight loss)
Atherosclerotic vascular disease (eg, carotid stenosis) is a risk factor for chronic ischemic colitis
(reduced blood flow to large intestine) and mesenteric ischemia (reduced blood flow to small
intestine). However, ischemic colitis is typically associated with hematochezia as opposed to
melena. In addition, patients with mesenteric ischemia report pain that is exacerbated by eating,
which often leads to food aversion and weight loss as opposed to weight gain.
This patient with fever, jaundice, and RUQ pain (Charcot triad) most likely has acute cholangitis
(AC). Confusion and hypotension (Reynolds pentad) are also sometimes seen in severe AC. If
not treated promptly, AC can lead to septic shock. Laboratory results usually show leukocytosis
and neutrophilia in addition to elevations in alkaline phosphatase, GGT, and direct bilirubin.
Biliary stasis predisposes to AC, and the MCC is due to bile duct obstruction from gallstones,
malignancy, or stenosis. In the setting of stasis, the bile-blood barrier can be disrupted, allowing
bacteria and toxins from the hepatobiliary system to translocate into the blood stream.
Ultrasound or CT confirms the diagnosis and most frequently shows common bile duct dilation.
Supportive care, broad-spectrum antibiotics, and biliary drainage, preferably by ERCP with
sphincterotomy, are the mainstays of treatment. Other options for biliary decompression include
percutaneous transhepatic cholangiography and open surgical decompression
Primary sclerosing cholangitis is a chronic disease. Although it has a cholestatic pattern on liver
tests, patients are frequently asymptomatic or have chronic fatigue and pruritus on presentation.
Acute hypotensive illness is not typical
This patient's clinical presentation is concerning for acute diverticulitis (inflammation due to
microperforation of a diverticulum). Chronic constipation and a low-fiber, high-fat diet are risk
factors for diverticular disease. Patients with diverticulitis present with LLQ pain, fever, nausea/
vomiting, and leukocytosis. Abdominal CT scan (oral and IV contrast) is the best diagnostic test
showing increased inflammation in pericolic fat, presence of diverticula, bowel wall thickening,
soft tissue masses (eg, phlegmons), and pericolic fluid collections suggesting abscess.
Upright abdominal film can reveal nonspecific findings sometimes associated with acute
diverticulitis (eg, dilated small/large bowel with air-fluid levels due to ileus or obstruction, soft
tissue densities due to abscess). It can also occasionally reveal air under the diaphragm or in the
retroperitoneum (perforation). However, abdominal CT is more sensitive (~94%) as well as
specific ( ~ 99%) for diagnosing diverticulitis and differentiating it from other causes of
abdominal pain.
Barium contrast enema can detect strictures or inflammation in ulcerative colitis or Crohn
disease. It is contraindicated in the setting of diverticulitis until perforation has been ruled out
Small bowel follow-through is performed by giving oral contrast to the patient and using
fluoroscopy to follow the contrast into the small bowel for diagnosing small bowel pathology
(eg, stricture, obstruction, masses) and is not as useful for diagnosing diverticulitis in the colon
This patient with likely cirrhosis (heavy alcohol use, ascites, thrombocytopenia) now has
progressive confusion, slurred speech, and asterixis (flapping tremor with outstretched hands),
which is suggestive of hepatic encephalopathy (HE). HE refers to impaired CNS function in
patients with cirrhosis due to ammonia neurotoxicity from impaired liver function. Treatment
involves identifying any underlying precipitant (eg, infection, sedative medications, GI bleed)
and lowering serum ammonia. Nonabsorbable disaccharides (eg, lactulose, lactitol) are preferred
for lowering serum ammonia. Colonic bacteria metabolize lactulose to short-chain fatty acids
(eg, lactic acid, acetic acid). This acidifies the colon to stimulate conversion of the absorbable
ammonia to the nonabsorbable ammonium (an ammonia trap) and causes bowel movements
(which facilitates fecal nitrogen excretion).
Thiamine is used to treat and prevent Wernicke encephalopathy seen in alcoholics due to poor
dietary intake of thiamine. It presents as altered mental status, ataxia, and nystagmus, but
asterixis is not generally present
Dantrolene is used in neuroleptic malignant syndrome and malignant hyperthermia. Patients with
these conditions usually have severe muscle rigidity, fever, and tachycardia
Chronic pancreatitis is most commonly due to alcohol consumption (including prolonged use of
socially acceptable amounts). CP typically presents with chronic epigastric abdominal pain that
can radiate to the back and is partially relieved by sitting upright or leaning forward. Patients can
have intermittent pain-free intervals lasting from months to a year. Early CP can present with
acute attacks that become continuous as the condition progressively worsens. Patients with CP
develop progressive pancreatic inflammation that causes nonreversible exocrine and endocrine
functional damage. Diarrhea, steatorrhea, and weight loss can develop due to fat malabsorption
from reduced levels of exocrine pancreatic enzymes (eg, amylase, protease, lipase). CP
eventually causes pancreatic endocrine failure with glucose intolerance or overt diabetes.
Pancreatic calcifications seen on abdominal plain films or CT scan establishes the diagnosis. In
addition, CT scan helps exclude other etiologies (eg, pancreatic cancer, pseudocyst).
Mesenteric angiogram can diagnose chronic mesenteric ischemia, which can present with dull
abdominal pain (usually after eating) and unintentional weight loss due to avoidance of food.
Most cases are due to atherosclerosis, for which this patient has no significant risk factors.
Diarrhea and improvement in pain with position changes are uncommon.
Upper Gl endoscopy is indicated for suspected peptic ulcer disease, gastritis, dysphagia, or
hematemesis
This patient's presentation (hepatocellular injury in the absence of viral hepatitis or autoimmune
disease) is most consistent with NAFLD. Affected patients are typically middle-aged, obese, and
have features of the metabolic syndrome (eg, central obesity, DM, hyperlipidemia, HTN).
NAFLD can range from bland steatosis to inflammation and necrosis (steatohepatitis) to fibrosis
and cirrhosis. NAFLD resembles alcohol-induced liver disease (macrovesicular fat deposition,
peripheral displacement of nuclei) but occurs in patients with minimal or no alcohol history.
NAFLD can be due to increased transport of FFA from adipose tissue to the liver, decreased
oxidation of FFA in the liver, or decreased clearance of FFA from the liver (due to decreased
VLDL production). It is frequently related to peripheral insulin resistance leading to increased
peripheral lipolysis, TG synthesis, and hepatic uptake of fatty acids. Hepatic FFA increases
oxidative stress and production of proinflammatory cytokines (eg, TNF-a)
Insulin decreases lipolysis in adipose cells. Insulin resistance, as seen in NAFLD, leads to
increased lipolysis and FFA release. FFA are then transported to the liver, where they participate
in pathogenesis of NAFLD and further contribute to insulin resistance by impairing insulin-
dependent glucose uptake and increasing hepatic gluconeogenesis.
Chronic dysphagia to both solids and liquids, regurgitation, difficulty belching, and mild weight
loss are seen in achalasia. Other symptoms include chest pain and heartburn; therefore, many
patients are initially diagnosed with GERD. On average, patients have symptoms for ~5 years
before receiving a diagnosis of achalasia. Achalasia is due to impaired peristalsis of the distal
esophagus and impaired relaxation of the LES. This prevents food or liquid from passing through
the LES until the hydrostatic pressure in the esophageal column is greater than the closing
pressure of the sphincter. Being in the upright position increases the pressure in the esophagus
and results in more effective swallowing. Manometry is the most sensitive test and key to
diagnosis. Barium esophagram, which may show a smooth "bird-beak" narrowing near the LES,
can be helpful in patients with nondiagnostic manometry
Esophageal webs are located in the upper esophagus and cause mild focal narrowing (dysphagia
to solids but not liquids). They are often a/w iron deficiency (Plummer-Vinson syndrome)
Patients with typical GERD require upper GI endoscopy if they have alarm symptoms
(dysphagia, odynophagia, weight loss, anemia, Gl bleeding, or recurrent vomiting) or are men
age >50 with chronic (>5 years) symptoms and cancer risk factors (eg, tobacco use). Patients
with esophagitis on endoscopy can receive treatment based on the diagnosis (eg, autoimmune
disease, Barrett's esophagus). Patients without esophagitis on endoscopy usually require further
evaluation (eg, esophageal manometry). Patients with typical GERD symptoms who do not meet
initial endoscopy criteria can receive an initial trial of PPI. Patients with refractory symptoms
should try another PPI or increase the use of PPI to twice daily. Patients with persistent
symptoms require further testing such as endoscopy or esophageal pH monitoring. This patient’s
presentation (age > 50, unintentional weight loss, dysphagia, tobacco use) warrants initial
endoscopy (rather than PPI or H2 blockers) to assess for complications of GERD
This patient with alcohol abuse, alcoholic hepatitis (2:1 ratio of AST to ALT), esophagitis, and
gastritis now has acute, bright red hematemesis following multiple episodes of vomiting. This
presentation is highly suggestive of a Mallory-Weiss tear (MWT) which is characterized by
longitudinal tears in the mucosa near the GE junction, with bleeding due to injury to the
submucosal arteries or veins. It occurs due to a sudden increase in intraabdominal pressure (eg,
retching, blunt abdominal trauma). MWT is a common cause of upper GI hemorrhage and is
often seen in association with alcohol abuse and hiatal hernia. The diagnosis can be confirmed on
endoscopy. Bleeding stops spontaneously in 90% of patients. Those with ongoing bleeding can
be treated endoscopically with electrocoagulation or local injection of epinephrine.
Esophageal rupture can be caused by severe retching (Boerhaave syndrome), penetrating trauma,
or as a complication of endoscopy. It is characterized by acute chest pain, subcutaneous
emphysema, and often a left-sided pleural effusion. When esophageal rupture occurs due to
endoscopy, it is usually apparent shortly after the procedure
Esophageal varices are submucosal veins that have dilated due to portal hypertension. Eventual
rupture causes upper GI bleeding. However, varices would likely have been visible on this
patient's endoscopy 3 days earlier.
Stress ulcers develop in the setting of severe and prolonged physiological stress. GI bleeding
from stress ulcers is usually seen in patients in an intensive care or burn unit
Both NSAIDs and aspirin can cause gastritis and/or gastric ulcers leading to chronic GI blood
loss and depletion of iron stores. Evaluation includes blood counts, iron studies, and fecal occult
blood testing. If iron deficiency is confirmed, especially if there is ongoing blood loss, definitive
diagnosis can be obtained with upper (and usually lower) GI endoscopy. Management typically
includes withholding NSAIDs and aspirin and initiating antisecretory medication (PPI)
Elderly patients may present additional challenges in evaluating acute anemia. Besides other
potential comorbidities that can cause chronic anemia at baseline (eg, renal insufficiency,
myelodysplastic syndromes, occult malignancy, nutritional deficiencies), a significant number of
elderly patients will have a baseline anemia for which no etiology is apparent, the so-called
"idiopathic anemia of aging." They are also more likely to have additional comorbidities such as
CHF or chronic lung disease, which make them poorly tolerant of even mild anemia.
Anemia of chronic disease (anemia of chronic inflammation) is caused by suppression of red cell
production by inflammatory cytokines. It commonly occurs in inflammatory arthropathies (eg,
RA, lupus) but is not associated with osteoarthritis.
Aplastic anemia is a disorder of bone marrow stem cells that is often induced by exposure to
various drugs, chemicals, ionizing radiation, or viruses. It can also be idiopathic.
Patients with cirrhosis and portal HTN frequently have abdominal ascites and peripheral edema
secondary to low albumin levels and resulting abnormal ECF volume regulation. A small number
of these patients may also develop pleural effusions which are not secondary to an underlying
cardiac or pulmonary abnormality; this is often referred to as hepatic hydrothorax. Hepatic
hydrothorax generally results in transudative pleural effusions and is more common on the right
side. It is thought to occur secondary to small defects in the diaphragm which permit abdominal
ascites fluid to pass into the pleural space. The primary treatment for most patients is a
therapeutic thoracentesis followed by a salt restricted diet and diuretics. However, this patient
has already been on diuretics and yet a moderate to large pleural effusion has still developed. The
next best treatment for this patient would be a transjugular intrahepatic portosystemic shunt
(TIPS). The best option for treatment is liver transplantation.
IBS presents most commonly in young women as chronic, crampy abdominal pain with
alternating episodes of constipation and diarrhea. Passage of stool often relieves the pain. It
can also present with nonspecific symptoms such as GERD, dysphagia, early satiety, and
chest pain. Patients with no alarm features, no family history of inflammatory bowel disease
or colorectal cancer do not require extensive workup for diagnosis. Warning signs and
symptoms such as rectal bleeeding, nocturnal or worsening abdominal pain, weight loss, and
abnormal lab studies (anemia, elevated inflammatory markers) do not suggest IBS and
require further workup
Intestinal villous atrophy is the hallmark of celiac disease resulting in gluten intolerance. Classic
presentation includes GI distress (eg, diarrhea, abdominal distention), malabsorption, and
nutritional deficiencies.
The hepatic metabolism of bilirubin occurs in the following four stages: uptake from the
bloodstream; storage within the hepatocyte; conjugation with glucuronic acid; and biliary
excretion. In the normal individual, serum total bilirubin is 0.2-1 mg/dl, of which < 0.2 mg/dl is
the direct fraction. Typically, elevated conjugated bilirubin levels are suggestive of hepatobiliary
disease (eg, cirrhosis or hepatitis) because the secretion of conjugated bilirubin into the bile is
slowed. In contrast, elevated unconjugated bilirubin levels indicate an increased level of bilirubin
formation (hemolysis) or a slowing in bilirubin conjugation. Gilbert's syndrome is a familial
disorder of bilirubin glucuronidation in which the production of UDP glucuronyl transferases is
reduced. Certain events such as hemolysis, fasting or consuming a fat-free diet, physical
exertion, febrile illness, stress, or fatigue are thought to be triggers for hyperbilirubinemia in
these patients. Presumptive diagnosis can be made when the unconjugated hyperbilirubinemia
persists with repeat testing, but liver function tests, CBC, blood smear, and reticulocyte count are
normal. Treatment is unnecessary. However, its mode of inheritance should be discussed with
patients to preventing unneccesary testing in similarly affected family members.
Rotor syndrome is a benign condition in which there is a defect of hepatic storage of conjugated
bilirubin, resulting in its leakage into the plasma. Chronic and mild hyperbilirubinemia of both
unconjugated and conjugated forms develops, without any suggestion of hemolysis. Liver
function tests are normal, and treatment is unnecessary
Alcoholic hepatitis (AH) best explains this patient's presentation with fever, jaundice, anorexia,
tender hepatomegaly, mild (<300 U/L) elevation in aminotransferases with an AST:ALT ratio >
2:1, macrocytic anemia, thrombocytopenia, and mild elevation in the INR. Alcohol use should be
confirmed and quantified by first obtaining the patient's social history and discussing substance
use. Diagnosis is clinical and often does not require further studies in patients with consistent
history and laboratory results. Radiographic imaging may reveal fatty liver disease, cirrhosis, or
ascites. Treatment involves abstinence, supportive care (eg, hydration and nutrition support), and
acid suppression. Liver biopsy can be helpful if there is diagnostic uncertainty
Methotrexate could cause elevated MCV, bone marrow suppression (anemia, thrombocytopenia),
and liver function test abnormalities; however, an AST:ALT ratio >2:1 makes AH more likely
Cholangitis can cause fever, abdominal pain, and jaundice (Charcot's triad) and could be
evaluated with a RUQ ultrasound. However, cholangitis would typically cause a more severe
presentation (eg, sepsis) with a marked elevation in the alkaline phosphatase level; in addition,
an AST/ALT ratio 2:1 with tender hepatomegaly and an elevated MCV are atypical for
cholangitis and more suggestive of AH
This patient's clinical presentation is highly suggestive of celiac disease, with multiple features
of malabsorption and characteristic abnormalities on biopsy of the small bowel (villous atrophy).
The diagnosis of celiac disease is highly correlated with positive results on serological studies,
primarily IgA anti-tissue transglutaminase and IgA anti-endomysial antibodies. However, many
patients with biopsy-confirmed celiac disease will have negative results on IgA antibody testing
due to an associated selective IgA deficiency, which is common in celiac disease. If IgA serology
is negative but the suspicion for celiac disease is high, total IgA should be measured (or IgG-
based serologic testing should be done).
Celiac disease should be suspected in any patient with malabsorption and iron deficiency anemia.
IgA anti-endomysial and anti-tissue transglutaminase antibodies are highly predictive of celiac
disease but may be absent if there is concurrent selective IgA deficiency
Aplastic anemia is an uncommon cause of anemia. In addition to symptoms of a low red cell
count, patients develop bleeding (due to thrombocytopenia) and infection (due to leukopenia).
Ferritin is usually normal and there is no malabsorption
This patient with a history of vascular disease most likely has chronic mesenteric ischemia
(CMI). It presents with crampy, postprandial epigastric pain (intestinal angina), food aversion,
and weight loss. Patients may also report nausea, early satiety, and diarrhea. The anginal pain
frequently starts within the first hour of eating and slowly resolves over the next 2 hours due to
shunting of blood away from the small intestine to meet the increased demand of the stomach. In
patients with atherosclerosis, the celiac or the SMA may be narrowed and unable to dilate to
maintain adequate blood flow to the intestines. Physical examination shows signs of malnutrition
and may reveal an abdominal bruit in ~50% of patients. Although abdominal x-ray and CT scans
may demonstrate calcified vessels, CT angiography is the preferred choice for diagnosis. Tx
involves risk reduction (eg, tobacco cessation), nutritional support, and revascularization
Ventricular aneurysms predispose to acute mesenteric ischemia (rather than CMI) due to
thrombus formation and embolization. The presentation of AMI is abrupt, with severe abdominal
pain out of proportion to examination findings. Both in pathophysiology and acuity, acute
mesenteric ischemia is analogous to acute MI whereas CMI is similar to stable angina
The diagnosis of CMI is suspected in patients with unexplained chronic abdominal pain, weight
loss, and food aversion. Most cases are due to atherosclerotic changes of the celiac or SMA.
This patient has pancreatitis given her epigastric abdominal pain radiating to the back and the
amylase/lipase levels >3 times the upper limit of normal (ULN). Gallstone pancreatitis is most
likely as she does not use alcohol, takes no medications, has normal TG and calcium levels, and
has had no recent major illness. Her high BMI, ALT >150 U/L , and elevated ALP also suggest
gallstone pancreatitis. A RUQ ultrasound is advised for all patients with suspected gallstone
pancreatitis. If the ultrasound is nondiagnostic and there is high clinical suspicion for common
bile duct disease, ERCP is performed to better visualize the biliary tree. CT scan is not required
for diagnosis in patients with typical abdominal pain and amylase/lipase levels >3 times ULN. It
can be used to confirm the dx and identify complications (eg, pancreatic hemorrhage or necrosis)
in patients without these classic findings (eg, atypical abdominal symptoms, amylase/lipase
elevation < 3 times ULN). However, CT is not as sensitive as ultrasound for detecting gallstones
Gallstones and chronic alcohol abuse are the most common causes of acute pancreatitis
Abdominal ultrasound is the most sensitive and specific study to detect gallstones and should be
performed in all patients with suspected gallstone-induced pancreatitis.
This patient with epigastric pain and intermittent melena has a duodenal ulcer (DU). The pain is
worse on an empty stomach (due to unopposed gastric acid emptying into the duodenum) and
improves with food (due to alkaline fluid secretion into the duodenum). By contrast, the pain of
gastric ulcers is often worse after eating due to increased acid secretion. Management of H
pylori-associated ulcer requires acid suppression and antibiotic eradication therapy
Primary biliary cholangitis (primary biliary cirrhosis) can present similar to PSC; however, it
classically affects middle-aged women and has no association with UC. Patients frequently have
positive serum antimitochondrial antibodies.
Polyarteritis nodosa is a necrotizing vasculitis that affects medium-sized arteries and presents
with systemic symptoms (eg, fever, malaise, weight loss), neuropathy, arthralgias/myalgias,
cutaneous findings (eg, livedo reticularis), and renal disease. The hepatobiliary system is rarely
affected and there is no association with antineutrophil cytoplasmic antibodies or UC
Primary sclerosing cholangitis is often asymptomatic but may present with fatigue and pruritus.
Affected individuals frequently have coexisting ulcerative colitis, and laboratory studies usually
show elevated liver function tests in a cholestatic pattern
Cirrhosis especially due to alcohol or hemochromatosis can cause hypogonadism due to primary
gonadal injury or hypothalamic-pituitary dysfunction. Cirrhosis also causes elevated estradiol
due to increased conversion from androgens. Excess estrogen causes telangiectasias, palmar
erythema, testicular atrophy, and gynecomastia (usually bilateral but can be unilateral). In
addition, the liver produces thyroxine-binding globulin, transthyretin, albumin, lipoproteins.
Decreased synthesis of these proteins lowers the total T3 and T4 in circulation; however, free T3
and T4 levels are unchanged, and TSH will be normal, reflecting a euthyroid status.
Patients with nephrotic syndrome typically develop anasarca and prominent bilateral peripheral
edema (unlike the unilateral leg edema in this patient). They may also have ascites and are prone
to thrombosis due to hypercoagulable state.
Chronic pulmonary emboli can lead to pulmonary HTN and subsequent RHF (cor pulmonale).
Patients have right-sided heart murmurs, elevated JVP, bilateral peripheral edema, and
hepatojugular reflux
This patient with weight loss and fatigue has multiple liver lesions on CT scan, suggesting
metastatic disease to the liver rather than primary liver cancer. Colorectal cancer is the most
common source of liver metastases (as blood from the colon moves through the portal circulation
directly to the liver). Abdominal CT is a useful screening test but can often miss primary
intraluminal tumors. Colonoscopy is the most appropriate next diagnostic step as it both localizes
the tumor and provides a tissue diagnosis
Lung and breast cancers also metastasize to the liver but less commonly than GI malignancies.
This patient is a smoker; however, he has a normal chest x-ray and no respiratory symptoms,
making metastatic lung cancer unlikely
PSA is a useful marker for prostate cancer. However it tends to be an indolent malignancy that
commonly metastasizes to the pelvic lymph nodes and bones rather than the liver.
Pancreatic cancer is the 4th leading cause of cancer deaths. Adenocarcinoma is the most common
type. Pancreatic cancer usually occurs after age 45 and is more common in men and African
American patients. However, it is diagnosed late in the disease course and has a high mortality
rate (98%). The major hereditary risk factors for pancreatic cancer include first-degree relatives
with pancreatic cancer, hereditary pancreatitis, and germline mutations (eg, BRCA1 , BRCA2,
Peutz-Jeghers syndrome). Environmental risk factors include cigarette smoking, nonhereditary
chronic pancreatitis, and obesity with low physical activity. Cigarette smoking is the most
consistent reversible risk factor. The risk of cancer increases significantly with the number of
cigarettes consumed, and heavy smokers have nearly 2-3 times higher risk than nonsmokers.
Studies have not shown a significantly decreased incidence of pancreatic cancer with alcohol or
caffeine reduction. Heavy alcohol use can lead to chronic pancreatitis, which is associated with a
slightly higher risk of pancreatic cancer. However, this is not as significant as the association
between smoking and pancreatic cancer.
Lynch syndrome (eg, HNPCC) is an autosomal dominant cancer syndrome that predisposes
individuals to colorectal cancer and other malignancies. Genetic testing should be performed in
patients with a strong family history of colon cancer (eg, >3 relatives involving multiple
generations). The condition is due to a germline mutation in a DNA mismatch repair gene. Once
the diagnosis of Lynch syndrome is established, patients should undergo screening for colon
cancer with colonoscopy. In addition to colon cancer, patients are at extremely high risk for
endometrial carcinoma. Endometrial cancer screening with annual endometrial biopsy should
begin at age 30-35. Ovarian cancer risk is also increased and may present at a relatively younger
age. Therefore, prophylactic hysterectomy and bilateral oophorectomy is recommended at age 40
or earlier if childbearing is complete
This patient presented with septic shock and developed AST and ALT elevations one day later.
This is most consistent with ischemic hepatic injury, or shock liver. The hallmark of ischemia is a
rapid and massive increase in the transaminases with modest elevations in total bilirubin and
ALP. In patients who survive the underlying cause of their hypotension (e.g septic shock, heart
failure), liver enzymes typically return to normal within one to two weeks.
Acute hepatitis A or B infection can present with large AST and ALT elevations, accompanied by
significant hyperbilirubinemia, nausea, and vomiting
Alcoholic liver disease typically causes less dramatic increases in the transaminases compared to
ischemic hepatic injury or Hep A and B infections. The AST/ALT ratio is usually 1.5 or greater,
and the AST itself is rarely greater than 300 units/L
Autoimmune hepatitis cause large increases in the AST and ALT. However, young women are
more commonly affected, and the associated serum bilirubin increases are more dramatic
Spontaneous bacterial peritonitis can have a subtle presentation and should be considered in any
patient with cirrhosis and ascites accompanied by either fever or a change in mental status. For
making the diagnosis, paracentesis is the test of choice with the main diagnostic criteria being a
positive ascites fluid culture and neutrophil count of >250. Paracentesis should be done before
antibiotic therapy is initiated as therapy often results in negative ascites cultures. Enteric
organisms such as E.coli and Klebsiella are most common followed by the streptococcal
species; empiric therapy usually includes a third-generation cephalosporin.
This patient has evidence of cirrhosis and is found to have medium-sized, nonbleeding
esophageal varices. Variceal hemorrhage develops in approximately one third of all patients and
is a major cause of morbidity and mortality. Most cirrhotic patients should undergo diagnostic
upper endoscopy to assess for varices and to determine their risk of hemorrhage. Those with
(medium- or large-sized) varices should be started on nonselective BB. Nonselective BB (eg,
propranolol, nadolol) are recommended to decrease progression to large varices and the risk of
variceal hemorrhage. They are thought to act by decreasing adrenergic tone in mesenteric
arterioles, which results in unopposed alpha-mediated vasoconstriction and decreased portal
venous flow. Endoscopic variceal ligation is an alternate primary preventive therapy in patients
with contraindications to BB therapy.
Patients with compensated cirrhosis typically are either asymptomatic or complain of vague
symptoms (eg, anorexia, weakness, fatigue). Those with decompensated cirrhosis may present
with jaundice, pruritus, upper GI bleeding, abdominal distention due to ascites, or confusion due
to hepatic encephalopathy. Management goals in cirrhotic patients include identifying and
treating reversible factors and potential complications (eg, variceal hemorrhage, HCC, HE).
Esophageal varices are the major cause of morbidity and mortality and can occur in up to 50% of
patients. As a result, all patients with cirrhosis should undergo diagnostic endoscopy to exclude
varices, determine risk of variceal hemorrhage, and indicate strategies (eg, nonselective BB) for
primary prevention of variceal hemorrhage
Antimitochondrial antibodies are usually positive in patients with primary biliary cirrhosis
(PBC). Although PBC can cause cirrhosis, it usually is more common in women and presents
with significantly elevated ALP out of proportion to AST and ALT levels
Patients with cirrhosis should undergo screening endoscopy to exclude varices, indicate the risk
of variceal hemorrhage, and determine strategies (eg, nonselective BB) for primary prevention of
variceal hemorrhage. All patients with cirrhosis, regardless of etiology, should also undergo
surveillance for HCC with ultrasound every 6 months.
The clinical scenario described is characteristic for inflammatory diarrhea. The typical cause is
idiopathic IBD. Infectious causes are less likely with chronic diarrhea (duration > 4 weeks) than
with acute diarrhea. In this case, the clues to the correct diagnosis include weight loss, anemia,
elevated ESR, and reactive thrombocytosis. The positive occult blood test is also a very
important clue. Furthermore, inflammatory changes in the blood (eg, anemia, elevated ESR,
acute phase reactants, reactive thrombocytosis) and blood/leukocyte positive stool are usually
absent in the other types of diarrhea
Osmotic diarrhea is caused by the ingestion of osmotically active, poorly absorbable substances.
Lactose intolerance is a classic example.
Ascites can be due to portal hypertensive (eg, cardiac ascites, cirrhosis) or non-portal
hypertensive (eg, malignancy, pancreatitis, nephrotic syndrome, tuberculosis) causes.
Assessment of color, neutrophil count, total protein, and the serum-to-ascites albumin gradient
(SAAG) in ascitic fluid analysis can help to diagnose the etiology. SAAG is calculated by
subtracting the peritoneal fluid albumin concentration from the serum albumin concentration and
is useful to differentiate between portal and non-portal hypertensive etiologies. A SAAG >1.1g/dl
indicates portal hypertension while a SAAG <1 .1 g/dl suggests other causes (malignancy,
pancreatitis, nephrotic syndrome, tuberculosis). This patient's SAAG of 1.3 g/dl (3.8- 2.5 g/dl) is
consistent with portal hypertension (eg, cirrhosis) with increased hydrostatic pressure within
hepatic capillary beds.
Although primary hyperaldosteronism causes sodium and water retention, it typically does not
cause significant ascites or edema due to spontaneous diuresis (aldosterone escape phenomena)
This patient presents with evidence of rhabdomyolysis and a first seizure following simultaneous
use of cocaine and heroin (i.e , speedball). Cocaine and opiate intoxication both predispose to
seizures, and cocaine can also lead to rhabdomyolysis. The muscle breakdown can lead to
significant myoglobinuria and eventual renal failure. As a result, this patient should initially
acutely be treated with aggressive fluid resuscitation to treat the rhabdomyolysis and prevent
renal failure. This patient also has elevated transaminases and positive anti- hepatitis C virus
(HCV) antibodies. Hepatitis C is transmitted by exposure to infected blood, IV drug use, tattoos,
and blood transfusions. Hepatitis B is also transmitted by infected blood (as well as by sexual
contact), and patients with risk factors for HCV are also at high risk for HBV. This patient is
negative for both HBsAg and anti-HBsAg, indicating that he is neither infected with nor
vaccinated against HBV. In addition to treating his acute condition, hepatitis B vaccination is
important to prevent future HBV infection. BB should be avoided in cocaine intoxication
because unopposed alpha activation can worsen HTN and lead to coronary vasospasm.
This patient with hepatitis C and alcoholic liver disease has an upper GI hemorrhage due to
esophageal varices. Initial treatment includes volume resuscitation through 2 to 3 large-bore
peripheral IV lines. Prophylactic antibiotics (eg, ceftriaxone) should be given to cirrhotic patients
with GI bleeding to decrease infectious complications, recurrent bleeding, and mortality.
Somatostatin analogues (eg, octreotide) inhibit the release of vasodilator hormones, which leads
indirectly to splanchnic vasoconstriction and decreased portal flow. Urgent endoscopy (eg,
endoscopic band ligation, sclerotherapy) can diagnose and treat active bleeding. Patients with
uncontrollable bleeding after urgent endoscopy require temporary balloon tamponade (eg,
Sengstaken-Blakemore, Minnesota, Linton-Nachlas tubes) as a short-term measure until more
definitive therapy, such as transjugular intrahepatic portosystemic shunt (TIPS) or shunt surgery
can be done. Patients without further bleeding after endoscopy can be monitored and receive
secondary prophylaxis (BB) with repeat endoscopic band ligation 1-2 weeks later.
Current guidelines suggest keeping the Hb >9 g/dl in variceal hemorrhage. This patient should
have serial blood counts drawn and may require transfusion if the Hb decreases to <9 g/dL.
Platelet transfusions are reserved for patients with active bleeding and platelet count < 50,000.
This patient's platelet count of 72,000 does not require platelet transfusion at this time.
Dysphagia to solids and liquids and a dilated esophagus with smooth tapering of the distal
esophagus suggests either primary achalasia (ie, loss of peristalsis in the distal esophagus with
lack of LES relaxation) or pseudoachalasia due to esophageal cancer. Several clues point to
pseudoachalasia (eg, narrowing of distal esophagus not due to denervation) caused by
malignancy. Tobacco use is a major risk factor for esophageal adenocarcinoma and SCC; alcohol
use is also an important risk factor for esophageal SCC. Significant weight loss, rapid symptom
onset (<6 months), and presentation at age >60 all increase the likelihood of malignancy (by
comparison, patients with achalasia have symptoms for approximately 5 years before receiving a
diagnosis, and they typically only have mild weight loss). Tumor metastasis (eg, mediastinal
lymph nodes) or local involvement may give a radiologic appearance similar to that seen with a
widened mediastinum. Endoscopic evaluation can differentiate between achalasia and
pseudoachalasia. In achalasia, this evaluation usually shows normal-appearing esophageal
mucosa and a dilated esophagus with possible residual material; in addition, it is generally
possible to easily pass the endoscope through the LES (unlike in malignancy)
If endoscopy shows a malignancy, a CT scan can be performed for staging. CT scan can also be
obtained if endoscopy is non revealing and there is still concern for malignancy
Laparoscopic myotomy and pneumatic balloon dilation are the treatments of choice for patients
with achalasia who are at low surgical risk. Options for patients at high surgical risk include
botulinum toxin injection, nitrates, and CCB. However, all these treatments should be considered
only after malignancy is excluded and the diagnosis of achalasia is confirmed
Pseudoachalasia, which is due to narrowing of the distal esophagus secondary to causes other
than denervation (eg, esophageal cancer), can closely mimic achalasia. Clues pointing to
pseudoachalasia include significant weight loss, rapid symptom onset, and presentation at age
>60. Consequently, endoscopy is recommended to exclude malignancy in all patients with
suspected achalasia.
Esophageal pH monitoring is used to confirm GERD in patients who have symptoms consistent
with GERD but who fail a trial of PPI therapy.
Hepatorenal syndrome (HRS) is one of the most dangerous complications of end-stage liver
disease, occurring in up to 10% of patients with cirrhosis. HRS is characterized by decreased
glomerular filtration in the absence of shock, proteinuria, or other clear cause of renal
dysfunction, and a failure to respond to a 1.5 L normal saline bolus. It’s due to renal
vasoconstriction in response to decreased total RBF and vasodilatory substance synthesis. The
MCC of death is infection and hemorrhage. Medication is not beneficial in HRS and the
mortality for these patients placed on dialysis is very high. Liver transplantation is the only
intervention with established benefit
This patient with abdominal pain, microcytic anemia, positive fecal occult blood, and
hepatomegaly with a hard edge on liver palpation has typical features of GI malignancy, likely
colon cancer, metastatic to the liver. The liver is the most common site of metastasis in colon
cancer. The moderate abnormalities in hepatic markers in this case (elevated ALP and slightly
elevated AST and ALT) are more suggestive of infiltrative or cholestatic disease than of
hepatocellular injury. The patient’s chest Xray also shows a small pleural effusion on the left that
may be malignant (Hepatic hydrothorax due to cirrhosis usually occurs on the right). This patient
should have a CT of the abdomen with IV contrast to evaluate for malignancy
Liver disease due to Alcohol is often shrunken and the edge is nonpalpable. AST > ALT by 1.5-2
Autoimmune hepatitis is seen in young to middle-aged women and may present as acute or
chronic hepatitis. There is significant hepatocellular injury, with transaminases often above 1,000
LV failure does not directly cause hepatomegaly, but may lead to RHF. Pulmonary HTN and
RHF may lead to congestive hepatopathy. In such cases, other signs of RV failure will be present:
peripheral edema, elevated JVP, positive hepatojugular reflux, and an S3 on cardiac exam
Nonalcoholic steatohepatitis (NASH) is often asymptomatic and manifests as hepatomegaly with
elevated transaminases. Imaging will reveal fatty infiltration of the liver. The leading causes of
NASH are obesity, DM, and hypertriglyceridemia. This patient’s hard hepatomegaly, anemia,
and positive fecal occult blood in the setting of minimal transaminase elevations are more
consistent with metastatic disease
Cirrhotic and noncirrhotic portal HTN result in ascites and significant peripheral edema. Other
signs of portal HTN include esophageal varices, spider nevi, palmar erythema, and caput
medusa. Thrombocytopenia and coagulopathy are often seen.
This patient has chronic diarrhea, steatorrhea, and weight loss suggesting malabsorption. Celiac
disease is characterized by atrophy of intestinal villi in the proximal small bowel due to exposure
to gluten-containing wheat products. D-xylose is a monosaccharide that can be absorbed in the
proximal small intestine without degradation by pancreatic or brush border enzymes. It is
subsequently excreted in the urine. In the D-xylose test, the patient is given an oral dose of D-
xylose, with subsequent assay of urine and venous blood. Patients with proximal small intestinal
disease can’t absorb D-xylose in the intestine, and urinary and venous D-xylose levels will be
low. By contrast, patients with malabsorption due to enzyme deficiencies (eg, chronic
pancreatitis) will have normal absorption of D-xylose. A false-positive D-xylose test (ie,
decreased urinary excretion of D-xylose despite normal mucosal absorption) can be seen in
patients with delayed gastric emptying or impaired glomerular filtration. This patient has low
urinary excretion (ie, low mucosal absorption) of D-xylose consistent with celiac disease.
This patient has abdominal pain, hematemesis, and melena, likely due to upper GI bleeding from
a peptic ulcer. His tachycardia suggests volume depletion. Patients with upper (but not lower) GI
bleeding often have an elevated BUN/creatinine ratio. Possible causes include increased urea
production from intestinal breakdown of Hb and increased urea reabsorption in the proximal
tubule due to associated hypovolemia
ALP is increased in biliary obstruction and skeletal disease with increased osteoblast activity (eg,
Paget disease). Mild elevations can be seen in IBD or intra-abdominal infections
Prolonged prothrombin time can be seen with warfarin use, vitamin K deficiency, certain
hereditary coagulation disorders, antibiotic use, and liver disease.
The urine sodium (FENa) is low in patients who are volume depleted as the kidney attempts to
retain sodium to restore circulatory volume. Increased FENa is seen in intrinsic renal disease.
This patient's abdominal pain, nausea, and elevated lipase suggests acute pancreatitis. He has no
history of alcohol use, and his ultrasound reveals no gallstones. His pancreatitis is likely due to
cholesterol emboli. Patients with risk factors for aortic atherosclerosis (eg, hypercholesterolemia,
diabetes, peripheral vascular disease) who undergo cardiac catheterization or a vascular
procedure are at increased risk for cholesterol emboli as a result of vascular manipulation. These
emboli can occlude blood vessels and cause livedo reticularis (reticulated, mottled, discolored
skin), blue toe syndrome, acute kidney injury, pancreatitis, and mesenteric ischemia. Supportive
care (eg, pain control, IV fluids, bowel rest) is recommended for uncorrectable causes of acute
pancreatitis (eg, hypotension, ischemia, viruses, atheroembolism). Most attacks are self-limiting
and improve in 4-7 days with conservative management. This patient should receive nothing by
mouth except essential medications (ie, antiplatelet therapy to prevent stent thrombosis).
Elective cholecystectomy is indicated for patients with gallstones and biliary colic/cholecystitis.
ERCP is indicated for sphincterotomy and stone removal in patients with gallstone pancreatitis
and cholangitis or in those who have high surgical risk for cholecystectomy. This patient has no
gallstones or common bile duct dilation on ultrasound.
Cholangitis is inflammation of bile duct system that carries bile from liver and gallbladder to
duodenum. It is due to bacterial infection.
For adenocarcinoma of the stomach, treatment options and prognosis are determined by the
disease stage at the time of diagnosis. Surgical removal of the affected tissues is the mainstay of
therapy, therefore early detection of gastric cancer improves the chances of successful
management. However, almost 90% of patients with gastric cancer are diagnosed at advanced
stages (III-IV), at which point radical resection is very complicated or impossible. For this
reason, evaluation of the extent of the cancer is the most important following initial histologic
diagnosis. A CT scan of the abdomen and pelvis allows disease staging and is sensitive for
revealing metastases (especially in the liver). Depending on CT findings, additional staging
procedures, such as laparoscopy, endoscopic ultrasound, chest CT, or PET/CT may be necessary.
Patients diagnosed with limited-stage disease may be considered for curative resection, but those
with more advanced disease staging are referred for palliative interventions. Laparotomy is
eventually required for curative or palliative management, but initial CT imaging is needed to
plan the appropriate next steps in evaluation.
H.pylori infection is a significant risk factor for gastric adenocarcinoma and MALT lymphoma.
Eradication is recommended for patients with resectable disease to reduce the risk of developing
a second cancer. Eradication of H pylori causes remission in some patients with gastric MALT
lymphoma, but it is not curative for adenocarcinoma.
Most patients with NAFLD have hepatomegaly, but other stigmata of chronic liver disease are
rarely seen. Mild elevations in AST and ALT, with an AST/ALT ratio <1.5; Diagnosis is based on
lab and imaging (eg, hyperechoic texture on ultrasound), but it can be confirmed with liver
biopsy. Hepatic fibrosis develops in 40% and cirrhosis in 10%-15%. Mgt includes weight loss
and control of metabolic risk factors. It is safe to continue statin therapy in NAFLD patients.
The hepatic steatosis in NAFLD resembles that found in alcoholic liver disease. However,
alcoholic liver disease is characterized by AST predominance (AST/AL T ratio 2:1) in contrast to
the parallel rise in NAFLD. In addition, alcoholic liver disease is unlikely with light to moderate
alcohol intake (<15 drinks/wk for men, <10/wk for women)
Antinuclear antibody titers are a sensitive marker for autoimmune hepatitis, and antismooth
muscle antibody titers are relatively sensitive and specific
This patient's worsening epigastric pain and weight loss in the setting of idiopathic chronic
pancreatitis suggest pancreatic cancer. Its Characterized by weight loss and insidious onset of
gnawing abdominal pain (usually epigastric and radiating to the back), which is worse at night,
with eating, or when lying supine. • Cancers in the head of the pancreas (60%-70%) present with
jaundice (common bile duct obstruction, elevated ALP and bilirubin) and steatorrhea (pancreatic
exocrine insufficiency or pancreatic duct blockage). Abdominal US is preferred for detecting
pancreatic head tumors and excluding other causes of biliary obstruction (choledocholithiasis).
• Cancers in the body and tail usually do not present with obstructive jaundice. Abdominal CT
scan is preferred since US is less sensitive for visualizing the pancreatic body and tail (due to
overlying bowel gas) and for detecting smaller (<3 cm) tumors.
ERCP is more invasive than CT scan thus reserved for patients with cholestasis (tumor
compressing the biliary system) who may require an intervention (eg, stenting). ERCP is usually
done to evaluate biliary pancreatitis (blocked duct causing pancreas inflammation). US may be
normal (especially if the stone is passed) but they have an elevated ALT level (>150 U/L). ERCP
should also be considered in patients with >1 episode of acute pancreatitis of unknown cause.
ERCP is usually performed in patients when initial ultrasonography or CT scan suggests the
presence of obstruction due to cholelithiasis or malignancy. ERCP in these settings can be both
diagnostic and therapeutic by relieving obstruction and facilitating biliary drainage
Secretin test directly measures the ability of pancreatic ductal cells to produce bicarbonate. It is
useful in diagnosing chronic pancreatitis, but not helpful in evaluating possible pancreatic cancer.
This patient's epigastric pain, vomiting, and markedly elevated lipase are consistent with acute
pancreatitis. The MCC of acute pancreatitis include alcohol and gallstones. Other causes include
hypertriglyceridemia (third MC), meds (eg, azathioprine, valproic acid, thiazides), infections (eg,
CMV), recent ERCP, atheroembolic/ischemic. The yellow-red papules on this patients arms and
shoulders suggest eruptive xanthomas due to subcutaneous fat deposition associated with marked
hypertriglyceridemia (>1,000 mg/dl), typically due to familial hypertriglyceridemia (which may
also explain the patient's father's MI at age 42). A fasting serum lipid profile can determine if
hypertriglyceridemia is the underlying cause of this patient's xanthomas and pancreatitis
Features of acute esophageal rupture include acute chest pain, tachypnea, and pleural effusion
(often left-sided). Patients also frequently have subcutaneous emphysema. Endoscopy is the
MCC of esophageal rupture (especially when performed with additional interventions such as
dilation of strictures), although rupture can also be due to severe retching (Boerhaave syndrome)
or penetrating trauma. Esophageal rupture is a life-threatening condition that requires emergent
evaluation. A contrast esophagram is recommended to confirm the diagnosis and demonstrates
contrast extravasation through the tear. Water-soluble contrast is preferred (less inflammatory to
tissues), but a barium study has higher sensitivity and its done if the initial water soluble contrast
study is nondiagnostic. If perforation is confirmed, primary closure of esophagus and drainage of
mediastinum must be attempted urgently to prevent the development of mediastinitis.
Serum amylase and lipase levels are elevated in acute pancreatitis (which sometimes occurs
following ERCP). Pancreatitis can cause a left pleural effusion but is less likely in this patient
given his lack of typical epigastric pain and his symptoms that abruptly worsened following
endoscopy. Pleural fluid (rather than serum) amylase would be elevated in esophageal rupture
Features typical of dyspepsia are epigastric fullness or discomfort after eating. Dyspepsia is often
a/w nausea and symptoms of GERD. In addition to GERD, common causes of dyspepsia include
NSAID, gastric or esophageal cancer, functional dyspepsia, and symptomatic H.pylori infection
(with or without peptic ulcer). Definitive diagnosis requires upper GI endoscopy and should be
considered in patients age >55 and those with alarm symptoms such as weight loss, gross/occult
bleeding, anemia, dysphagia, persistent vomiting, or early satiety. Young patients without alarm
symptoms can be managed with a trial of antisecretory therapy, either a PPI or H2 blocker. A PPI
is preferred for patients with dyspepsia associated with use of NSAID. Patients with an increased
risk for H. pylori (regions with high prevalence or household exposure) may pursue a "test-and-
treat" strategy. A test for active H pylori infection (breath test or stool assay, but not serology) is
administered, and patients receive appropriate antibiotic therapy if positive. Regardless of alarm
symptoms, patients who fail to improve after initial measures should undergo endoscopy.
This patient's pale skin, nail beds, and conjunctiva suggest anemia. His diet lacks folate (found in
fresh green leafy vegetables and liver). Folic acid stores become depleted within 4-5 months,
leading to decreased RBC production and macrocytic anemia.
Strict vegetarians become deficient in vitamin B12 causing anemia accompanied by neurologic
deficits (in contrast to folate deficiency, which causes anemia alone). The body has B12 stores
sufficient to last at least 3-4 years.
Vitamin C deficiency can cause scurvy. Patients present with perifollicular hemorrhage, swollen
gums, and poor wound healing
Vitamin E deficiency can cause RBC fragility, hyporeflexia, muscle weakness, and blindness
This patient has elevated conjugated hyperbilirubinemia and ALP consistent with cholestasis due
to extrahepatic or intrahepatic biliary obstruction. The next appropriate step in management for
patients with predominantly cholestatic pattern is to obtain an abdominal US to assess hepatic
parenchyma and biliary ducts. The presence of biliary dilatation is suggestive of extrahepatic
cholestasis; the absence of biliary dilatation suggests intrahepatic cholestasis.
UC presents with leukocytosis, iron deficiency anemia, reactive thrombocytosis, and elevated
ESR. Endoscopy shows erythematous and friable mucosa with ulcers.The lesions are continuous
and circumferential (unlike in CD). Biopsy shows inflammation limited to the mucosa and
submucosa (compared to transmural inflammation in CD) with crypt abscesses and crypt
distortion. Patients commonly have abdominal pain, bloody diarrhea, tenesmus, and weight loss.
UC may be complicated by toxic megacolon. The most common extraintestinal manifestations
include arthritis, uveitis, episcleritis, erythema nodosum, and Primary sclerosing cholangitis.
This patient has jaundice and icterus with a positive urine bilirubin assay, consistent with a
buildup of conjugated bilirubin. The etiology is not likely due to hepatic injury (normal
transaminases) or biliary duct obstruction (normal ALP). Rotor’s syndrome is a benign condition
with conjugated hyperbilirubinemia due to a defect in hepatic secretion. An understanding of the
bilirubin metabolism pathway clarifies how a positive urine bilirubin assay reflects a buildup of
conjugated bilirubin. Following Hb breakdown, the majority (95%) of bilirubin is in the
unconjugated form which undergoes hepatic conjugation. Conjugated bilirubin then goes into the
intestines where it is deconjugated and recycled (enterohepatic circulation) to be reconjugated in
the liver. Alternatively, some of its metabolism products are excreted in feces as urobilinogen,
with ~ 1% of urobilinogen being renally excreted. Unconjugated bilirubin (tightly bound to
albumin) is highly insoluble, so it cannot be excreted in urine. Excess unconjugated bilirubin (eg,
hemolysis) undergoes the regular conjugation pathway with eventual production of urobilinogen,
and the resultant excess urobilinogen is primarily recycled; however, a higher than normal
amount of urobilinogen is excreted in feces or urine, leading to a positive urine urobilinogen
assay (not seen in this patient). On the other hand, conjugated bilirubin (loosely bound to
albumin) is water soluble, so it can be excreted in urine. Despite this, no bilirubin is normally
seen on urinalysis (negative urine bilirubin assay) because conjugated bilirubin is usually
degraded in the intestines. However, when there is hepatic dysfunction, biliary obstruction, or a
defect in hepatic bilirubin secretion (as in this patient), there is plasma buildup of conjugated
bilirubin, which leaks into urine. This results in dark urine and a positive urine bilirubin assay
Laxative abuse is characterized by very frequent, watery, nocturnal diarrhea. The diagnosis can
be confirmed with the characteristic biopsy finding of dark brown discoloration of the colon with
lymph follicles shining through as pale patches (melanosis coli)
Hepatitis A is an RNA picornavirus with incubation period of 30 days. Onset is acute, and
symptoms can include malaise, fatigue, anorexia, nausea, vomiting, mild abdominal pain, and an
aversion to smoking. AST and ALT spike early in the illness, followed by increases in bilirubin
and ALP. It is self-limiting disease and does not progress to chronic hepatitis, cirrhosis, or HCC.
The mortality rate is less than 0.2%, although a significantly prolonged PT time correlates with
increased mortality. Treatment is largely supportive, with complete recovery in 3-6 weeks. Close
contacts should promptly be given immune globulin. People considered at high risk (eg, those
living in or traveling to endemic areas, those with chronic liver disease or clotting-factor
disorders, men who have sex with men) should be given the hepatitis A vaccine as prophylaxis
PAN is more commonly a/w HBV than HCV. It is a necrotizing vasculitis of small and medium
vessels due to the deposition of antigen-antibody immune complexes in blood vessel walls.
A patient with familial colonic polyposis has a 100% risk of cancer, if not treated appropriately
with a proctocolectomy at the time of diagnosis
Chronic GERD and Barrett's esophagus are risk factors for adenocarcinoma of the esophagus.
Patients with Barrett's esophagus have a 1% per year risk of developing adenocarcinoma of the
esophagus. However, the major risk factors for SCC of the esophagus are smoking and alcohol.
Spontaneous bacterial peritonitis (SBP) should be suspected in any patient with cirrhosis and
ascites who presents with low-grade fever, abdominal discomfort, or altered mental status. The
Reitan trail test, a timed connect-the-numbers test, helps detect subtle mental status changes.
Patients with cirrhosis are often relatively hypothermic thus any temperature > 37.8C (100F)
warrants investigation. Hypotension, hypothermia, or paralytic ileus (dilated loops of bowel on
x-ray) indicate severe SBP. As the vast majority of cases of SBP are associated with cirrhosis, the
SAAG is usually > 1.1(SAAG < 1.1 g/dl makes SBP unlikely). Ascitic fluid with PMN >250/mm
and positive peritoneal fluid culture confirm the diagnosis.
Alcoholic hepatitis can also present similarly to SBP with fever, RUQ abdominal pain, and
peripheral leukocytosis. However, alcoholic hepatitis does not usually cause diffuse abdominal
pain and decreased bowel sounds. Encephalopathy is seen only in severe cases.
Small bowel obstruction (SBO) may present with diffuse abdominal pain and dilated loops of
small bowel, but nausea and vomiting are much more common and bowel sounds tend to be
high-pitched initially. Dilated loops of large bowel would not be expected with SBO (although
air in the colon can sometimes be seen with partial SBO)
Lethargy, confusion, and asterixis suggests hepatic encephalopathy (HE) due to neurotoxicity
from ammonia (NH3) caused by impaired liver function. In this patient, HE was likely triggered
by the recent initiation of diuretics and poor oral intake leading to low IV volume (hypotension,
dry mucous membranes). Hypokalemia can exacerbate HE as the resultant intracellular acidosis
(excreted intracellular potassium replaced by hydrogen ions to maintain electroneutrality) causes
increased NH3 production (glutamine conversion) in renal tubular cells. Metabolic alkalosis
(elevated bicarbonate) can also exacerbate HE as it promotes conversion of ammonium (NH4),
which cannot enter the CNS, to NH3 which can. Patients with HE and hypokalemia require
prompt potassium and IV volume repletion. Disaccharides (eg, lactulose, lactitol) are also
administered to lower NH3 Ievels
Neomycin is a nonabsorbable antibiotic used to treat HE in patients unresponsive to lactulose
and those unable to tolerate rifaximin
Portal vein: Takes blood from spleen, stomach, pancreas, and intestines to the liver
During the TIPS procedure, a tunnel is made through the liver with a needle, connecting the
portal vein to one of the hepatic veins. A metal stent is placed in this tunnel to keep it open. The
procedure reroutes blood flow in the liver and reduces pressure in abnormal veins, not only in the
stomach and esophagus, but also in the bowel and the liver.
TIPS is performed when a patient has ascites that does not respond to medical therapy (eg,
diuretics) or has ongoing active or recurrent variceal bleeding even after appropriate treatment
with upper endoscopy. TIPS is associated with HE in up to 35% of patients due to (NH3-rich)
blood bypassing the liver.
Most colon cancers develop from polyps. The risk factors for a polyp progressing into cancer are
villous adenoma, sessile adenoma, and size >2.5cm. Only adenomatous polyps are premalignant,
but <1% of such lesions progress to malignancy. Hyperplastic polyps are non-neoplastic and do
not require further work-up.
1. Hyperplastic polyps: These are the most common non-neoplastic polyps in the colon. These
arise from hyperplastic mucosal proliferation. No further work-up is needed.
2. Hamartomatous polyps: These include juvenile polyp (a non-malignant lesion, generally
removed due to the risk of bleeding) and Peutz Jeghers polyp (generally non-malignant).
3. Adenoma: Most common type of polyp thats present in 30-50 % of elderly and are potentially
premalignant; however, <1% of such polyps become malignant. The likelihood of an
adenomatous lesion containing invasive cancer also depends on the size of the polyp. The risk is
negligible (<2%) with <1.5 cm, intermediate (2-10%) with 1.5-2.5 cm, and substantial (10%)
with >2.5 cm.
This patient had acute symptoms due to a small kidney stone, which has now passed.
However, she also had an incidental finding of asymptomatic gallstones. Only 20% of
patients with asymptomatic gallstones will develop symptoms within 15 years. In light of
this, no treatment is typically required for such patients. Exceptions to this rule include
patients who are at increased risk for developing gallbladder carcinoma (eg, porcelain
gallbladder)
Lamivudine (3TC) is a reverse transcriptase inhibitor used to treat HIV and chronic hepatitis B.
This woman's HBsAg is negative, arguing against active infection with hepatitis B.
Patients with cirrhosis are intravascularly depleted and impressive salt retainers. A low-salt diet
is beneficial in decreasing ascites and peripheral edema by decreasing the total amount of sodium
that can be retained. This patient does not have cirrhosis, so a low-salt diet is not necessary
Prednisone is used to treat severe alcoholic hepatitis. This disease typically presents with fever,
abdominal pain, jaundice, nausea, and vomiting
Diverticular bleeding is typically painless, but large-volume bleeding may be associated with
lightheadedness and hemodynamic instability. Low- or moderate-volume bleeding from the right
colon will mix with stool and pass as dark or maroon-colored hematochezia. Large-volume
hemorrhage can lead to passage of frank red blood. The diagnosis is confirmed on colonoscopy.
Most cases of diverticular hemorrhage will resolve spontaneously, but a minority of patients will
require endoscopic or surgical intervention.
Colonic angiodysplasia can cause painless bleeding in the right colon. However, it is less
common than diverticular hemorrhage. Angiodysplasia usually also causes low-volume (venous)
bleeding, whereas diverticulosis can cause large-volume arterial hemorrhage.
Acute mesenteric thrombosis presents with abdominal pain out of proportion to physical
findings, nausea/vomiting, and bloody diarrhea due to mucosal sloughing. Patients have
numerous atherosclerotic risk factors
This patient has abrupt-onset retrosternal pain and severe odynophagia, suggestive of
medication-induced esophagitis (pill esophagitis). Pill esophagitis is due to a direct effect of
certain medications on esophageal mucosa. Mucosal injury can be due to acid effect (eg,
tetracyclines), osmotic tissue injury (eg, potassium chloride), or disruption of normal
gastroesophageal protection (eg, NSAID). Patients usually do not have prior esophageal disease,
although pill esophagitis can be worse in those with concurrent gastroesophageal reflux. It is
most common in the mid esophagus due to compression by the aortic arch or an enlarged left
atrium. The diagnosis is usually made clinically but can be confirmed on endoscopy, which
shows discrete ulcers with relatively normal-appearing surrounding mucosa. Treatment includes
stopping the offending medication to prevent future injury
Endoscopy in Candida esophagitis is characterized by white plaques, and oral thrush. Ulcerating
lesions are seen in esophagitis due to HSV (vesicles and round/ovoid ulcers) or CMV (large
linear ulcers) and are most common in immunocompromised patients
Esophageal cancer typically presents with progressive solid food dysphagia and unintentional
weight loss. Risk factors for SCC include alcohol and tobacco use, whereas risk factors for
adenocarcinoma include Barrett esophagus, GERD, smoking, and obesity
GERD typically causes recurrent or persistent burning pain in the upper abdomen and chest.
Symptoms are worse following large meals or certain foods (eg, chocolate, peppermint) or when
lying down. Symptoms are usually subacute to chronic rather than abrupt
Acute cholecystitis presents with sudden onset of steady epigastric or RUQ pain after a large or
fatty meal. The pain may radiate to the right scapula or be accompanied by Murphy's sign,
described as worsening of RUQ pain with inspiration. Other classical findings include fever,
vomiting, and leukocytosis. Uncomplicated cholecystitis can cause mild elevations in
transaminases, total serum bilirubin from 1-4 mg/dl, and serum amylase without obvious
common bile duct or pancreatic disease. In these cases, the laboratory elevations are usually due
to passage of sludge or pus in the common bile duct. However ALP is usually not elevated
without associated cholangitis or choledocholithiasis. Acute cholecystitis is secondary to
gallstone formation commonly arising when a gallstone blocks the cystic duct. Ingestion of fatty
foods then stimulates the contraction of the gallbladder against the obstructed cystic duct, and
leads to severe colicky pain. The tissue behind the duct obstruction becomes inflamed, from
stasis leading to bacterial overgrowth. The subsequent ischemic changes lead to gangrene and
perforation, generalized peritonitis, or a well-circumscribed abscess. Other potential
complications include cholangitis and chronic cholecystitis. Treatment for acute cholecystitis
includes supportive care (nothing by mouth, IV antibiotics, and analgesics). Laparoscopic
cholecystectomy is recommended shortly after hospitalization and should be performed
immediately in cases of perforation or gangrene
Acute liver failure (ALF) is characterized by severe acute liver injury in a patient without
cirrhosis or underlying liver disease. The diagnosis requires • Severe acute liver injury as
evidenced by elevated aminotransferases (often >1000U L), • Signs of hepatic encephalopathy
(HE), • Impaired hepatic synthetic function (defined as INR > 1.5). The presence of HE
differentiates ALF from acute hepatitis, which has a much better prognosis than ALF. Other
common manifestations of ALF include fatigue, lethargy, nausea, vomiting, jaundice, pruritus,
and RUQ pain. The MCC of ALF are drug toxicity (acetaminophen overdose) and acute viral
hepatitis (hepatitis A virus, hepatitis B virus). Other causes include autoimmune hepatitis,
ischemia, Wilson disease, and malignant infiltration of the liver. Acute superinfection with
hepatitis D virus carries a high risk of ALF in IV drug users with chronic HBV.
Acute liver failure is defined as acute onset of severe liver injury with encephalopathy and
impaired synthetic function (defined as INR >1.5) in a patient without cirrhosis or underlying
liver disease. Drug toxicity and acute viral hepatitis are the most common causes.
This patient's presentation is consistent with acute severe pancreatitis that has progressed to
multisystem organ dysfunction (eg, shock, renal failure, early respiratory failure). Most patients
with acute pancreatitis have mild disease and recover with conservative management (eg, fluids,
bowel rest, pain medication) in 3-5 days. However, nearly 15%-20% of patients can develop
severe acute pancreatitis, defined as pancreatitis with failure of at least 1 organ. Clinical markers
associated with increased risk for severe pancreatitis include age >75, alcoholism, obesity, C-
reactive protein >150 mg/dl at 48 hours, and increased BUN/creatinine in the first 48 hours.
Abdominal imaging (CT or MRCP) is indicated for suspected severe pancreatitis to look for
pancreatic necrosis and extrapancreatic inflammation. Severe pancreatitis causes local release of
activated pancreatic enzymes that enter the vascular system and increase vascular permeability
within and around the pancreas leading to large volumes of fluid migrating from the vascular
system to the surrounding retroperitoneum. Systemic inflammation also ensues as the
inflammatory mediators enter the vascular system. The net effect is widespread vasodilation,
capillary leak, shock, and associated end-organ damage. Treatment usually involves supportive
care with several liters of IV fluid to replace the lost intravascular volume.
Acute pancreatitis complicated by hypotension is thought to arise from intravascular volume loss
secondary to local and systemic vascular endothelial injury. This causes vasodilation, increased
vascular permeability, and plasma leak into the retroperitoneum, resulting in systemic
hypotension
Chronic GERD with new dysphagia and symmetric lower esophageal narrowing suggests
esophageal (peptic) stricture. Chronic GERD predisposes to Barrett's esophagus (intestinal
metaplasia of the lower esophagus) and esophageal strictures. Both conditions are due to body’s
response to chronic gastric acid exposure and can occur simultaneously. Benign strictures affect
5%- 15% of patients with GERD. Other causes of peptic strictures include radiation, systemic
sclerosis, and caustic ingestions. Strictures typically cause slowly progressive dysphagia to solid
foods without anorexia or weight loss. As they progress, they can actually block reflux, leading
to improvement of heartburn symptoms (as seen in this patient). Strictures tend to appear as
symmetric, circumferential narrowing on barium swallow. Stricture in the setting of Barrett’s
esophagus, biopsy is necessary to rule out adenocarcinoma via endoscopy which may be
diagnostic and therapeutic (dilation is performed if no malignancy is detected)
Adenocarcinoma occurs in patients who have had GERD symptoms for >20 years. Early
symptoms are subtle and include retrosternal discomfort, mild dysphagia to solid foods, and/or a
burning sensation. Barium swallow shows asymmetric narrowing of the esophageal lumen.
GERD predisposes to Barrett's esophagus, erosive esophagitis, and esophageal (peptic) stricture
formation. Peptic strictures cause symmetric and circumferential narrowing of the involved
esophagus with dysphagia for solids but typically no weight loss. Other causes of peptic
strictures include radiation, systemic sclerosis, and caustic ingestions
Achalasia is an esophageal motility disorder that presents with dysphagia (both solids and
liquids) and regurgitation of undigested food or saliva. Barium swallow typically shows
aperistalsis, poor emptying of barium, dilation of the proximal esophagus (caused by retention of
food), and narrowing in a "bird beak" pattern at the gastroesophageal junction
A hiatal hernia is a protrusion of the stomach above the diaphragm. These patients may have
GERD and are also at risk for Barrett's esophagus, peptic strictures, and adenocarcinoma.
Barium swallow shows gastric folds protruding above the diaphragm
This is the classic presentation of biliary colic secondary to gallstones. Ingestion of a large
(usually fatty) meal stimulates gallbladder contraction. The intra-gallbladder pressure increases
on gallbladder contraction against an obstructed cystic duct and causes pain. Subsequent
gallbladder relaxation allows the stone to fall back from the duct with subsequent pain
resolution. The pain is in the RUQ or epigastric region, often constant (rather than colicky), and
accompanied by nausea, vomiting, and right-sided shoulder or subscapular discomfort (referred
pain). Patients may have recurrent symptoms that resolve between episodes. Features that
distinguish biliary colic from cholecystitis are pain resolution within 4-6 hours and absence of
abdominal tenderness, fever, and leukocytosis.
In acute cholecystitis, the gallbladder mucosa is inflamed. The cystic duct is obstructed with
gallstones or sludge. Although the pain is similar to biliary colic pain in location, radiation, and
associated symptoms, it usually lasts longer than 6 hours. Furthermore, patients frequently have
fever, leukocytosis, and tenderness to palpation in the midaxillary line under the right costal
margin (Murphy's sign)
Bowel ischemia from vascular obstruction causes severe abdominal pain out of proportion to
findings on physical exam. The pain is periumbilical, lasts longer than that of biliary colic, does
not resolve spontaneously, and is frequently accompanied by nausea, vomiting, and fecal blood
Lactose intolerance is characterized by a positive hydrogen breath test, positive stool test for
reducing substances, low stool pH and increased stool osmotic gap. There is no steatorrhea.
Pancreatic carcinoma presents with a dull upper abdominal pain that radiates to the back, weight
loss, or jaundice. Tumors in the pancreatic body or tail present with pain and weight loss while
tumors in the pancreatic head present with steatorrhea, weight loss, and jaundice. Physical
examination is often unremarkable aside from jaundice, with an abdominal mass or ascites
present in only 20% of patients. Classic findings include a nontender but palpable gallbladder at
the right costal margin in a jaundiced patient (Courvoisier's sign) or left supraclavicular
adenopathy (Virchow's node) in a patient with metastatic disease. Labs show increased serum
bilirubin and ALP with a mild anemia. Abdominal ultrasound is the initial imaging performed on
patients with jaundice. When the ultrasound is nondiagnostic, the next step is to obtain an
abdominal CT scan which detects bile and pancreatic duct dilation, mass lesions within the
pancreas, and indications of extrahepatic spread (eg, metastases or ascites).
ERCP is an excellent tool in the diagnosis of pancreatic cancer, with a sensitivity and specificity
of 90-95%. Because it is an invasive procedure, it is reserved for patients who have already
undergone a nondiagnostic ultrasound and a nondiagnostic CT scan.
Most (60%-70%) of pancreatic cancers occur in the head of the pancreas compressing the
pancreatic duct and common bile duct (leading to painless jaundice), sometimes seen on imaging
as the double duct sign. Subsequent backup of bile leads to intra- and extrahepatic biliary duct
dilation and a nontender, distended gallbladder at the right costal margin (Courvoisier sign). The
jaundice can also lead to pruritus, pale stools, and dark urine. In contrast, cancers in the body or
tail of the pancreas usually present with abdominal pain but without jaundice
Tumors in the head of the pancreas can present with weight loss, jaundice, and a nontender,
distended gallbladder on examination. Characteristic findings on imaging include intra- and
extrahepatic biliary tract dilation
Patients with ulcerative colitis have an increased risk of colorectal cancer (CRC), and the risk is
proportionate to the duration and extent of disease. CRC risk is also increased in patients with
Crohn’s disease involving the colon (Crohn colitis). CRC screening with colonoscopy and
mucosal sampling should be offered to patients with UC beginning 8 years after the initial
diagnosis (disease limited to the rectum and left colon may begin 12-15 years post diagnosis).
Repeat colonoscopy should be performed every 1-2 years thereafter. Colonic dysplasia is
associated with progression to adenocarcinoma, and prophylactic colectomy is advised if
dysplasia is identified. A screening colonoscopy interval of 5 years is appropriate for patients
without IBD who are found to have 1 or 2 small adenomatous polyps. Five years is also an
appropriate interval for patients with a first-degree family history of CRC or adenomatous polyps
This patient has several features of cholestasis (impaired biliary flow), including fatigue,
pruritus, and elevated ALP. A RUQ ultrasound distinguishes intrahepatic (no biliary tract
dilation) from extrahepatic (biliary tract dilation due to gallstones) cholestasis. If ultrasound
suggests intrahepatic cholestasis (as with this patient), the next step is to obtain serum anti-
mitochondrial antibody titers, which have high sensitivity and specificity for primary biliary
cholangitis (previously primary biliary cirrhosis). PBC is a chronic liver disease characterized by
autoimmune destruction of the intrahepatic bile ducts with resulting cholestasis. It presents most
commonly in middle-aged women and is insidious in onset. As the disease progresses, jaundice,
hepatomegaly, steatorrhea, and portal HTN may develop. Additional complications include
severe hyperlipidemia (with xanthelasma) and metabolic bone disease.
Autoimmune hepatitis is associated with elevated titers of antinuclear antibodies and anti-smooth
muscle antibodies. It is characterized by fluctuating hepatocellular injury (ie, elevated
transaminases) rather than cholestasis. First-line treatment includes oral glucocorticoids
Primary biliary cholangitis is a chronic liver disease characterized by intrahepatic cholestasis due
to autoimmune destruction of small bile ducts. It presents in middle-aged women with fatigue,
pruritus, hepatomegaly, and elevated ALP. The diagnosis is confirmed with serum anti-
mitochondrial antibody titers
Metastatic disease is the MCC of liver mass and is more common than primary liver cancer.
Undiagnosed CRC is likely in this patient with a solitary liver mass, probable iron deficiency
anemia (low MCV), and positive fecal occult blood screen. GI malignancies, such as colorectal
or pancreatic cancer, are the most frequent source of liver metastases as their venous drainage is
through the portal system directly to the liver. Lung, breast, and skin cancers (melanoma) often
also spread to the liver. The liver is a common site of metastatic disease due to its dual blood
supply (systemic and portal) and hepatic sinusoidal fenestrations allowing for easier metastatic
deposition. Liver metastases are often clinically silent unless pressure on the liver capsule or
obstruction of the biliary tree causes pain or jaundice. Multiple hepatic nodules are typically seen
in metastatic disease; however, solitary lesions are not uncommon. If a primary tumor is
identified (eg, with colonoscopy), liver biopsy is usually not needed.
HCC typically emerges from a chronically inflamed liver (chronic hepatitis B or C infection). It
is the MC type of primary liver cancer but is less common cause of hepatic malignancy than
metastasis from another source. Microcytic anemia and a positive fecal occult blood screen make
CRC more likely in this patient.
Hepatic angiosarcoma is a rare liver neoplasm more common in older men who have been
exposed to toxins (eg, vinyl chloride gas, inorganic arsenic compounds, thorium dioxide).
Cholangiocarcinoma (bile duct cancer): can be both intrahepatic and extrahepatic. It presents
with symptoms of biliary obstruction - jaundice, pruritus, light-colored stools, and dark urine.
The main risk factor is a history of primary sclerosing cholangitis
Focal nodular hyperplasia (FNH) and hepatic adenoma are benign liver tumors seen in young
women. FNH is typically asymptomatic; hepatic adenoma may cause RUQ pain and is a/w OCP.
Erythema Nodosum is inflammation of fat cells under the skin that results in reddish, painful,
tender lumps most commonly located in the front of the legs below the knees
This patient with chronic abdominal pain, diarrhea, weight loss, and inflammation (ESR of
48) likely has Crohn’s disease. Other GI manifestations include microscopic bleeding, fistula
and stricture formation, abdominal abscesses, and malabsorption. It can involve any part of
the GI tract from mouth (eg, aphthous ulcers) to anus. Common extraintestinal symptoms
include arthritis, uveitis, scleritis, and erythema nodosum. Laboratory findings are similar to
other chronic inflammatory diseases and can include leukocytosis, anemia, reactive
thrombocytosis, and elevated inflammatory markers (eg, ESR). Diagnosis is confirmed with
endoscopic or radiographic studies, and treatment involves immunosuppressive therapy.
Infectious colitis due to a number of bacteria (including Yersinia), parasites, or amoebae can
closely mimic Crohn disease, especially if it presents with ileitis. However, infectious colitis is a
more acute process and does not involve the oral mucosa. Ulcerative colitis more frequently
presents as bloody (rather than nonbloody) diarrhea with tenesmus and incontinence, and does
not involve the oral mucosa.
Irritable bowel syndrome and lactose intolerance are not commonly associated with weight loss,
anemia, leukocytosis, or elevated inflammatory markers. Such findings should prompt evaluation
for other causes of GI complaints
Chronic giardiasis presents with loose, greasy stools, significant weight loss, and abdominal
cramping and patients have clear risk factors for infection (eg, international travel, hiking)
Toxic megacolon typically presents with total or segmental nonobstructive colonic dilation,
severe bloody diarrhea, and systemic findings (eg, fever, tachycardia). IBD patients are at the
highest risk of developing toxic megacolon. Other causes of toxic megacolon include ischemic
colitis, volvulus, diverticulitis, infections (eg, C.difficile), and obstructive colon cancer (less
common). Diagnosis is confirmed by plain abdominal x-rays and 2 of 3 of the following: fever
>38C (100.4F), pulse >120/min, WBC >10,500, and anemia. Plain films reveal dilated right or
transverse colon (>6cm), multiple air-fluid levels, and thick haustral markings that do not extend
across the entire lumen. Toxic megacolon is medical emergency that can result in colonic
perforation. Treatment includes IV fluids, broad-spectrum antibiotics, and bowel rest. IV
corticosteroids are preferred for treating IBD-induced toxic megacolon. Emergency surgery
(subtotal colectomy with end-ileostomy is the procedure of choice) if the colitis does not resolve.
This patient has no risk factors (aortoiliac surgery, cardiopulmonary bypass, MI, hemodialysis)
for ischemic colitis. Even if the patient had these risk factors, her presentation would be acute
rather than after 2 months of symptoms
Pseudomembranous colitis is usually due to Clostridium difficile that is most commonly due to
recent antibiotic use (eg, clindamycin, fluoroquinolones). Patients typically develop diarrhea,
abdominal pain, and findings of colitis in the sigmoid/rectal area. This patient’s absence of recent
antibiotic use makes this less likely.
Colon cancer can cause painless chronic bleeding. However, a cancer capable of causing gross
bleeding (as seen in this patient) is unlikely to have been missed on colonoscopy and would
likely have led to microcytic anemia (with MCV <80 ). It is more likely that angiodysplasia
rather than colon cancer would be missed on colonoscopy
Diverticulosis is also unlikely to have been missed on colonoscopy. In addition, bleeding from
diverticula is frequently arterial, and typically results in passage of bright red blood. Maroon-
colored stools are more characteristic of right colonic angiodysplasia
Hemorrhoids cause bright red rectal bleeding, with blood on the surface of the stool or dripping
into the toilet. They are usually apparent on rectal examination or during colonoscopy.
Ischemic colitis usually presents with sudden onset of abdominal pain and tenderness followed
by rectal bleeding or bloody diarrhea within 24 hours. This patient has no abdominal pain
This patient's steatorrhea is most likely due to chronic pancreatitis related to long-standing
alcohol abuse. His previous hospitalizations for vomiting and epigastric pain that radiates to the
back raise suspicion for recurrent episodes of acute alcoholic pancreatitis. Repeated pancreatic
injury causes progressive inflammation and fibrosis and results in impaired endocrine and
exocrine (loss of digestive enzymes) functions. Patients with chronic pancreatitis may initially be
asymptomatic, but disease progression ultimately leads to malabsorption, steatorrhea, and
postprandial epigastric pain that can become continuous. Symptomatic improvement can be
achieved with alcohol cessation and pancreatic enzyme supplementation
Biliary colic (gallbladder attack or gallstone attack) is when a colic (sudden pain) occurs in the
RUQ due to a gallstone temporarily blocking the cystic duct that lasts one to a few hrs
Cholecystectomy and stone removal should be considered in patients with biliary colic which
presents with postprandial RUQ/epigastric pain associated with nausea/vomiting. Steatorrhea and
malabsorption are not characteristic.
In healthy people an INR of 1.1 or below is considered normal. An INR of 2.0 to 3.0 is an
effective therapeutic range for people taking warfarin for disorders such as atrial fibrillation or a
blood clot in the leg or lung
This patient has upper GI bleeding with ongoing hematemesis. Given her history of cirrhosis and
variceal band ligation, esophageal variceal hemorrhage is the most likely cause. She is
developing hemodynamic instability and a depressed level of consciousness and could rapidly
decompensate. Initial evaluation should begin with the ABCs. This patient's hematemesis with a
depressed level of consciousness is a set up for aspiration. Endotracheal intubation would be
indicated at this time to secure her airway. Two large bore IVs have already been placed to begin
aggressive fluid resuscitation, and a type and screen should be performed since the patient will
need a blood transfusion. Platelet transfusion is not indicated until levels fall < 50,000. Bleeding
may be exacerbated by this patient's elevated INR (1.9) for which FFP could be considered, but it
is often of minimal effectiveness in cirrhotic patients receiving aggressive fluid resuscitation.
Prophylactic antibiotics, and somatostatin analog (octreotide) should also be considered.
Upper endoscopy should be done in an attempt to stop this patient's variceal hemorrhage, but this
should be done after the patient is stabilized and intubated.
Patients with upper GI bleeding who have a depressed level of consciousness and ongoing
hematemesis should be intubated to protect the airway as a part of initial stabilization and
resuscitation. Prompt endoscopic treatment (upper endoscopy) with band ligation or
sclerotherapy should then be performed to stop the bleeding
The classic triad of RCC is hematuria, abdominal mass, and flank pain. On abdominal CT, RCC
presents as a lesion within the kidney parenchyma that enhances with contrast administration.
Mesenteric ischemia presents with abdominal pain out of proportion to physical exam
findings. It is most common in elderly patients, and may result from a cardiac embolus (AF),
arterial thrombosis (atherosclerosis), or other etiology (shock or vasoconstriction). CT findings
include bowel wall thickening, pneumatosis intestinalis (gas), and mesenteric thrombi.
This patient with multiple peptic ulcers and diarrhea has clinical features of Zollinger-Ellison
syndrome (ZES). ZES is due to a gastrin-producing tumor in the pancreas or duodenum.
Uncontrolled gastrin secretion leads to parietal cell hyperplasia, with excessive production of
gastric acid. Multiple duodenal (sometimes jejunal) ulcers are typical that are refractory to
standard acid-reducing meds. The excess gastric acid in the small intestine causes diarrhea and
steatorrhea due to inactivation of pancreatic enzymes and injury to the mucosal brush border.
ZES is diagnosed by a markedly elevated serum gastrin level (>1000 pg/ml) in the presence of
normal gastric acid pH (<4). Endoscopy reveals ulcers (>90% of patients with ZES) and
occasionally will identify a primary duodenal gastrinoma. CT, MRI, and somatostatin receptor
scintigraphy can identify pancreatic tumors and metastatic disease. If gastrinoma is confirmed,
patients should be screened for MEN1 with assays for PTH, ionized calcium, and prolactin
Bile acids are primarily reabsorbed in the ileum; impaired absorption can cause malabsorption of
fats. This is typically seen following extensive ileal resection in Crohn disease (CD), which
presents with chronic or recurrent abdominal pain, fever, diarrhea, and weight loss.
Abdominal succussion splash is elicited by placing the stethoscope over the upper abdomen and
rocking the patient back and forth at the hips. Retained gastric material >3 hours after a meal will
generate a splash sound, indicating the presence of a hollow viscus filled with fluid and gas.
Although the test has modest sensitivity and specificity for diagnosing gastric outlet obstruction
(GOO), it may suggest the need for more definitive evaluations. Initial management includes NG
suctioning to decompress the stomach, IV hydration, and endoscopy for definitive diagnosis
AST and ALT are distributed widely throughout the body, with AST present and very active in
the liver, heart, kidney, and skeletal muscle. While ALT is also present in reduced quantities in
the kidney, heart, and skeletal muscle, it is predominantly found in the liver and is therefore more
specific for hepatocyte injury. The patient has a mild (< 250 U/ L), asymptomatic elevation of
serum transaminases. The first step in the evaluation should be a careful screening for all
hepatitis risk factors, including drug and alcohol intake, travel outside the USA, blood
transfusions, or high-risk sexual practices. The next step in the evaluation process is to repeat the
liver function tests. If the transaminases remain elevated over a six-month period, they are
categorized as chronic. Testing for viral hepatitis B and C, hemochromatosis, and fatty liver is
done to further evaluate chronically elevated transaminases. If these tests prove unremarkable
then search for muscle disorders (polymyositis) and thyroid disease
Esophageal cancer should be strongly suspected in this patient given his age, persistent burning
chest pain, probable dysphagia (carefully chewing his food before swallowing), and significant
weight loss. Most esophageal malignancies are either adenocarcinoma or SCC. Adenocarcinoma
usually arises within an area of Barrett esophagus near the GE junction; risk is increased with
smoking and GERD. SCC can occur anywhere in the esophagus; risk is increased with smoking
and heavy alcohol consumption. Definitive diagnosis of esophageal cancer requires esophageal
endoscopy with biopsy. Young, low-risk patients with undetermined esophageal symptoms may
start with barium esophagram, but those who are age >55 as well as those with alarm symptoms
(eg, weight loss, gross or occult bleeding, early satiety) should proceed directly to endoscopy.
Subsequent staging procedures may include CT and PET scan, and surgery for definitive cure for
patients with limited-stage disease.
Bronchoscopy is indicated for evaluation of accessible pulmonary masses or for patients with
additional symptoms (eg, hemoptysis) of an endobronchial lesion. This patient has a normal
chest x-ray and possible dysphagia, making esophageal cancer more likely than lung malignancy
Risk factors for trace mineral deficiency include malabsorption, bowel resection, poor nutritional
intake, and dependence on parenteral nutrition. Zinc is obtained from meat, nuts, and fortified
cereal. It is an essential component of numerous enzymes in the body and plays an important role
in gene transcription and cell division. Zinc deficiency in adults causes hypogonadism, impaired
wound healing, impaired taste, and immune dysfunction. Physical findings include alopecia as
well as a skin rash (erythematous pustules around body orifices (mouth) and on the extremities).
Porcelain gallbladder (calcium-laden gallbladder wall with bluish color and brittle consistency)
is associated with chronic cholecystitis. The pathogenesis of the condition remains unclear, but it
is thought that calcium salts are deposited intramurally due to the natural progression of chronic
inflammation/irritation from gallstones. Patients can be asymptomatic, have RUQ pain, or have a
firm and nontender RUQ mass on examination. Plain x-rays show a rimlike calcification in the
area of the gallbladder, and CT scan reveals a calcified rim in the gallbladder wall with a central
bile-filled dark area. Porcelain gallbladder has been associated with increased risk for gallbladder
adenocarcinoma. Cholecystectomy is considered for patients with porcelain gallbladder,
particularly if they are symptomatic or have incomplete mural calcification.
This patient's clinical findings - recurrent peptic ulcer disease with multiple ulcers and jejunal
ulceration - suggest gastrinoma (ZES). Gastrinomas usually occur in patients age 20-50 with
dyspepsia, reflux symptoms, abdominal pain, weight loss, diarrhea, or frank GI bleeding.
Endoscopy shows thickened gastric folds, multiple peptic ulcers, refractory ulcers despite PPI
use, or ulcers in the jejunum (suggesting excess gastric acid that cannot be fully neutralized in
the duodenum). Fasting serum gastrin level (off PPI therapy for 1 week) should be checked in
suspected gastrinoma; a level <110 rules it out, and a level >1000 is diagnostic. If gastrin is
elevated, gastric pH should be measured because gastrin may also be elevated due to failure of
gastric acid secretion (achlorhydria). A gastrin level of 110-1000 is non-diagnostic and requires a
follow-up secretin stimulation test. Secretin stimulates the release of gastrin by gastrinoma cells.
Normal gastric G cells are inhibited by secretin; therefore, secretin administration should not
cause a rise in serum gastrin concentrations in patients with other causes of hypergastrinemia
Gastrinoma (ZES) should be suspected in patients with multiple stomach ulcers and thickened
gastric folds on endoscopy. It is diagnosed by fasting serum gastrin level > 1000. Patients with
non-diagnostic serum gastrin levels should be evaluated with a secretin stimulation test
Triple therapy (PPI, amoxicillin, and clarithromycin) treats H. pylori infection, which can cause
gastric and duodenal ulcers. However, this patient's diarrhea and multiple ulcers in different sites
are more suggestive of gastrinoma.
This patient has minimal bright red blood per rectum (BRBPR), small amounts of bright red
blood on toilet paper after wiping, a few drops of blood in the toilet bowl after defecation, or
small amounts of blood on the surface of the stool is consistent with hemorrhoids or rectal
fissures. However, more serious disorders (eg, proctitis, rectal ulcers, colorectal polyps, cancer)
are possible. The evaluation of BRBPR depends on the patient's presentation and risk factors.
Clinical factors associated with increased risk of serious disease include blood mixed with stool,
systemic symptoms (eg, fever, weight loss), diarrhea, anemia, change in bowel habits, and
abdominal pain. Age also correlates strongly with risk of malignancy. Patients age >50 are at
elevated risk for colorectal cancer and should undergo colonoscopy unless they have had a
normal colonoscopy within the last 2-3 years. Patients age 40-49 are at intermediate risk and
could be considered for sigmoidoscopy as an alternative to colonoscopy. If the patient is age <40
and has no other risk factors for colon cancer, office-based anoscopy or proctoscopy should be
performed first. If no etiology is found, colonoscopy or sigmoidoscopy is considered. Anoscopy
is also useful in older patients to visualize a palpable abnormality found on physical exam.
Esophageal motility studies and upper GI endoscopy are used for evaluating esophageal
dysphagia, which presents with a sensation of food getting stuck in the esophagus (not throat) a
few seconds after a swallow but does not cause difficulty initiating swallowing. This patient's
findings are more concerning for oropharyngeal dysphagia (His wife reports that he coughs,
chokes, and has nasal regurgitation when swallowing liquids or solids). The patient admits the
food "gets stuck" in his throat when swallowing. He has a history of HTN, type 2 diabetes,
ischemic stroke, chronic kidney disease, coronary artery disease, and advanced dementia.)
Antimitochondrial antibodies are elevated in primary biliary cholangitis. These patients also have
pruritus, fatigue, jaundice along with abdominal pain. Although PBC can present with lab similar
to those seen in this patient, imaging typically does not show a dilated common bile duct
Liver biopsy should be done when initial evaluation (laboratory analysis and imaging) cannot
provide a definitive diagnosis or determine the severity of liver disease. This patient may require
liver biopsy if ERCP and other laboratory tests for hepatitis are unremarkable.
Ursodeoxycholic acid can treat cholesterol gallstones in patients with mild symptoms who are
not candidates for cholecystectomy. It is also used to treat primary biliary cholangitis and
primary sclerosing cholangitis
Primary biliary cholangitis (PBC), formerly known as primary biliary cirrhosis, is a chronic liver
disease resulting from progressive destruction of the bile ducts in the liver. Bile produced in the
liver travels via these ducts to the small intestine where it aids in the digestion of fat and fat-
soluble vitamins (A, D, E and K). When the ducts are destroyed, bile builds up in the liver
contributing to inflammation and scarring (fibrosis) eventually leading to cirrhosis
Primary sclerosing cholangitis is a rare, chronic cholestatic liver disease characterized by
intrahepatic, extrahepatic stricturing with bile duct fibrosis. Inflammation and fibrosis of bile
ducts and the liver cause impaired bile formation or flow and progressive liver dysfunction.
Primary biliary cholangitis (PBC) and primary sclerosing cholangitis (PSC) are two major types
of chronic cholestatic liver disease. Cholestasis results from impaired bile formation and/or flow
and may manifest clinically as fatigue, pruritus, and jaundice. Early biochemical markers of
cholestasis include elevated levels of serum ALP and GGT with conjugated hyperbilirubinemia
in more advanced stages. Chronic cholestasis is generally defined by persistent abnormalities
lasting >6 months. Clinical findings include fatigue, pruritus, and fat-soluble vitamin deficiency.
HBsAg and anti-HBc are diagnostic tests for acute hepatitis B infection as they are both elevated
during initial infection and anti-HBc will remain elevated during the window period
HBcAg is not detectable in serum and is therefore not useful for establishing the diagnosis
HBeAg is a good indicator of infectivity but is a poor test for acute hepatitis B infection as levels
typically fall early in the course of the disease.
Testing for HBsAg and anti-HBs can miss the diagnosis of acute hepatitis B infection in patients
who are in the window period
Although hepatitis B virus DNA may be detectable prior to the appearance of HBsAg or HBeAg,
this test is not performed to diagnose acute infection. It is obtained in patients with chronic
hepatitis B to determine candidacy for antiviral therapy or monitor response to treatment.
Crohn disease can involve any component of the GI tract from the mouth to the anus and
characteristically has skip areas of involvement. Oral ulcers, anal fissures, skin tags and fistulas
are commonly seen. Extraintestinal manifestations include arthritis, skin lesions, and lung
disease. Diagnosis can be confirmed with endoscopy and shows strictures. Biopsy shows focal
ulcerations with transmural inflammation. Granulomas are also common. Treatment involves 5-
ASA drugs for patients without systemic symptoms and corticosteroids or biologic agents (eg,
infliximab) for patients with more severe disease.
Tuberculous enteritis should be included in the differential of CD as it may present similarly with
abdominal pain, diarrhea, strictures, and fistula. However, patients classically have fever, night
sweats, and weight loss, and oral ulcers would be unusual
Ulcerative colitis is associated with diffuse colitis with no skip lesions and presents with colicky
abdominal pain and inflammatory diarrhea (eg, blood, mucus). However, GI involvement is
limited to the colon, and both oral and perianal disease would be unusual.
Ulcerative colitis presents with persistent diarrhea and gross rectal bleeding, often with systemic
symptoms such as weight loss and fever. UC involves the rectosigmoid, and severe cases can
have continuous involvement of the entire colon. Colonoscopy shows erythematous, friable
mucosa, with biopsy characterized by mucosal inflammation and crypt abscesses. Crohn disease
limited to the colon (Crohn colitis) can present similarly but can be differentiated from UC by
the presence of skip lesions (discontinuous areas of inflammation), noncaseating granulomas,
and transmural inflammation. Common extracolonic manifestations of UC include erythema
nodosum and pyoderma gangrenosum in the skin, episcleritis, spondyloarthritis, and sclerosing
cholangitis. In addition, the incidence of CRC is increased in patients with UC, with the risk
proportionate to the duration and extent of disease. Colonoscopy is advised in patients with UC
even in the absence of symptoms (beginning 8-10 years after the initial diagnosis)
Patients with UC are at increased risk for primary sclerosing cholangitis (PSC). An elevated
serum ALP level in a patient with UC should raise suspicion for PSC, but routine surveillance is
not recommended. Toxic megacolon is a severe complication of UC that can lead to peritonitis
and death, but no form of regular surveillance has been shown to prevent this complication.
Uveitis is an extraintestinal manifestation of UC but does not necessitate regular surveillance.
Acute erosive gastritis is characterized by severe hemorrhagic erosive lesions after exposure of
the gastric mucosa to various injurious agents or after a substantial reduction in blood flow.
Aspirin decreases the protective prostaglandin production. In addition, aspirin and alcohol cause
direct mucosal injury. Acute mucosal injury decreases the normal protective barriers (decreased
levels of secreted mucins and bicarbonate, and decreased integrity of the epithelium), thus
permitting acid and other luminal substances (e.g , proteases and bile acids) to penetrate into the
lamina propria, thereby causing additional injury to the vasculature and subsequent hemorrhage
As in this case, patients may manifest with hematemesis and abdominal pain.
Pellagra is due to niacin deficiency and is characterized by the "3 Ds": dermatitis, diarrhea, and
dementia: • Dermatitis is on sun-exposed areas of the body and is characterized by rough,
hyperpigmented, scaly skin. • Diarrhea is often associated with abdominal pain, nausea, and loss
of appetite. • Dementia is due to neuronal degeneration in the brain and spinal cord and can lead
to memory loss, affective symptoms (eg, depressed mood in this patient), and psychosis. Niacin
deficiency in developing countries is seen in populations that subsist primarily on corn products
(niacin in corn occurs in a bound, unabsorbable form). In developed countries, it is seen in
patients with impaired nutritional intake (eg, alcoholism, chronic illness). Pellagra can also be
seen in those with carcinoid syndrome (due to depletion of tryptophan) or Hartnup disease
(congenital disorder of tryptophan absorption). Prolonged isoniazid therapy can interfere with
metabolism of tryptophan and occasionally lead to pellagra.
Acute intermittent porphyria (AIP) causes abdominal pain, vomiting, and diarrhea, often with
neurologic symptoms (eg, agitation, paresthesias, confusion). Although AIP may be triggered by
isoniazid, the symptoms are episodic rather than chronic
Conjugated hyperbilirubinemia with predominantly elevated AST and ALT: Viral hepatitis,
Autoimmune hepatitis, Toxin/drug-related hepatitis, Hemochromatosis, Ischemic hepatitis,
Alcoholic hepatitis
Conjugated hyperbilirubminemia with Normal AST, ALT, and Alkaline phosphatase: Dubin
johnson, Rotor’s syndrome
Conjugatd hyperbilirubinemia with normal transaminases (AST, ALT) and ALP suggest inherited
bilirubin metabolism disorders (eg, Dubin-Johnson syndrome). Conjugated hyperbilirubinemia
with predominantly elevated transaminases with normal ALP favors intrinsic liver disease (eg,
viral hepatitis, hemochromatosis). Conjugated hyperbilirubinemia with elevated ALP out of
proportion to the transaminases suggests intrahepatic cholestasis or biliary obstruction. This
patient's conjugated hyperbilirubinemia, elevated ALP, painless jaundice, and systemic
symptoms (fatigue, weight loss) suggest malignant obstruction of the biliary system likely due to
pancreatic adenocarcinoma or cholangiocarcinoma. The next step should be abdominal imaging
(US or CT), with consideration for ERCP if initial imaging is nondiagnostic
Acute choledocholithiasis (gallstone in common bile duct) can present similarly, with conjugated
hyperbilirubinemia and markedly elevated ALP. However, patients typically have acute-onset
RUQ or epigastric pain rather than the painless jaundice seen in this patient. Pancreatic cancer
causes epigastric pain in the majority of patients but compared to acute choledocholithiasis, the
pain is more chronic and typically worsens in the supine position and at night
Intrinsic liver disorders (eg, sarcoidosis, hemochromatosis, hepatic malignancy) usually cause
hepatocellular injury with predominantly elevated transaminases. Hemochromatosis is also
associated with DM, erectile dysfunction, skin pigmentation, arthropathy, increased risk of
cirrhosis and HCC. However, bilirubin and ALP levels are usually normal or mildly elevated
without clinically obvious jaundice
Primary biliary cholangitis may present with fatigue, itching, jaundice, weight loss, and
elevated ALP levels. However, antimitochondrial antibodies would be positive
A 52-year-old woman comes to the physician with symptoms of a "pounding" sensation in her
neck and lower extremity edema. She has lost weight recently. Her other medical problems
include migraine disorder, chronic diarrhea, and severe post-menopausal flushes. The patient has
a 30-pack-year smoking history. Physical examination shows poor oral hygiene. A 2/6 murmur is
heard at the lower sternal border that increases with inspiration. Echocardiography shows
retracted and immobile tricuspid valve leaflets with poor coaptation and severe tricuspid
insufficiency. Which of the following is the most likely diagnosis?
This patient's presentation- episodic pounding sensation (due to flushing and associated rise in
pulse rate), chronic diarrhea, weight loss, and valvular heart disease with tricuspid regurgitation
is consistent with carcinoid syndrome. Carcinoids are well-differentiated neuroendocrine tumors
found in the distal small intestine, proximal colon, and lung, with a strong propensity for
metastasis to liver. These tumors secrete histamine, serotonin, and VIP that are metabolized in
the liver. Episodic flushing is the hallmark of carcinoid syndrome and occurs in 85% of patients.
Other common features include cutaneous telangiectasias, bronchospasm, and tricuspid
regurgitation. Pathognomonic plaque-like deposits of fibrous tissue occur on the right heart,
leading to TR and right-sided heart failure. Elevated 24-hour urinary 5HIAA is diagnosistic
Right-sided infective endocarditis (IE) is associated with IV drug use. Patients present with
constitutional symptoms (fever, malaise, and arthralgias), vascular or immunologic phenomena,
and a new valvular regurgitant murmur. Flushing and chronic diarrhea are not symptoms of IE.
Myxomatous valve disease occurs due to weakening of the connective tissue that commonly
affects the mitral valve, resulting in MVP. The tricuspid valve may be affected by myxomatous
degeneration, symptoms of flushing and diarrhea are more suggestive of carcinoid syndrome
Systemic lupus erythematosus (SLE) typically has skin (butterfly rash and photosensitivity),
musculoskeletal (arthralgias), hematologic (cytopenias), cardiac (pericarditis), renal, and
neurologic manifestations (cognitive defects, seizures). However, isolated TR is not common
This asymptomatic patient has elevated ALP with normal ALT and AST, normal RUQ US, and
positive antimitochondrial antibody assay suggesting primary biliary cholangitis. It is a chronic
progressive liver disease characterized by cholestasis with AI destruction of intrahepatic bile
ducts. Pruritus and fatigue are the first symptoms but as the disease progresses, jaundice,
steatorrhea, hepatomegaly, eyelid xanthelasma, portal hypertension, and osteopenia may develop.
Ursodeoxycholic acid (UDCA) is the DOC in PBC. UDCA is a hydrophilic bile acid that
decreases biliary injury by the more hydrophobic endogenous bile acids. It also increases biliary
secretion and has anti-inflammatory and immunomodulatory effects. UDCA delays histologic
progression and may improve symptoms and possibly survival. It should be initiated as soon as
the diagnosis is made, even in asymptomatic patients. Treatment is less effective in advanced
disease, and many patients will go on to require liver transplantation.
Liver transplantation is the definitive cure for progressive PBC but is indicated only in those
with severe liver damage or cirrhosis. As this patient has no signs or symptoms of cirrhosis, a
trial of UDCA is more appropriate
Glucocorticoids are used for auto immune hepatitis, which is characterized by elevated liver
transaminases and a positive antinuclear antibody titer. Glucocorticoids are ineffective in PBC.
Statins cause a hepatocellular rather than cholestatic pattern of injury, characterized by elevated
AST & ALT. This patient has normal transaminases and an elevated ALP level
This patient has typical features of giardiasis, including diarrhea acquired during international
travel, abdominal cramps, foul-smelling stools, bloating, and benign abdominal exam. Giardia
duodenalis (AKA G Iamblia/G intestinalis) is common in developing countries with incubation
period of 1-2 weeks. Commonly transmitted by contaminated water but can be foodborne or
transmitted person-to-person via a fecal-oral route. Most patients are asymptomatic; however, a
minority develop chronic giardiasis characterized by malabsorption, weight loss, or persistent GI
distress. The preferred confirmatory test is a stool antigen assay (direct immunofluorescence or
ELISA). Stool microscopy for oocysts and trophozoites is useful in resource-poor settings or if
other parasitic organisms are suspected. Some facilities also offer a nucleic acid amplification
assay. Metronidazole is the preferred treatment. Asymptomatic carriers do not need treatment
A short course of ciprofloxacin is advised for empiric treatment of traveler's diarrhea (most
commonly due to Escherichia coli). However, the preferred treatment for giardiasis is
metronidazole. Diagnostic confirmation should be obtained prior to additional antibiotic therapy
in this patient with a prolonged course of diarrhea who has failed ciprofloxacin
Clostridium difficile causes antibiotic-associated colitis. This patient was symptomatic prior to
receiving antibiotics, so C difficile would be less likely
Glucocorticoids are not indicated in ALF as they increase the risk of infection and have not
demonstrated benefit in most etiologies of ALF. They may provide benefit in alcoholic hepatitis
and in ALF due to autoimmune hepatitis
Liver transplantation should be considered in all patients with ALF and indications that the
disease is worsening or failing to improve
This patient’s elevated LFT and liver biopsy result (panlobular mononuclear infiltration and
hepatic cell necrosis) suggest acute hepatitis. Since this patient is on isoniazid, idiosyncratic liver
injury with a histological picture similar to viral hepatitis must be considered. While extrahepatic
symptoms like rash, arthralgias, fever, leukocytosis, and eosinophilia are common in patients
with drug-induced liver injury, they are absent in cases of isoniazid-induced liver injury. Drugs
and toxins cause hepatic injury either through direct toxic effects or through idiosyncratic
reactions. The direct toxic effects are dose-dependent and have short latent periods. Some
examples of direct toxins include carbon tetrachloride, acetaminophen, tetracycline, and
substances found in the Amanita phalloides mushroom. Idiosyncratic reactions are not dose-
dependent and have variable latent periods. Some examples of pharmacological agents that cause
idiosyncratic reactions include isoniazid, chlorpromazine, halothane, and antiretroviral therapy.
OCPs cause abnormalities in LFT without evidence of necrosis or fatty change. This patient's
liver biopsy demonstrated necrosis thus OCPs are not cause of her hepatic dysfunction
Autoimmune hepatitis should be considered once all possible causes of drug-induced or viral
hepatitis have been ruled out. It will show positive ANA and/or positive smooth muscle antibody
This patient, who has an extensive history of alcohol use, most likely has alcoholic hepatitis with
jaundice, anorexia, and tender hepatomegaly. There is no evidence of ascites or cirrhosis (eg,
spider angiomas, gynecomastia, asterixis). He has no weight loss or systemic symptoms to
suggest malignancy, and there are no signs of infection. Alcoholic liver disease is generally
characterized by modest elevations in AST and ALT usually <300 and almost always <500. A
ratio of AST to ALT >2 (thought to be due to hepatic deficiency of pyridoxal 5'-phosphate, an
ALT enzyme cofactor) is very common in alcoholic liver disease. There is no correlation
between the degree of elevation and liver disease severity. Elevations in GGT, and in ferritin (an
acute phase reactant) would likely be seen in alcoholic liver disease.
If marked elevations (>25 times the upper limit) of AST and ALT are present, toxin-induced (eg,
acetaminophen), ischemic, or viral hepatitis should be suspected
Alcoholic hepatitis is generally characterized by a ratio of AST to ALT > 2, elevated GGT, and
elevated ferritin. The absolute values of AST and ALT are almost always < 500.
Hematology
Vitamin B12 deficiency should be suspected in strict vegetarians with anemia and neurologic
complications. Folic acid supplementation may correct the anemia in these patients, but it leads
to rapid progression of neurologic complications
Whole blood (for transfusions) is mixed with solutions containing citrate anticoagulant prior to
storage. Packed cells derived from these whole blood collections also contain citrate. Patients
who have received the equivalent of more than one blood volume of blood transfusions or
packed RBC over 24 hours may develop elevated plasma levels of citrate. Citrate chelates
calcium and magnesium and may reduce their plasma levels, causing paresthesias
The treatment for hereditary spherocytosis involves supportive care with oral folic acid and
blood transfusions during periods of extreme anemia. Splenectomy is considered if patients have
moderate to severe spherocytosis, or are refractory to medical management. Life-threatening
anemia and the need for regular transfusions may be abolished by splenectomy, although a mild
degree of anemia usually persists. The most feared long-term complication is overwhelming
sepsis with encapsulated bacteria, most commonly Streptococcus pneumoniae which is present
up to 30 years or longer after splenectomy. To reduce this risk administration of anti
pneumococcal, Haemophilus, and Meningococcal vaccines several weeks before the operation,
and daily oral penicillin prophylaxis for three to five years following splenectomy or until
adulthood (for pediatric patients). In view of reported deaths from sepsis up to 30 years or more
after splenectomy, a case can be made for lifetime penicillin prophylaxis. Alternatively,
antibiotics can be made available at home for immediate treatment of any significant fever.
Tumor lysis syndrome (TLS) can develop in patients with aggressive hematologic malignancies
who begin cytotoxic chemotherapy. Symptoms of TLS are due to electrolyte abnormalities and
include nausea, vomiting, diarrhea, muscle cramps, seizures, and tetany. Cardiac arrhythmias
(hyperkalemia, hypocalcemia) and acute kidney injury (renal tubule deposition of uric acid and
calcium phosphate) are common complications. The use of IV fluids to flush the kidneys and
uric acid metabolism inhibitors (allopurinol, rasburicase) helps lower the risk of uric acid
mediated renal damage; however, calcium phosphate-induced renal injury may still occur. The
following are often observed:
• Hyperuricemia - Nucleic acids are released and metabolized into uric acid.
• Hyperkalemia and hyperphosphatemia - Intracellular ions are liberated.
• Hypocalcemia - Phosphate binds and precipitates calcium, reducing intravascular levels.
This patient's male sex, family history of a bleeding disorder, and thigh hematoma after minor
trauma are concerning for hemophilia. Hemophilia is an X-linked recessive bleeding disorder
due to deficiency of factor VIII (hemophilia A) or factor IX (hemophilia B).The most frequent
sites of bleeding are the joints (80%), especially the knee. Hemarthrosis can occur with minimal
or no trauma. Hemorrhage into the skeletal muscle after minor trauma is also common. Lab
findings include a prolonged activated PTT with normal platelet count, bleeding time, and PT.
Treatment involves replacing the missing factor with a recombinant or purified product
Melena (black, tarry stool) is seen in upper GI bleeding; hematochezia (bright red stool) occurs
with lower GI bleeding. Although patients with hemophilia are at an increased risk of GI bleeds,
the bleeding usually occurs in the setting of esophagitis, gastritis, polyps, and diverticuli. In
addition, these issues are less common than hemarthrosis.
SOB, right leg swelling, and sinus tachycardia with an elevated D-dimer are concerning for a
pulmonary embolism (PE) that began as a DVT. The patient is young and has no obvious risk
factors (eg, immobility, recent travel, trauma), so a hypercoagulable disorder should be
considered. Testing is recommended in young patients (<45) with a first-time unprovoked DVT/
PE, patients with recurrent DVT/PE, and patients with unusual sites of thrombi (eg, cerebral,
mesentery, portal veins). The most common disorder is factor V Leiden (FVL). Most patients
with FVL have an AD point mutation in factor V gene that makes it unable to respond to
activated protein C, an innate anticoagulant. This mutation leads to slowed degradation of
procoagulant active factor V, leading to continued thrombin formation, and to slowed
degradation of active factor VIII.
Antiphospholipid antibodies (aPL Ab) cause venous and arterial thrombi in patients with
underlying conditions (eg, lupus) characterized by a prolonged activated PTT that fails to correct
on a mixing study due to the presence of aPL Ab acting as an inhibitor.
Antithrombin (AT) deficiency is acquired in the setting of DIC, cirrhosis, or nephrotic syndrome
Pulmonary HTN and polycythemia commonly co-exist. First, polycythemia can cause pulmonary
HTN by increasing blood viscosity within the pulmonary vasculature. Additionally, in patients
with chronic hypoxemia, both pulmonary HTN and polycythemia can result. However, AVMs
and pulmonary HTN are generally not related.
Competitive athletes using exogenous androgens, autologous blood transfusions, or EPO to
enhance athletic performance can develop an elevated Hct. Findings suggesting exogenous
androgens include gynecomastia, testicular atrophy, mood disturbances, and hepatotoxicity
An autologous blood transfusion involves removing blood several weeks before a competitive
event and then transfusing the blood back into the athlete prior to the event. The net effect is
increased hematocrit, oxygen-carrying capacity, and performance. However, this process
typically does not cause gynecomastia or mood changes, as seen in this patient.
Multiple myeloma (MM) is also a plasma cell neoplasm that produces a monoclonal antibody
spike by SPEP; however, patients with MM rarely (<0.5%) produce monoclonal IgM. MM is
typically characterized by monoclonal IgG, IgA, or light chains. In addition, patients with MM
tend to present with bone pain rather than hyperviscosity, neuropathy, and tissue infiltration.
Hodgkin lymphoma presents with painless lymphadenopathy and B symptoms (night sweats,
fever) with normal peripheral blood smear and CBC. Massive lymphocytosis is unlikely
A 55-year-old woman comes to the physician complaining of pain, itching, and red streaks on her
left arm. She had a similar episode on her chest almost 2 weeks ago that improved on its own.
Review of systems is positive for heartburn and mild upper abdominal pain for the last several
months that is partially relieved by antacids. The patient has a 15- to 30-pack-year smoking
history and drinks alcohol occasionally. Physical examination shows mild epigastric tenderness
to palpation, The lungs are clear to auscultation. There are no murmurs on cardiac examination.
She has tender, erythematous, and palpable cord-like veins on the left arm and upper chest.
Which of the following is the best next step in management of this patient?
D-dimer testing is useful in low-risk patients with suspected DVT and/or pulmonary embolism.
However, results are less useful in patients with signs of migratory superficial thrombophlebitis
and suspected malignancy.
Hypochromic/microcytic anemia with low serum iron and ferritin levels is typical for iron-
deficiency anemia. The MCC of iron-deficiency anemia is chronic blood loss. Iron-deficiency
anemia in an adult male or a post-menopausal woman should prompt the physician to look for GI
causes of blood loss. Tests for occult blood in the stool should be done first. It is not unusual for
right-sided colon cancer to manifest as iron-deficiency anemia. Other GI causes of chronic blood
loss include peptic ulcer disease, angiodysplasia, colonic diverticula, etc. Iron supplementation
restores iron reserves, but the identification of the cause of the problem is most important
Febrile nonhemolytic transfusion reaction occurs within 1-6 hours of transfusion. When red cells
and plasma are separated from whole blood, small amounts of residual plasma and/or leukocyte
debris remain in the red cell concentrate. During blood storage, these leukocytes release
cytokines, which when transfused cause transient fevers, chills, and malaise, without hemolysis.
Management includes stopping the transfusion to exclude other serious reactions, administering
antipyretics, and using leuko reduced blood products for future transfusions. Leukoreduction
involves reducing the number of transfused leukocytes through filtering or other methods such as
saline washing, freezing and deglycerolizing, or buffy coat removal. It also reduces the risk of
HLA alloimmunization and transmission of CMV (which typically resides in leukocytes).
Premedication with antipyretics and antihistamines does not reduce risk of transfusion reactions.
Careful cross-matching of blood prevents acute hemolytic reactions which present with fever,
chills, flank pain, and hemoglobinuria within an hour of transfusion which can progress to renal
failure and DIC. The direct antiglobulin test (Coombs) is positive and plasma free Hb is >25mg/
dl. Urinalysis also shows hemoglobinuria
Calcium gluconate infusion is used to avoid or treat severe hypocalcemia following massive
blood transfusion. Similarly, warming blood is recommended only during rapid massive
transfusion to prevent hypothermia
Whole blood contains numerous leukocytes and increases the risk of febrile reaction. Therefore,
it is rarely used except in cases of acute massive hemorrhage (eg, trauma)
Residual plasma in red cell concentrates contains proteins, including IgA. IgA-deficient patients
develop antibodies against IgA which react with IgA-containing donor products and induce an
anaphylactic reaction (angioedema, hypotension, and respiratory distress) that progress to loss of
consciousness, shock, and respiratory failure. Red cells should be washed to remove as much of
the plasma as possible for patients with IgA deficiency or prior allergic transfusion reaction.
Schistocytes or helmet cells are formed during mechanical destruction of erythrocytes in the
vasculature. Artificial (mechanical) valves are more traumatic for RBCs than porcine prostheses,
and frequently cause hemolysis. Schistocytes also occur due to narrowing of vascular spaces or
disseminated thrombosis. Hemolytic anemia causes increased indirect bilirubin and urinary
urobilinogen. Intravascular destruction results in free Hb in the serum (hemoglobinemia) and in
the urine (hemoglobinuria) as well as increased serum LDH. Haptoglobin is a serum protein that
binds free Hb and promotes its excretion by the reticuloendothelial system. In intravascular
hemolysis, the amount of free Hb in the serum exceeds the binding capacity of haptoglobin
thereby decreasing the level of haptoglobin
Schistocytes (helmet cells) are fragmented erythrocytes. They occur in MAHA (DIC, HUS, TTP)
and due to RBC destruction by prosthetic cardiac valves. Hemolytic anemias are characterized
by a decreased serum haptoglobin level as well as an increased LDH and indirect bilirubin.
This patient, with fatigue and dyspnea following a respiratory infection treated with amoxicillin,
has splenomegaly and anemia with reticulocytosis suggestive of warm agglutinin AIHA, possibly
due to antibiotic therapy. Causes of AIHA include lymphoproliferative disorders, viral infections,
autoimmune conditions, and drugs. Patients usually have normocytic anemia with evidence of
hemolysis, jaundice, elevated indirect bilirubin, increased serum LDH, decreased serum
haptoglobin. Splenomegaly due to erythrocyte entrapment commonly develops. The bone
marrow response appears as reticulocytosis. Peripheral smear typically shows spherocytes,
microspherocytes, elliptocytes, or increased numbers of polychromatophilic cells. Diagnosis of
warm AIHA is confirmed by a direct antiglobulin (Coombs) test showing autoantibodies (usually
anti-IgG) or complement components (anti-C3) bound to the surface of the patient’s RBCs.
Patients with cold AIHA, associated with IM or Mycoplasma infection, usually have IgM
autoantibodies. Treatment of warm AIHA starts with high-dose glucocorticoids (eg, prednisone)
to decrease autoantibody production
This patient's photosensitive rash and symmetric oligoarthritis are highly suggestive of SLE. She
has had 2 miscarriages and now has venous thromboembolism (VTE), including DVT of the leg
and pulmonary embolism. Thrombosis and a history of miscarriages in a patient with suspected
SLE indicates antiphospholipid syndrome (APS). APS occurs at higher frequency in those with
an underlying disease such as SLE, but it can occur independently. The lupus anticoagulant, an
anti-phospholipid antibody, is a pro-thrombotic Ig that causes prolonged PTT in vitro. Specific
tests include the diluted Russell viper venom test and the kaolin clotting time. About 20%-40%
of patients with APS may have thrombocytopenia. However, it is usually mild, does not prolong
bleeding time, and does not require treatment. VWF level and activity are also normal
Autoimmune hemolytic anemia (AIHA) and hereditary spherocytosis (HS) can cause
extravascular hemolytic anemia. A negative family history and positive Coombs test suggest
AIHA; a positive family history and negative Coombs test suggest HS. The peripheral blood
smear in both conditions may show spherocytes without central pallor
PNH is caused by a defect in the cell membrane anchor that leads to complement-mediated
hemolysis with a negative Coombs test but evidence of venous thrombosis and sometimes
episodic intravascular hemolysis
G6PD deficiency is an X-linked hemolytic anemia triggered by medications (eg, sulfa drugs).
Peripheral blood smear shows Heinz bodies.
This patient has back pain, normocytic anemia, and arm pain with an x-ray showing multiple
osteolytic (lucent/dark) lesions. Pathology stems from bone marrow infiltration (fractures,
hypercalcemia, anemia) and monoclonal protein production (renal insufficiency). SPEP detects
elevated serum monoclonal protein (M-spike). Bone marrow biopsy, a more invasive procedure,
can then confirm diagnosis (>10% clonal plasma cells). Bone marrow biopsy is an invasive test
and performed only after gathering more information with peripheral smear (rouleaux
formation), serum free light chain analysis, SPEP, and urine protein electrophoresis
Prostate cancer can cause bone lesions, but they tend to be osteoblastic (sclerotic/white) not
osteolytic (lucent/dark).
Bone pain is a common presenting symptom in MM. In an elderly patient with evidence of
osteolytic lesions on x-ray, MM should always be suspected. The screening tests of choice are
SPEP, UPEP, and free light chain analysis. Diagnosis is confirmed with bone marrow biopsy
Warfarin is an oral anticoagulant that can cause an acquired protein C deficiency, which may
manifest as skin necrosis. However, this patient has not received any oral anticoagulation.
Warfarin is often used for chronic anticoagulation maintenance in HIT but not until argatroban or
fondaparinux are initiated and platelets are >150,000
Cholesterol embolization can occur due to plaque embolization into small arteries following
coronary angiography. Cholesterol embolization classically causes skin findings (eg, livedo
reticularis [mottled erythema], gangrene, cyanosis) that most commonly affect the lower
extremities. With necrotic lesions on the abdominal wall, HIT is much more likely
This patient has hairy cell leukemia (leukemic reticuloendotheliosis), which is a type of
B-lymphocyte derived chronic leukemia. The bone marrow may become fibrotic, thus explaining
why bone marrow aspirates are frequently unsuccessful (dry tap). The cytochemical feature
includes a TRAP stain. The DOC is the purine analog cladribine. It is toxic to bone marrow, and
its adverse effects include neurological and kidney damage
This patient with constipation, back pain, anemia, renal insufficiency, and hypercalcemia likely
has multiple myeloma. MM commonly presents with manifestations related to bone marrow
infiltration (bone pain, fractures, hypercalcemia, anemia) or elevations in serum monoclonal
protein (renal insufficiency). At the time of diagnosis, approximately 25% of patients with MM
have hypercalcemia due to osteolytic bone destruction by the neoplastic plasma cells. Although
hypercalcemia may be asymptomatic with a calcium level <12 mg/dl, fatigue, constipation, and
depression are common. This patient's constipation is likely a result of hypercalcemia. Patients
with suspected MM should have serum/urine protein electrophoresis (M-spike), a peripheral
blood smear (rouleaux), and a serum free light chain analysis. If these screening tests suggest
MM, diagnosis can be confirmed by bone marrow biopsy
This patient, who is exposed to automobile exhaust in an enclosed space, has intermittent
headaches, dizziness, nausea, and polycythemia (elevated hematocrit), most likely due to chronic
CO poisoning. CO is a byproduct of combusting organic matter (eg, oil, gas, wood). CO tightly
binds to Hb forming carboxyhemoglobin which has greater affinity for Hb than oxygen.
Nonsmokers have low levels (<3%) of carboxyhemoglobin (due to normal enzymatic reactions)
but its as high as 10% in smokers. Even though patients at this level are generally asymptomatic,
small additional amounts of CO exposure may cause toxicity manifesting as headache, malaise,
and nausea. Carboxyhemoglobin shifts the oxygen dissociation curve to the left, impairing the
ability of heme to unload oxygen at the tissue level causing tissue hypoxia. The kidney responds
to tissue hypoxia by producing more EPO which stimulates the bone marrow to differentiate
more RBCs. Chronic CO toxicity is a cause of secondary polycythemia. Pulse oximetry does not
differentiate between carboxyhemoglobin and oxyhemoglobin; it cannot be used in the diagnosis
of CO poisoning. Diagnosis is made by arterial blood gas with CO-oximetry
Polycythemia vera is a clonal myeloproliferative disorder that causes erythrocytosis. Patients are
often asymptomatic, but signs of increased blood viscosity (eg, transient neurologic symptoms,
thrombosis), aquagenic pruritus, gout, and increases in all 3 cell lines are common.
Pica is an appetite for items other than food, such as ice, paper products, clay, or dirt that
develops in iron deficiency anemia. It may be present before anemia develops and responds
rapidly to iron supplementation. Pica may also be a manifestation of psychiatric disease; these
patients may eat very odd items, including light bulbs or hair. This patient has a low-iron diet and
likely chronic GI bleeding from the ulcer, which has led to iron deficiency and pica
This patient with a fever, productive cough, and right lower lobe infiltrate likely has bacterial
pneumonia which cause a mild to moderate leukocytosis with a predominance of neutrophils.
This patient's CBC demonstrates a dramatic leukocytosis, primarily with lymphocytes. In an
elderly pt, severe lymphocytosis, hepatosplenomegaly, lymphadenopathy, and bicytopenia
(anemia, thrombocytopenia) suggests underlying CLL. Although patients with CLL have high
WBC counts, their immune systems are abnormal; hypogammaglobulinemia and defects in cell
signaling increase the risk and severity of infections (a major complication of CLL). CLL is
diagnosed by flow cytometry showing a clonality of mature B cells. Lymph node biopsy is not
generally needed for diagnosis. LN biopsy is useful in diagnosing some lymphomas (eg,
Hodgkin lymphoma, non-Hodgkin lymphoma)
EBV serology can be used to diagnose infectious mononucleosis. IM affects young adults and
presents with fever, lymphadenopathy, malaise, and pharyngitis. Although splenomegaly,
lymphadenopathy, and lymphocytosis can occur with IM, leukocytosis is generally mild
(12,000-18,000) and cytopenias are rare.
HS is due to genetic defect in the ankyrin gene and results in abnormal RBC plasma-membrane
scaffolding proteins thus they are less deformable than normal RBCs and more prone to splenic
sequestration. Spherocytes can be seen in other conditions (eg, G6PD deficiency, AIHA), but
they do not have the characteristic lab findings seen in HS. Similarly, Coombs test is negative in
HS but positive in AIHA. Patients present with the classic triad of hemolytic anemia, jaundice,
and splenomegaly. Patients are at increased risk for bilirubin gallstones and parvovirus B19
infection causing aplastic crisis. Lab studies show elevated MCHC due to membrane loss and
RBC dehydration. Other findings include reticulocytosis, normal or slightly low MCV, and
elevated RBC distribution due to the various forms (eg, reticulocytes, spherocytes, normal
RBCs) in the peripheral smear. Diagnosis is made clinically but can be confirmed with specific
tests (eg, osmotic fragility test, eosin-5-maleimide binding test). Treatment is supportive (eg,
folic acid, transfusions for severe anemia). Splenectomy improves anemia and reduce gallstone
risk, but it does not change the increased MCHC in the RBCs.
Preconception counseling and testing identifies couples who are at risk for hemoglobinopathies
that might affect their offspring during pregnancy or after birth. The most important initial
screening test in a female is a CBC. Based on the results, other appropriate screening tests should
be obtained. Testing of the patient's partner is offered if an abnormal Hb level is diagnosed in pt
Patients with B-thalassemia trait usually are asymptomatic with mild anemia, disproportionately
high RBC count, low MCV, and Hb >10. B- thalassemia minor is commonly confused with iron
deficiency anemia. However iron deficiency anemia usually has low RBC count and rarely
becomes microcytic until the Hb is <10. No therapy is required for B-thalassemia minor.
Folic acid is indicated in folate deficiency and also in severe forms of hemolytic anemia (HS,
PNH, sickle cell disease) and B-thalassemia major due to increased demand for erythropoiesis
Erythropoietin is the treatment of choice for anemia of chronic disease, anemia associated with
chronic kidney disease and transient bone marrow failure after chemotherapy or bone marrow
transplantation. It has no role in thalassemia.
Prednisone is the DOC for AIHA. A high number of spherocytes is seen on peripheral smear
Hydroxyurea decreases the frequency and severity of pain crises in patients with sickle cell
anemia by increasing HBF levels.
Rapid deterioration following blood transfusion is most consistent with an anaphylactic reaction.
Anaphylaxis typically occurs seconds to minutes after transfusion. IgA-deficient individuals are
at risk due to the presence of anti-IgA IgG antibodies. Patients present with difficulty breathing
(wheezing or bronchospasm), angioedema, and hypotension that progresses to respiratory failure,
loss of consciousness, and shock. Transfusion must be stopped immediately and IM epinephrine
administered. Histamine blockers and glucocorticoids should be administered as well.
Hemodynamic and respiratory support with vasopressors and mechanical ventilation may be
required. Future transfusions should include IgA-deficient plasma and washed red cell products
ABO mismatching is rare, but it can still occur due to clerical errors and cause acute hemolytic
transfusion reaction. Patients present with fever, chills, flank pain, and hemoglobinuria within an
hour of transfusion. This can progress to renal failure and DIC. Rapid onset of shock and
respiratory failure suggest anaphylactic reaction to transfusion.
Febrile nonhemolytic transfusion reaction, the most common adverse reaction to transfusion,
occurs within 1-6 hours of transfusion. During blood storage, leukocytes release cytokines that,
when transfused, cause transient fevers, chills, and malaise. Respiratory distress and shock are
not features of this reaction.
Warfarin-induced skin necrosis occurs within the first few days of warfarin therapy (usually at
large loading doses). Warfarin inhibits production of vitamin K-dependent clotting factors II, VII,
IX, and X. It also inhibits production of the natural anticoagulants proteins C and S. This
decreases protein C anticoagulant activity to 50% within the first day while levels of
procoagulant factors (II, IX, and X) decline more slowly, leading to a transient hypercoagulable
state. This increases the risk for venous thromboembolism and skin necrosis, especially in
patients with underlying hereditary protein C deficiency. Skin lesions typically occur on the
extremities, breast, trunk, and penis and marginate over a period of hours. If left untreated,
affected areas become edematous, purpuric, and ultimately necrotic. Treatment involves
immediate cessation of warfarin and administration of protein C concentrate
HIT is caused by autoantibodies to platelet factor 4 (PF4) complexed with heparin, and is
characterized by thrombocytopenia, arterial or venous thrombosis, and necrotic skin lesions at
heparin injection sites within 5-10 days of therapy
Factor V Leiden mutation increases the risk for venous thromboembolism (DVT or PE). It also
increases the risk for cerebral, mesenteric, and portal vein thrombosis. These patients are treated
with anticoagulants, and initiation of warfarin should not cause unusual hypercoagulability
Antithrombin III is a vitamin K-independent inhibitor of the clotting cascade. Antithrombin III
deficiency predisposes to thrombus formation.
Factor VII deficiency causes a bleeding diathesis characterized by bruising and hemorrhage
This patient was initiated on LMWH (enoxaparin) due to an acute DVT. A drop in platelets by
>50% or a new thrombus within 5-10 days of heparin initiation should raise suspicion for HIT.
(HIT may manifest sooner in patients previously exposed to heparin). Heparin induces a
conformational change to a platelet surface protein (PF4), which creates a neoantigen. The
immune system responds by forming an IgG autoantibody (HIT antibody) that coat the surface of
platelets and the RES (largely the spleen) removes antibody-coated platelets, causing mild to
moderate thrombocytopenia (mean nadirs of 60,000). Severe thrombocytopenia (<20,000) is
uncommon. Thrombus- HIT antibodies activate platelets, resulting in platelet aggregation and the
release of procoagulant factors increasing the risk for arterial and venous thrombus. Venous
thrombosis in the leg veins, cardiac vessels, and skin (at the site of heparin injection) is common.
Arterial thrombosis may occur in the heart, limbs, or CNS. If HIT is suspected, all heparin
products should be discontinued immediately and anticoagulation should be initiated with a non-
heparin medication (eg, argatroban, fondaparinux). Diagnosis is confirmed by functional assay
(eg, serotonin-release assay which is the gold standard) or high-titer immunoassay of the blood
Heparin causes conformational change on surface of platelet which create a new antigen.
Immune system responds by creating IgG antibody that coat surface of platelets and then the
spleen removes these antibody coated platelets resulting in thrombocytopenia. Thrombus-HIT
antibodies activate platelets resulting in platelet aggregation and release of procoagulant factors
thus increasing risk of arterial and venous thrombosis.
G6PD deficiency is characterized by acute hemolysis after ingesting primaquine or sulfa drugs.
Labs show low Hb, increased indirect bilirubin, increased LDH, and decreased haptoglobin.
Coombs test is negative. The peripheral blood smear shows bite cells with Heinz bodies
(denatured Hb precipitates). G6PD activity can be used as a screening test, but has reduced
sensitivity during an acute hemolytic episode. Most erythrocytes that are severely G6PD
deficient are hemolyzed early in a hemolytic episode, and reticulocytes, which have normal
G6PD levels, are circulating at abnormally high levels. It is best to wait 3 months before
retesting. Treatment is supportive, with removal or management of the underlying insult
An 80-year-old man with advanced prostate cancer and bony metastasis is complaining of severe
back pain that has been progressively worsening for the past two weeks. His pain is so severe
that it restricts him from playing golf. Eight months ago, he underwent orchiectomy, after which
he was free from bone pain until now. Physical examination reveals tenderness at two sites in the
lumbar region. Radionuclide bone scan shows an increased uptake in these areas. Which of the
following is the most appropriate next step in the management of this patient?
Radiation therapy is the most appropriate for the management of progressive pain in a patient
with prostate cancer and bony metastases after androgen ablation (orchiectomy). Focal external
beam therapy is an excellent choice in this patient, as metastasis is localized to few sites.
Radiation therapy is useful in managing bone pain in patients with prostate cancer who have
undergone orchiectomy
Etidronate disodium and other bisphosphonates may be useful in patients with bony metastasis as
it reduces bone resorption. It is useful in controlling chronic pain from bony metastasis, but its
onset of action in alleviating acute bone pain is not as rapid as radiation therapy
Some anti-epileptic drugs including phenytoin, primidone and phenobarbital can cause
megaloblastic anemia that is usually mild due to impaired absorption of folic acid in the small
intestine. Folic acid supplementation can effectively prevent this condition. Other drugs can also
cause folic acid deficiency. Trimethoprim and Methotrexate inhibits dihydrofolate reductase and
in high doses can cause megaloblastic pancytopenia. Folinic acid (leucovorin) is indicated to
reverse the chemotherapeutic anti-folate effect of methotrexate.